Bugrov, Nikolsky - A Collection of Problems (Higher Mathematics) - Mir - 1984

You might also like

Download as pdf or txt
Download as pdf or txt
You are on page 1of 203

Higher Mathematic/

YaS.BUG RO l/ S.M. NIKOLSKY

o collection
or
problem/

££ MIR PUBLISHERS MOSCOW


ABOUT THE BOOK

This collection of about 1200 problems


has been compiled for the following
three textbooks by the same authors:
Fundamentals o f Linear Algebra and
Analytical Geometry. Differential and
Integral Calculus. Differential Equa­
tions. Multiple Integrals. Series. Theory
Functions o f a Complex Variable, thus
completing a course in higher mathe­
matics for engineering students and
forming a four-book series entitled
“Higher Mathematics”. Academician
S. Nikolsky is the author of the
two-volume textbook A Course o f
Mathematical Analysis issued in Eng­
lish by Mir Publishers in 1977 and re­
printed in 1981.
All the problems are provided with ans­
wers, some of the problems are sup­
plied with hints. The book contains
many worked problems.
At the beginning of each section refer­
ences are given indicating the chapters
and sections of the above mentioned
books where the corresponding theo­
retical material can be found.
Higher IWothe—otic/
BblCLUAfl MATEMATMKA

A. C. ByrpoB
C. M. HMKOJIbCKMM

3 A flA M H M K

Mockbo kl3AaTp/ibCTBQ «HayKa»


Higher mathematic/
YaS.BUGROI/
S.M. NIKOLSKY

o collection
of
problem/
Translated from the Russian
by
Leonid Levant

MIR PUBLISHERS
/MOSCOW/
First published 1984
Revised from the 1982 Russian edition

Ha QH2AUUCKOM H3 blKC

© H3flaTejibCTBo «Hayica», rjiaBHaa peaaKUHn <J>H3HKo-MaTeMaTHHecieo0


jiHTcpaTypbi, 1982
© English translation, Mir Publishers, 19X4
Contents

Preface .............................................................................................................. 9

Chapter 1. Introduction to Analysis

Sec. 1.1. Real Numbers. Sets ......................................................................... 11


Sec. 1.2. Limit of a Sequence ....................................................................... 12
Sec. 1.3. Functions. The Limit of a Function ............................................. 14
Sec. 1.4. Derivatives......................... 16

Chapter 2. Integrals

Sec. 2.1. Indefinite Integral ............................................................................. 25


Sec. 2.2. Definite Integral ................................................................................. 28
Sec. 2.3. Applications of Definite Integral ................................................... 29
Sec. 2.4. Improper Integrals ............................................................... 32

Chapter 3. Fundamentals of Linear Algebra and Analytical


Geometry

Sec. 3.1. Determinants and M atrices............................................................. 33


Sec. 3.2. Systems of Linear Equations ....................................................... 34
Sec 3.3. Vectors ............................................................................................. 35
Sec. 3.4. Dividing a Line Segment in a Given Ratio ................................. 36
Sec. 3.5. The Straight Line ............................................................................. 36
Sec. 3.6. The P la n e ........................................................................................... 37
Sec. 3.7. A Straight Line in Space ............................................................... 38
Sec. 3.8. Orientation of a System of Vectors. The Vector Product of Two
Vectors. Triple Scalar Product ..................................................... 38
See. 3.9. Dependent and Independent Systemsof Vectors 43
6 Contents

Sec. 3.10. Linear Operators. The Basis ....................................................... 43


Sec. 3.11. Linear Subspaces ........................................................................... 47
Sec. 3.12. Self-adjoint Operators. Quadratic Forms ................................. 49
Sec. 3.13. Second-order Curves •.................................................................... 50
Sec. 3.14. Second-order Surfaces ................................................................... 52

C h ap ter 4. Functions of Several Variables

Sec. 4.1. Basic Concepts ................................................................................ 57


Sec. 4.2. Limit of a Function. Continuity ................................................ 58
Sec. 4.3. Partial Derivatives. Differentials .................................................. 60
Sec. 4.4. Partial Derivatives and Differentials of Higher Orders ........... 61
Sec. 4.5. Tangent Plane and a Normal to a Surface ........... ....................... 61
Sec. 4.6. Taylor’s Formula ............................................................................ 62
Sec. 4.7. Extrema ........................................................................................... 62
Sec. 4.8. Implicit Functions. Conditional E xtrem um ................................ 63

C h apter 5. Series

Sec. 5.1. Number S eries.................................................................................. 64


Sec. 5.2. Functional Series ......................................................... 67
Sec. 5.3. Power Series ................................................................................... 67

C h apter 6. Differential Equations

Sec. 6.1. General Concepts .......................................................................... 69


Sec. 6.2. First-order E quations...................................................................... 69
Sec. 6.3. Metric Spaces. Contraction Operators. Solution Existence
T heorem ............................................................................................. 70
Sec. 6.4. Equations Not Resolved with Respect to Derivative. Singular
Solutions ........................................................................................... 72
Sec. 6.5. Reducing the Order of a Differential Equation ......................... 73
Sec. 6.6. Linear Equations with Constant Coefficients.............................. 73
Sec. 6.7. Euler’s Equation. Equations with Variable Coefficients............. 75
Sec. 6.8. The Method of Variation of Constants 76
Sec. 6.9. Systems of Differential Equations ................................................ 76
Sec. 6.10. Solving Equations with the Aid of Power Series ...................... 77
Sec. 6.11. Stability in the Sense of Lyapunov ............................................. 78
Confetits T

Chapter 7. Multiple Integrals

Sec. 7.1. Integrals Depending on a P aram eter............ 79>


Sec. 7.2. Multiple Integrals ..................................; -- ------ 80
Sec. 7.3. Change of Variables in a Multiple'Integral*---- ---- . . . . . . . . 81'
Sec. 7.4. Application of Multiple Integrals .................................. . 82'
Sec. 7.5. Improper Integrals ........................................................................... 841

Chapter 8. Vector Analysis

Sec. 8.1. Line Integrals of the First K in d .................................................... 86


Sec. 8.2. Integral of a Vector Along a C u rv e.............................................. 88
Sec. 8.3. Potential. The Curl of a Vector .................................................... 89
Sec. 8.4. Exact Differential Equations of First O rd e r................................ 90
Sec. 8.5. Green’s F o rm u la............................................................................. 90
Sec. 8.6. Surface Integral of the First Kind .............................................. 91
Sec. 8.7. A Vector Flux Through an Oriented Surface (Surface Integral of
the Second Kind) ............................................................................. 93
Sec. 8.%. Gauss-Ostrogradski Formula ........................................................ 97
Sec. 8.9. Stokes’ Formula .............................................................................. 98

Chapter 9. Fourier Series and Fourier Integral

Sec. 9.1. Trigonometric Series ....................................................................... 100


Sec. 9.2. Fourier Series ................................................................................... 100
Sec. 9.3. Orthogonal Systems of Functions ................................................. 102
Sec. 9.4. Fourier Integral ............................................................................. 104

Chapter 10. Equations of Mathematical Physics

Chapter 11. Functions of a Complex Variable

Sec. 11.1. General Concepts ......................................... 108


Sec. 11.2. Limit of a Function. Derivatives ................................................. 109
Sec. 11.3. Cauchy-Riemann Conditions. Harmonic Functions ................ 110
8 Contents

Sec. 11.4. Simplest Conformal M appings..................................................... I l l


Sec. 11.5. Integration of Functions of a Complex Variable .................... 112
Sec. 11.6. Cauchy’s Integral Formula ......................................................... 114
Sec. 11.7. Series ir^a Complex Domain ....................................................... 115
Sec. 11.8. Isolated Singularities. Residues ................................................... 116
Sec. 11.9. Computing Integrals with the aid of Residues............................ 118

Chapter 12. Operational Calculus


Sec. 12.1. Transforms of Simplest F unctions............................................... 122
Sec. 12.2. Finding Object Function by Its Transform ................................ 123
Sec. 12.3. Applications of Operational Calculus ........................................ 124

Answers .......................................................................................................... 126


Preface

This collection of problems has been compiled to suit the text­


books entering into our series “Higher Mathematics”. Tljese
textbooks are referred to in the present collection a s:
fl] — Differential and Integral Calculus;
[2] — Fundamentals of Linear Algebra and Analytical Geometry;
[3] — Differential Equations. Multiple Integrals. Series. Theory of
Functions of a Complex Variable
Indicated at the beginning of each section of the present book
are the chanter and section from the above mentioned textbooks
where the relevant theoretical material can be found.
As a rule, each section contains a minimum number of problems
which corresponds to the number of teaching periods assigned to
study a certain topic. The odd-numbered problems may be
recommended to be solved in auditorium, while the even-num­
bered problems must be solved by students at home independently.
The problems found in textbooks [l]-[3] may also be used for
practical studies. These are not contained in the present Collec­
tion.

Yakov S. Bugrov
Sergei M. Nikolsky
C hapter 1
INTRODUCTION TO ANALYSIS

Sec. 1.1. Real Numbers. Sets


Using the method of mathematical induction, prove the fol­
lowing relationships:
1. 1-+-2+3+ . . . +n = n (n + 1)/2.
2. l 2+ 2 2+ . . . +H2 = /!(/!+ 1)(2/1+ 1)/6.
3. (1 + „y) w 1+ /ix, —1.
. 1 3 2/j-l 1
4 . ------ . . . --------- < .
2 4 2n y /ln + 1
To solve the below problems, it is necessary to study Chapter 1
from [1].
5. Let the set A consist of the youths of a given group, and the
set B o f the girls of the same group. Find A\JB, AO B, A\B . Also,
consider the case when A or B is an empty set.
6. Let A = {2ai}, B = {2/z+1}. Find A +B , AB, A\B (n natural).
7. Which number is greater, a or b: a = 1.(1234512), b =
= 1.(12345); a = 1.(12302), 6 = 1 .(1 2 3 ); a = 1.(123412), b =
= 1.(1234)?
8. Find out to what number a the sequence of real numbers
ai = 0.1010101010...,
02 = 0.1100110011...,
03 = 0.111000111000...,
04 = 0.111100001111...,

an = 0 . 1 1 .. .1 0 0 . . . 0 1 . . . 1 0 . . . 0 . . . ,
n tim es n times

is stabilized.
9. Find the sum of real numbers a = 0.(12) and b = 0.(13).
10. Given the sets A = [2, 5] and B = (3, 6). Find A + B , AB,
A\B.
15 A Collection o f Problems

11. Solve the following inequalities:


(a) |* + 3 | < 0 .1 ; (b) \x —3\ ^ 1 0 ;
(c) | x | > | x + 3 | ; (d) \3x—1 1 < \ x - \ |;
x -2
(e) 3x+ 1 1.

12. Which of the two numbers is greater: a or ( —a)!


13. Let a ^ 0. For what numbers b do the following relations
take place
(a) \a+b\ = \a\ + \b\;
(b) \ a - b \ = \a\ + \b\;
(c) \a+b\ < \a\ + \b\\
(d) \a—b\ < \ a\-\-\b\l
14. Find the modulus of the given number: (a) In (1 /e);
(b) sin(3jr/2), (c) cos (7^/4).

Sec. 1.2. Limit of a Sequence


(See [1], C hapter 2)
15. Prove that
lim — —= 1,

and determine for every e > 0 the number n0 = n0 (e) such that
n± l < e if n > Hq.
n
Fill in the table:

e 0.1 0.001 0.00001 ...

n0

In problems 16 to 19 find the indicated limits.

16. lim . 17. lim ------——- (0 < a < 1).


^ o o ^ 1 /i^oo n + l v

18' ,1i n.( i+?+ ' +V)-


19. lim ( i ! + ' + ... +
Ch. I. Introduction to Analysis 13

20 . Prove that the variable a* is an infinitesimal if


1
OC/i —
w3+ 1 '\/r iz+ 1

2l Prove that the variable is an infinitely large quantity if


\ fin = ( - l )-«*; fin = 2^ i fi„ = ln(n+1).
22. Will the sequence = «<-1)n/2 be infinitely large?
In Problems 23 and 24 prove the given equalities.

23. lim ( \^3n-h 10—a /3 « ) = 0.


n —►oo
3«2+ 5 3^
24. lim
2«2+ 3 /i+ 1 2 •

Using the theorem on existence of the limit of a monotone sequence,


prove that the following sequences are convergent (Problems 25
and 26):

25

, _ 2 3 4 5 "+ 1
ZO. x„ - J • 3 - 5 • 7 ••• 2rt—l •

27. Find the largest term of the following sequences:


_ n2 _ \/n + 1
— 2^ ’ ~ 10+ n -

28. Find the smallest term of the following sequences:

x„ = + 1 x» = n2- 9 n - 10.

29. Find inf x„, sup x„(n € N), lim x,„ lim x„ if

,„ = 1 -1 ; x„ = +— f-- (« € N ).

30. Which numbers are the partial limits of the sequence


1 —
19 2 9 —1 1 —
19 19 3 9 —1 1 —
19 19 4 ’ —1
!> 1
! » • • • -9

A partial limit of an arbitrary bounded sequence is understood


as the limit o f its convergent subsequence. The existence of such
subsequences in a bounded sequence is implied by the Bolzano-
Wcicrstrass theorem.
14 A Collection o f Problems

Applying Cauchy’s test for convergence, prove that the following


sequences are convergent (Problems 31 to 33):
n
sin l 2 ( sin 22 , , sin n2 sin k 2
31. xn — 2 ^ h ' ** ' 2*~ T " *

cos &!
32. 33. =
I
k=l * ( £ + !)•
*

Sec. 1.3. Functions. The Limit of a Function


(See [1], C hapter 3)
Find the domain E o f definition of the function y = f ix ) and th e
image E\ = f ( E ) o f the set E with the aid of the function / (Prob-"
lems 34 and 35).
34. y = . 35. y = \ ^ 2 + 3 x - x 2.
36. Find /( 0 ) , f ( x + 2 ) , / ( i/x), f ( x ) + 1, 1/ / ( * ) if
\ 1 ~~X*

In Problems 37 to 41 find the graph of the given function.

3 1 .y = i x - 2 x \ 38. y = — .

39. y = - x 2+ 2 x - l . 40. y = — . 41. y = ^ ± \ .


J ' 1+x J 4 x -3
42. Determine the lower and the upper bound to the set of values
of the function/ (x) if
f { x ) = x2 on [—2, 5]; <p(x) = x + — on (0, 3].

Hint. On the set (0, 3] <p(x) > 2.


43. Construct the graphs o f the functions
f(x) = sup {sin/}; <p(x) = inf {sin/}.
0 f ==x 0 ;c
In Problems 44 to 48 find the limits of the indicated functions.
x 2—1 x 2—1
44. (a) lim (b) lim
x -* 0 2x2—x —1 ’ *-►1 2x2—x —1
x* -\ ( l+ * ) 3- (1 + 3 x )
(c) lim (d) lim
X -*• « x —+ 0 x* + 3x3
Ch. 1. Introduction to Analysis 15

t \ V Vx + 13~2V x i-l . /*x 1- \A -6 + 2 #


(g) l™ ^------ jc2—9 ; (h) lim —-r -g — >
jc—
►3 x y X-+-2 x 0
^ -V a+ v * ^ ,
(i) lim v
\ jc —
►a V x 2- a 2
(j) lim [sin \ / x + i —sin \ / x ] ,
X —►+oo
/xi* sin 3jc /ux r 1-cos2jc tan Ax
45. (a) l i m ------ ; (b) l i m ------ ^— ; (c) lim
jc —►0 x x -+ 0 x

s in y *
46. lim
A . j: — o+o
%
4% (a) lim ( l + (b) lim (1 + 3*)1'*;
X —►oo ' X / JC—►0

(c) lim (sin x)^nx;


x -► n!2

48. (a) lim '“4 + ^ ( b) lim f r f - («>0)-


JC-+-0 x b

In Problems 49 to 56 investigate, the given functions for conti­


nuity, represent the functions graphically, and determine the cha­
racter o f the points of discontinuity.
;c2- l
X ^ l ,
4 9 ./( x ) = | x - l | . 50 . / ( x ) = 1 x - 1 ’
A , x — \.
1+5
51. / ( x ) = sgn (x2—2 x —3). 52. y —~
+*»

53. y = 54. y = sgn (cos x).


1+ jc'
[ 2*, 1, [ cos x, x^s 0,
55- y = I 0 0 56. / ( x ) = i
[ 2 - x, 1 < x <= 2. ^ [ a + x, x > 0.
In Problems 57 and 58 prove that the functions / (x) are uni­
formly continuous on [<a, 6], i.e. V £ > 0 there is 6(e) > 0 (indepen­
dent of the points of the interval) such that
l / ( * i ) - / ( * 2 ) | < e,
whenever
| x i — x 2 | < 6.

f x2, 0 ^ x < 2,
57. / ( x ) = | *2’
I 20 - 8 x , 2 ^ x < 3 ,
58. / ( x ) = x3, 0 ^ x ^ 2,
16 A Collection o f Problems

59. Find the inverse o f the function

y = w -b c ^o ).
Let a -►0. Separate the leading term o f the form Axm (Problems
60 and 61).
60. / ( a ) = 3A+A4.61. / ( a ) = y / l + x —y / l —x.

Sec. 1.4. Derivatives


(See [1], C hapter 4)
62. Find /'(0 ), /'(2 ), if f (x ) = 2 - 2 a + a 3.

63. Find /'(0 ), / ' ( l ) , if /(a ) = x arc sin ■— .


In Problems 64 to 71 find the derivatives of the given functions.
2x
64. y =
I-* 2'
/u , t i l
65. (a) y = x + y / x ; (b ) y = - I + , -;
* y x yx

^ y (1 —x ) 2 (1 +A-)*;
(d) y = x ^ l + x-;

(f) y = ~ \ / x + -y/x+ \ / x ;

(g) y = V x 2— - ; (h) v — tan ——cot — ;


Vx 2 2
sin * —x cos * .
(i) y = c o s * + * s in * ’ (j) y = ^ x ' (h) y = 2*“ *;
(1) y - e‘x+ e e'x; (m) y = xa°+ a xa (a > 0);

(n) y = arc sin ; (o) y = arc cos y / l —x2.

66. y = tan a |- * tan3 x. 67. y = e x\

68. (a) y = sin a2; (b) y = sin2 a; (c ) y =*sin3 a7;


(d) = cos (sin a); (e) y = cos a2;(f) y = cos2 a4.
69. (a) y = arc sin (a /a); (b) y = arc tan (a /a);
(c) y — In (a + y / a2+ a 2); (d) y = arc sin (sin a);
(e) y = arc cos (a la), (f) y = e*3+x,
70. y = In tan (a / 2).
Ch. 1. Introduction to Analysis 17

71. (a) y = x arc tan x; (b) y = In3 x2;

(c) y = In-(In (In x ) ) ; (d) y = — In ~ ;

(e) y = v V f - l - M l + V ^ + l ) ; ( 0 y = !n ta n (y + ^ -);

(g) y — -^-0n3;c+ 3 ln2x + 6 In x + 6 );

(h) y = - | ( l - \ / T + ^ ) 2+ 3 In (1 + %/T+x 2) ;

(i) y = '\Jx—axc tan \ / x ; (j) y = arc tan ;

n n 1 i x * - x 2+ l V3
(k) y = — I n ----------- : arc tan
12 (*2+ l)2 2 a/3 ' “ 2*2—1
JC®
(1) y = j -+ - i2-+arc tan x6; (m) j = arc tan (tan2 x);

(n) y = x arc tan x —0.5 In (1 + x2) —0.5 (arc tan x)2;


(o) y = arc tan ( x + \ / l + x2).
(p) There takes place the formula
<hi(x) ■ ai„(x)

ain(x) n fik-i, i(x) .. ■ ak_Un(x)


d_
dx =z 4i(x) ■ 4n(x)
k=l Ok+ u W • • • &k+l, n(x)
ani{x) &nn(.X)

a„i(x) • ^nnijX)
where the elements of the determinant a,;(x) are differentiable
functions. Hence, the derivative of a determinant o f order n is
equal to the sum o f n determinants o f order n each of which dif­
fers from the original determinant by that the appropriate row in it
is replaced by the row made up from the derivatives of the ele­
ments o f the replaced row.
Prove the differentiation formula for determinants of the second
and third order.
In Problems 72 and 73 find the derivatives and construct the
graphs o f the given functions and their derivatives.
1 —x, —2 < x < 1,
72. y = (1 —x )( 2 —x), 1 ^ x ^ 2,
- ( 2 —x), 2 < x < 4,
18 A Collection o f Problems

x, x < 0,
73. y = {
ln (l + x), x s* 0.
Find the logarithmic derivatives (i.e. y'ly) of the indicated func-
tions y (Problems 74 and 75).
—x
74. y = x y / \
T x'
75. y = cosh2 x.
In Problems 76 to 79 find the derivatives of the indicated hyper­
bolic functions.
76. (a) y = sinh(x2+ 1); (b) y = sinha x°.
77. y = cosh2 (x2+ x + 1).
78. (a) y = tanh2 x; (b) y — tanh x2.
79. (a) y = tanh (In x + 1).; (b) y = Arsinh x;
(c) y = Arsinh (x-F y / \ + x2) ; (d) y = In sinh x;
(e) y = cosh In x; (f) y = etanh x;

(g) y = (sinh x)cosh v (x > 0);

(i) y = tanh y - c o t h — .

80. For the fu n ction /(x) = x2-l-x + 1 determine the differential


and the increment at the point x = 1 for Zlx = 0.1.
In Problems 81 to 84 find the differentials of the given functions.
81. d(xe*).
82. d(sinh x).
83. c/(sinh x —x cosh x).
84. d(ln (1 —x2)).
85. Find the second-order derivative of the following functions:
(a) y = e~xl = exp ( —x2); (b) y = x y / \ + x2.
86. Let there be given the Wronskian
yi(x)
y'xW ■• • y ’n(x)
W{x) =
Vi"-1)(x) .
where the functions ji(x ), . . . , yn(x) are continuous on (a, b)
together with their derivatives up to order n inclusive.
Ch. 1. Introduction to Analysis 19

Prove that
y iW • • yn(x)
X(x) ■• • y'n(x)
dW __
dx ~
/ r 2\ x ) . . . * - « ( * )
yV W • • • yt\x )
87. y = x5, find dAy,
88. y = ex In x, find d3y.
In Problems 89 to 92 find the derivatives y'x and y'J o f the
functions represented parametrically if :
89. x = 2 t —t2,y = 3t—t3. 90. x = 2 cos t , y = 2 sin t.
9 1 . x = /'(* ), y =
92. x = sin t, y = 1—cos t.
93. (a) Write the equations of the tangent and the normal to the
curve y = 2 + x - x 3 at the point A = (2, —4).
(b) Find out whether or not the graphs o f the functions
y = sinh x and y = ln ( l+ 2 x ) have a common tangent at the
point (0, 0).
The angle between two curves y = /i(x ) and y = /*2(x) at the
point o f their intersection with the abscissa x = x0 is defined as the
angle q>between the tangents to the curves at that point. Therefore

tan ?r = tan Yyj 1 + tan <3pl tan <p2 1 + /1'(x0)/2(^0)


where (pu <pz are the angles formed by the indicated tangents with
the x-axis (see Fig. 1).
94. At what angle do the curves y = sin x and y = cos x
(0 < x < it) intersect?
i+*
95. At what angle do the curves y = x* and y = x1-* (O' 1)
intersect at the point (1, 1)?
96. At what angle does the curve ^ /iW
y = In ( l + ( x / \ / 3 )) intersect the
x-axis?
97. (a) Check the validity of
Rolle’s theorem for the function
y = ( * - l ) ( x - 2 ) on [1, 2].
(b) The polynomial P4(x) = x4—
—x3+ x 2—x has the root x = 1.
Prove that the polynomial — Pi{x)
has a real root belonging to the open interval (0, 1).
20 A Collection o f Problems

(c) Prove that all the roots o f the polynomial P„(x) =


dn
= ^ (1 —x2)" are real and belong to the interval ( —1, 1).

98. Check the validity of Lagrange’s theorem f ( b ) —f(a) =


= f '( c )( b —a) for the function
(a) y = 1+ x + x
(b ) / ( x ) = & + A x2+ B x + C 9 where A , B , C are real numbers;
(c) f (x ) = Ax3+ B x + C ; on [0, 1]. Find the point c,
99. Prove that if a function / (x) has a bounded derivative on
(a, b)(\f'(x)\ =s M), then
(a) f (x) is uniformly continuous on (.a, b).
(b) If a and b are finite numbers* then / (x) is bounded on (a9 b).
Hint . Let x be an arbitrary point, and let Xo be a fixed point
within (a, b). Then
|/(X )| = |/( x ) - /( * o ) + /( * o ) |
36 \f(x )-f(x o ) I+ \f(x0) I = I/'(c) I | X - Xo I+ |/ ( x 0) I,
where c is found between the points x and x0.
(c) If the interval (a, b) is infinite, then the function / (x) may be
unbounded. Consider the function / (x) = In x on (1, «>).
100. Determine the intervals of monotonicity for the functions
(a) y = 3 + x —x2; (b) y = 4x—x4.
If a function <p(x) is continuous on [a, b] and has a positive
(negative) derivative on (a, b)9 then it is increasing (decreasing) on
[a, b].
This fact can be utilized when proving inequalities.
For instance, the function <p(x) = ex—1 —x is continuous on
[0, oo). It is increasing on [0, <»), since <p'(x) = ex—l > 0 on
(0, «,). Further, 99( 0 ) = 0, therefore
ex—1—x > 0 , V * € (0 , <*>).
On ( — 00 , 0) the function<p(x) is decreasing, therefore ex—1 —x >
> 99( 0 ) = 0. Hence, V* ^ 0
ex > 1+ x.
In Problems 101 and 102 prove the indicated inequalities.

101. x — < sin x < x (x > 0).

102. cos x > 1— - (x > 0).


Ch. I. Introduction to Analysis 21

In Problems 103 to 109 compute the indicated limits using


L’Hospital’s rule.
sin i- tan 2*
103. lim rsr^ . 104. l i m -------
*-*osm b x .r j .s J x->-o *
arc sin 2 x—2 arc sin v tan 2.v - sin 2x
105. lira 106. lim
x -►o x —*■0
-V2 I 2.v
107. lim 4v- | l * 108. lim .\Y.
x |0
sin x \ 1i*'
m ( a ) h m ( ^ p S; (b) l i m ;
x -+ o

, N ,. sinh ax tanh 3*
(d) lim
<c> J ™ s m ; tan *
ta n h * -*
(e) lim •vcotf - 1 ( f) lim i— ;
x —►0 x v 7 v_ « * —sinhx
tan 3.\
(g) lim
tan x
(h) lim (coth .v— 1);
x - * n il x -> o ' x*

(i) lim xl ~x\ (j) lim (in —V ; (k) lim (tanhx)*}


X -v l X -* -+ o ' X ' X —► + OO

(m)

(n) lim i (n—an integer); a&\


x —►0 x

(o) lim V l T ^ - V I T f r (m, n—integers);


x -+ 0 x

, •. y/x —1
(p) lim — — ;

(q) lim x + \ / x + \ / x —'s/x^. Hint. Multiply and


divide by the conjugate expression;
ta n * -s in *
(r) lim (1 —*) tan (s) lim
X —►1 2 ** 5 X -v 0 sinJ *

110. Is it possible to apply L’Hospital’s rule in


* —sin *
lim
* + sin *

'' In Problems 111 to 114 write the expansion of the given functions
in powers of x.
22 A Collection o f Problems

111. / ( x ) = tan x up to the term x5.


112. / (x) = e2x~x~up to the term x3.
113. y = In cos x up to the term x4.
114. y = sin (sin x) up to the term x3.
In Problems 115 to 118 find the indicated limits using the expan­
sion by Taylor’s formula.
... *• c o s * -e x p (-,v 7 2 )
115. lim ---------- —-----—
.r + o *4 sin x j

jc-sinh x 1 —cosh x 2
117. lim 118. lim
jc -► 0

119. Applying Taylor’s formula compute the following num­


bers:
(a) e with an accuracy to 10”6;
(b) sin 1° with an accuracy to 10~r>;
(c) \ / 5 with an accuracy to 10"3;
(d) In 2 and In 3 with an accuracy to 10~5 (see [1], Sec. 9.14,
(8 )).
120. Investigate the following functions for a local extremum:
(a) y = 2 - x - x2; (b) y = | x \;
(c) y = 2x2—x4; (d) y = x + -- ;

(e) y = e* sin x (0 ^ x ^ 2jt); (f) y — l/( 4 + x 2);


(g) y = x/(l + 4 x 2); (h) y = x3- 6 x 2+ 9 x - 4 ;
(i) y = cos x + 0 .5 cos 2x;
(j) y = sin x + 0.5 sin 2x.
121. Find the greatest and the least values of the given function:
(a ) y = 2* on [0, 5]; (b) y = x + — on [— , loj.

122. Find the distance from the curve y = x2 to the straight line
y —x + 2 = 0.
123. Find sup and inf of the following functions:
0 ^ x ^ 1, ^+ x 2
(a ) y =i1 4 —2x, 1 < x < 2;
on (0, «>)

124. Find the intervals of concavity and points of inflection for


the following functions: (a) y = 3x2—x3; (b ) y = exp ( —x2).
Ch. 1. Introduction to Analysis 23'

125. Find the asymptotes to the graphs of the following func­


tions :

(a) y = x+"~-; : (b) y '= ;;

(fc) )y = expf ( - x2) j (d) y = In (1 + eK):


1161 Graph the^below functions on having carried out a complete
investigation of their* bfehaviour1(extrema;, points off inffection1, *
zeros o f function, direction of concavity, asymptotes)::
x —3 .
(a) y = (b ) y = ~e- '
Vf+P5 \+ x
127. Graph the functions represented parametrically (see Sec.
4.22 [1]).
2 + t2 Jl
(a) x = 2t—t \ y = 3 t —fi\ (b) x =
1+ t29 y i +? *

128. (a) In an ellipse -+ ^ = 1 inscribe a rectangle with


sides parallel to the coordinate axes having the greatest area (see
Fig. 2).
(b) Given a perfectly square sheet of tin with side a. Small
squares of equal size with side x are cut away from its corners to
make a rectangular box by bending the sheet along the dashed
lines (Fig. 3). For what x will the volume of the box be the greatest?

(c) The ship K (see Fig. 4) is situated at a distance of 9 km from


the nearest point B of the rectilinear seashore. A messenger has to
be sent from the ship to the camp L found on the shore at a distance
o f 15 km from the point B (as measured along the shore). At what
point P must the messenger pull in to the shore to get to the camp
in the shortest time if he goes on foot at a speed of 5 km/hr and by
boat at 4 km/hr?
24 A Collection o f Problems

(d) From a round log of diameter d a girder o f a rectangular


cross-section with base a and altitude h has to be made up (Fig. 5).
For what a and h will the girder be the strongest if it is known that
the strength of a girder is proportional to aKlcl
129. In the parabola given by the equation y — 3 —x2 inscribe a
rectangle having the greatest area so that one of its sides lie on the
x-axis and two pf its vertices on the parabola (Fig. 6).

130. Two ships A and B sail at constant speeds u and v along


straight lines forming an angle 0. Determine the shortest distance
between the ships if at a certain instant of time their distances from
the point o f intersection o f their paths were equal to a and b9
respectively (Fig. 7).
131. Determine the radius of curvature of the given curves:
(a) y 2 = 2px\ (b) = 1;

(c) y = y (1—cos t), x = y (t—sin t) (cycloid).


132. Write the equation of the evolute o f the cycloid
x = a{t—sin t), y — a(l —cos t).
C hapter 2
INTEGRALS

Sec. 2.1. Indefinite Integral


(See [1], C hapter 5)
Using the table integrals, compute the following integrals (Prob­
lems 133 to 143):

133. J x2(5 - x f d x . 134. J ( l - * 2)2 dx. 135. J .

136. J (1 +sin a + c o s x) dx. 137. J -j dx.

138. J tan2 a* dx. 139. J lanh2 a dx. 140. j coth2 a dx.

141. | V l + ^ + V l - * ; dx 142. | (2x—3)09 dx.

143. (a> J i f h d ’ (b) / 0 '- • * ) (-1~ 2x ) 0 - 3* ) dx>

(h) j dx ; (i) J (j) J (2JC+3*)2rfx;

(k) J (a sinh x + b cosh x) dx; (1) J (2x— 9)10 dx;

(m) f dx . ; (n) f d x ;
J V2- 3*2 Jy / l + i x 2
(o) | ~ ^ = ; (p) J (e-**+ e~**) dx.

Transforming the integrand in a proper way or using suitable


substitutions, compute the following integrals (Problems 144 to
152):
26 A'Collection o f Problims"

147 f dx .
J \ / l+x +\ /x - l
148. Jf tan ,v cl*.. 1*97. fj' dx-
JJak-x)n/x.
.

150. J x \ / 2—5
xdx.
3 • J 9*-4* •
151 [ 2xy‘ dx

(b) J x2\ / l + x 3rfx;

<«> f < W W > /ra: W /?S=-.!


(g) | dv; (h) | cot x dx;

«/sinL^ dx
<J> JC cosh,-- /i(k)\ Jfarel+,’
tan* 7
dx'
(1) J sinh2 * <7x; (m) J cosh2 x dx; (n) J ^ ;
(p) | .v3\ / l + v2dx; (q) J i + e i .

Applying the method of integration by parts, compute the fol­


lowing integrals (Problems 153 to 160) :

153. J arc tan x dx. 154. J xe~xdx. 155. Jxsinh x dx.


156. J x sin x dx. 157. J arc sin x dx.

158. J x cos x dx. 159. J arc tan \ / x d x .

160. (a) J x2 sin 2x dx; (b) J x arc tan x d x ;

(c) J x 2e~ 2
xdx; (d) J x?e~x~dx; (e) J In x dx;

(f ) J x" In x dx (n 9 ±— 1); (g) J x3 cosh 3x dx;

J x dx; J x x)dx.
(h) x2 sinh (i) sin In (tan

Applying the method of undetermined coefficients, find the


integrals of the indicated rational functions (Problems 161 to 164):

.
161 (a) j lt- 2i<*.5> dx', (b) J , . 5) dx.
Ch. 2. Integrals 27

(*>/ (x + l)(-v2+ lj ’ mj (AT*+l)*(jC-h2) ‘


dx dx
163.

dx AC*/aC
• <a) Ja + IKl --I)1 ’
164 m j ( * + l)(jt + 2)(;t + 3) ’
, . f xx10«/x
10dx f dx , ^x C x dx
^ Jx * + x - 2 ’ <d> J . H T - & J ^ T ;
dx
(0 / ; <g)
In Problems 165 and 166 integrate the given linear-fractional
surds:
dx
"M i +Vx '
166.
(a) 1 ( l + V x ) W x
(C)
f
(e) f --
dx
v':. (i i v ' o ' '

,■
dx
,- - ,
J l + y x + \/ \ + x

(o j; * ( n—a natural number).


y/ix-d r+'-ix-by-'
In Problems 167 and 168 compute the integrals of the indicated
quadratic surds.

167. f . 168. f \ / x * - 2 x + 2 dx.


J V l+ x+ x* J
In Problems 169 to 172 integrate the indicated trigonometric
functions:

169. (a) J sin4 x dx (b) j ~ .

170. (a) J cos3 x dx; (b) J .

171. f ------- r (/ = t a n 4 ) .
J 2 sin ac—cos ac—1 \ 2/

172. (a) j (p Xitp x x p cmt x (( = tan x);

(e) J [tanh (2 x + l)+ c o th (2 x —1)] dx.


28 A Collection o f Problems

Sec. 2.2. Definite Integral


(See [1], Chapter 6)
173. Prove that the Dirichlet function
{ 0, x—irrational,
1, .v—rational,
is not integrable on any closed interval [a, b].
174. Without evaluating the below integrals, find out which of
them is greater:
71/2 Jl/ 2 ti/ 2
(a) J sin x clx, J x dx , J — dx;

(b) J e* dx, J ( 1 - f x) dx.


o
7 In Problems 175 to 179 evaluate the indicated definite integrals
with the aid of the Newlon-Leibniz formula:
Va n! 4

175. (a) J — 2- ; (b) J sin 2 x dx ;


1/V3 0
7il2 71/4 1/2

0
(d) I tan x dx;
J
0
(e) f
J
-1/2
dx

sinh 2 2 2
m J V idx+x2 (g) f | i - * l
J
(h) f x l
J
sinh 1 0 1
t i /2
d*r
176. Jf — SI
Ct sin2x + 62 co s2 a:
(a, fe > 0).

In 2

177. J x s in x d x . 178. J xe~x dx.


o o
i
179. J x/" (x) dx.
o
180. Prove that for integral k and /:
. JO, if * * / ,
(a) sin kx sm lx dx = { „f
* I if fc = /;
Ch. 2. Integrals 29

2tt
o, if k * I,
< b )j cos kx cos lx dx =
o
31, {if k = /;
2n
(£) J cos kx sin lx dx = 0 , V K I-
o
Hint . Transform the integrand to obtain a sum of trigonometric
functions.
181. Find the derivative of the given integral:
x b b

(a) ^ J sin f dt ; (b) - - J \ / \ + t2 dt; (c) — J sin x 2 </x.


a a a

182. Check to see whether the mean-value theorem


b

jf(x)dx = m ( b - a )

is fulfilled for the function / (x) = x 2 on [0 , 1].


183. Evaluate the definite integral of the function
=x 1,
/(* ) = { 0
1 4 —2x, 1 x 2.

Sec. 2.3. Applications of Definite Integral


(See [1], C hapter 7)
In Problems 184 to 187 compute the area of the figure bounded
by the indicated curves.
184. y = x2, y = 2 —x.
\%5. y = y —0 (h > 0, b > 0) (Fig. 8 ).
r2 v2
186. ^ + ^ = 1 .

187. r = a (l+ c o s y ), r, <p are polar coordinates (see Fig 9).


188. Compute the arc length of the curve
y = --y ln x (l=sx= s£?).

189. Find the arc length of the astroid (Fig. 10)


x ‘M + y j.ls - a 2/3_
30 A Collection o f Problems

190. Find the arc length of one arch of the cycloid (Fig. 11)
x — a(t—sin t \ y = a(l —cos t) (0 < t 2 jr).

191. Derive the formula for the arc length of a curve given in
polar coordinates by the equation g = / ( 0).
192. Find the arc length of the curve given by the equation
r = a cos3 — in polar coordinates (Fig. 12).
193. Find the arc length of the cardioid r = a (l-f cos 99) (see
Fig. 9).
194. Find the volume of the
(a) cone with the generatrix y = (0 ^ x h) (Fig. 13);
(b) conical barrel with the radii of its bases r± and r 2 and altitude
h (Fig. 14);
(c) solid generated by revolving one arch of the sinusoid
y = sin x (0 ^ x < jr);

Fig. 10.
Ch. 2. Integrals 31

(d) solid generated by rotating the parabola y 1 = 2px about the


.v-axis (0 ^ x a) (Fig. 15).
195. Compute the surface area formed by revolving the curve
(a) x = a(t—sin t), y = a( 1—cos t) ( O ^ t ^ 2 jz) about the axis
of symmetry of the curve;
(b) 9y 2 = x ( 3 - x ) 2 (0 < a 3) about the A-axis;
(c) y = a3 (0 < a ^ 1) about the A-axis;
(d) a = a cos3 /, y = a sin31 (astroid) about the A-axis.
196. Evaluate the given integrals by the rectangle formula,
trapezoid rule, Simpson’s rule and estimate the error:

(“> J r h <" = 8>; <b> . f r £ . -


o o
197. Construct Lagrange’s interpolation polynomial of the
third degree for the function f ( x ) if / ( 0 ) = 0, f ( 1) = 1, /( 2 ) = 2,
/ (3) = 0.
32 A Collection o f Problems

Sec. 2.4. Im proper Integrals


(See [1], C hapter 6)
198. Evaluate the indicated integrals:

0
w/<W
0
<■-<*
1 1/2

0 0
p; (d)J ^ -
199. Investigate the given integrals for convergence applying
the comparison test:

(a) J (b) J ~ =
oo oo

dx
e x p ( - x 2) dx;

« 0 / ^ .
1 0
200. Compute the below integrals using the method of integra­
tion by parts:
oo oo

(a) J xe~x dx; (b) J e~axcos bx dx (a > 0);


o o
71

(c) J e* x dx. sin

201. Investigate the given integrals for convergence:


J oo
. v f cosaxdx , ^ rf sin
s in **ddx
x , ^
Xp+X*
o

w / E f <»-»>•
C hapter 3
FUNDAMENTALS OF LINEAR ALGEBRA
AND ANALYTICAL GEOMETRY

Sec. 3.1. D eterm inants and Matrices


(see [2], Secs. 1-3)
In Problems 202 to 210 evaluate the given determinants:
5 3 1 2 8 5
202. 203. 204.
6 4 3 4 * 3 2 ’
cos a —sin a cos a sin a
205. 206.
sin a cos a sin /? cos
x 2x + l
a+b a—b \+ x ~l+ x
207. 208.
a—b a+b -1 x
1+ x 1+ x
2 1 1 0 x 0

209. 1 2 1 210. x 1 x .
1 1 2 0 * 0
211. Find out whether the given permutation is even or odd:
(a) 1 ,2 , 4, 3, 5; (b) 5, 1, 2, 3, 4;
(c) 1,3 , 2, 5, 4; (d) 1 ,4 , 3, 2,5.
212. Find the cofactors o f all the elements in the determinant
a x x
A= x b x
x x c
and check to see that
3
A = Yj alkAik'
*=1
213. Evaluate the below determinants by accumulating zeros in
a row or a column.
1 1 4 1 i 2 3 4
2 1 3 0 2 2 3 4
; (b)
3 1 2 1 3 3 3 4
4 1 1 0 4 4 4 4
34 A Collection o f Problems

214. Evaluate the following Vandermonde determinants:


1 2 22 23 /il d/|2
11C
1 3 3 2 33
(a) ; (b) 1 b b2
1 5 52 5:*
1 c c2
1 6 62 6 s
215. Multiply the determinants
1 2 3 2
Ai = 3 3 2 = 5 and A2 = 3 1 = 7
2 3 2 1 2
using all the four possible ways (i. e. by multiplying the rows or
columns of Ai by the rows or columns of A2). Check to be sure
that in all the cases the product of the determinants A = A±A2 is
equal to 35.
216. Let there be given two matrices

Find the matrix C = XA + p B if (a) A = 1, = 2; (b) A = —5,


/* = 2 .
217. For the matrix

find its transpose, or the conjugate matrix. A* and determine the


rank o f A.

Sec. 3.2. Systems of Linear Equations


(See [2], Sec. 4)
In Problems 218 to 222 solve the given systems of equations
using Cramer’s rule.

218.
I
Ch. 3. Fundamentals o f Linear Algebra and Analytical Geometry 35

x — y + 3 z = 9,
222. 3 x —5y+ z = —4,
4x—l y + z = 5.
223. By transforming the augmented matrix B find out whether
the system
2x-f-7y-f 3 z+ t = 5,
x + 3 y + 5 z —2t = 3,
x + 5y—9z+$t = 1,
5x+ 1 8 y + 4 z+ 5 f = 12
is solvable.
224. Find the ranks of the matrices
1 1 1 r
r2 1 4 5\
1 2 1 2
A= 1 0 1 2 , B=
3 1 3 1
a 2 4 0/
,0 1 1 0,
by transforming the rows and columns of the given matrices (by
accumulating zeros).

Sec. 3.3. Vectors


(See [2], Sec. 5)
225. (a) Find the projection of the vector a = (1, 4) on the di­
rection determined by the vector b = ( \ / \ / 2 , l / \ / 2 ).
(b) Compute the projections x, y, z of the vector a on the coordinate
axes if |a | = 2, a = jt/4, fi = tt/3, y = 2rc/3, where a, /?, y are the
angles formed by the vector a with the x - 9 y-, and z-axes, respec­
tively.
(c) Find the projections of the vector a from (b) on the directed
straight line L with the unit vector b = ( 1/ 2 , 1/ 2 , l / \ / r2 ).
226. Let there be given two vectors : a = (1 ,2 ,2 ), b = (2,1, —1).
bind the moduli o f these vectors, the distance between the points
a and b (provided that the vectors a and b are laid off from the
origin) and the scalar product ab.
227. Find the cosine o f the angle between the following pairs of
vectors:
(a) a = ( 2 , - 4 , 4 ) , b = ( - 3 ,2 ,6 ) ;
( b ) a = (V 2 , 1, - 1 ) , b = (1 ,0 ,0 );
(c) a = (1, 3, V 6 ), b = (1, 1, 0).
36 A Collection o f Problems

228. Can the vector a = (x, y, z) make the angles a = t t / 6,


P = jr/4 with the coordinate axes?
229. Find the coordinates of vector a if | a | = 3, a = (} = y.
230. Given two vectors a and b such that |a | = 13, |b| = 19,
|a + b | = 24. Find |a —b|.
231. Given two vectors a and b such that |a | = 11, |b| = 23,
| a —b| = 30. Find |a + b |.
232. Find the angle between the vectors a = (1, 1, 1, 1) and
b = ( 0 , 1, 0 , 1).
233. Let the vectors a and b ^ 0 be orthogonal. For what
value o f the parameter A is the vector a+Ab orthogonal to the
vector a + b ?

Sec. 3.4. Dividing a Line Segment in a Given Ratio


(See [2], Sec. 7)
234. On the line segment in the three-dimensional space Rs,
connecting the points O = (0, 0, 0) and A = (1, 2, 2), find the
point M = ( x , y, z) dividing this segment in a ratio 2 :3 .
235. Find the coordinates of the centre of gravity M = (x, y)
o f the system o f two material points A = (3, —5), B = ( —1, 1)
at which the masses q = p = 1 are concentrated.
236. If in Problem 235 q = 3, p = 5, find the coordinates of the
centre o f gravity.
237. A line segment with the end points A = (1, —5), B = (4, 3)
is divided into three equal parts. Determine the coordinates of the
division points.

Sec. 3.5. The Straight Line


(See [2], Sec. 8)
238. Find out which of the points Mi = (3, 1), M 2 = (2, 3),
Ms = ( —2, 1) lie on the straight line 2 x + 3 y —13 = 0.
239. Write the equation of the straight line 2 x + 3 y —\3 = 0
in the slope form and also as the equation of a straight line passing
through some point in a given direction.
240. Given a straight line x + 2 y + 1 = 0 . Write the equation of
the straight line passing through the point M 0 = (2, 1): (a) parallel
to the given line; (b) perpendicular to the given line.
241. Given: the equations of two sides of a rectangle
2 x —3 y + 5 = 0, 3 x + 2 y —1 = 0
and one of its vertices O = (0 , 0 ). Write the equations of two other
sides of this rectangle.
Ch. 3. Fundamentals o f Linear Algebra and Analytical Geometry 37

242. Reduce the equations o f straight lines: (a) 2x+5y +A = 0;


(b) x + y —1 = 0; (c) 2x—y + 3 = 0 to the normal form.
243. Find the distance from the point A = (1, 2) to the straight
line: (a) 2x+Ay—5 = 0; (b) 2x+ &y+ l = 0; (c) x + y — 0. 4

Sec. 3.6. The Plane


(See [2], Sec. 9)
244. Write the equation of the plane passing through the point
Mo = (1, 2, —3) perpendicular to the vector v = (1, —2, 3).
245. Write the equation of the plane passing through the three
points: ( 1, 1, 1), ( 1, —1, 0 ), (2 ,1 ,3 ).
246. Write the equation of the plane passing through the point
( 1, 1, 1): (a) perpendicular and (b) parallel to the plane
2 x + 4 y + z - 5 = 0.
247. Which pairs of the below equations define parallel planes
(a) 4 x + 2 y —Az+5 = 0, 2 i t ^ - 2 z - l = 0;
(b) x —3z + 2 = 0 , 2 x —6 z —l = 0 ;

{c)2x—3 y + 5 z —l = 0, 4jc—6 y + \0z—14 = 0;


(d) 2x—3 y + 5 z —1 = 0, Ax—3y+ lOz—14 = 0.
248. Reduce the equation of the given plane: (a) 2x—3 y + 6 z —
—7 = 0; (b) Ax—y + S z —14 = 0 to the normal form.
249. Find the distance from the point A = (1, 2, 1) to the indi­
cated plane: (a) 2x—3 y + 6 z —l = 0 ; (b) 2 x + y —2 z — 1 = 0 .
250. Find the angle between the planes from the preceding prob­
lem.
251. Write the equation of the spherical surface with centre at the
origin touching the plane 2 x + 3y + A z —12 = 0.
252. Write the equation o f the plane 2 x + y —5z—6 = 0 in the
intercept form.
253. Write the equation o f the plane which passes through the
point (2, —1, 1) perpendicular to the planes 2x—y + 3 z —1 = 0
and x + 2 y + z = 0 .
254. Determine the angles a, /?, y formed by the normal to the
plane: (a) x + y \ / 2 + z —1 = 0; (b) x \ / 3 + y + l = 0 with the
coordinate axes.
255. Compute the distance between the parallel planes:
(a) x —2 y —2 z —\ = 0 , (b) 2x—3y +6 z —l = 0 ,
x —2y—2z—6 = 0, Ax—6 y + \ 2 z + \ = 0.
38 A Collection o f Problems

Sec. 3.7. A Straight Line in Space


(See [2], Set. 10)
256. Find the points at which the straight line
f 2 x + y —z —3 = 0,
1 x+y+z- 1 = 0
pierces the coordinate planes.
257. Find the relations which must be satisfied by the coefficients
o f the straight line
| A\x-\- Biy-\-C\z-\rD\ = 0,
{ A 2 X-\-B2 y-\-C2 Z-\- Di = 0,
in order that the latter: (a) intersect the axis of abscissas, (b) coin­
cide with it.
258. Write the canonical equations of the straight line passing
through the point (1 ,0 , —1) parallel to the vector a = (2, - 3 , 5).
259. Write the parametric equations of the straight line passing
through the point ( 1, —1, —3) parallel to the vector a = ( 2 , 1, 5).
260. Write the canonical equations o f the straight line:
x —2 y + 3 z - 4 = 0 , j 5x+ y + z =0,
(a)
3 x + 2 y —5z—4 — 0 ; { 2 x + 3 y —2 z + 5 = 0.
261. Find the angle 90 between the straight lines
x —2 _ y —3 _ z a + 1 _ y —2 _ z + 5
1 ~~ - 1 ~ V 2’ 1 1 ~ \/2 *
262. Given two straight lines
x +2 _ y _ z —1x - 3 _ y - 1 _ z - 7
2“ - _3 - ~ 4 ~ » ~T~ ~ T “ •
Foi^what value o f / do they intersect?
263. Write the equation of the straight line passing through the
point (1, —1, 0) perpendicular to the plane 2x—4 y + z = 3.

Sec. 3.8. O rientation of a System of Vectors.


The Vector Product of Two Vectors.
Triple Scalar Product
(See [2]9 Secs. 10-13)
264. Given two vectors:
(a) a = (1, 2), b = (3, 5); (b) a = (1, 2), b = (3, 7).
Find out their orientation with respect to the xy-system.
Ch. 3. Fundamentals o f Linear Algebra and Analytical Geometry 39

265. Let the vectors a and b form an anule (n = ~r/ 6 and let
|a | = 7, |b| = 6. Find |a X b |.
266. Find out whether the vectors a = (1, 0, 3) and b = (2, 0, 6)
are collinear.
267. What condition must be satisfied by the vectors a and b
in order that the vectors a + b and a —b be collinear?
268. Prove that if a + b -f c = 0, then a x b = bX c = c x a .
269. Compute the sine of the angle formed by the vectors a =
= ( - 2 , 2, 1) and b = (6, 3, 2).
270. Find the area S of the parallelogram constructed on the
plane vectors a = (1,2) and b = (3, 4).
Remark 1. If a triangle with vertices A = (xi, ^i), B = (x2, yf)-,
C = (x3, y 3) is given in the xy-plane, then the area of this triangle
is obviously equal to half the area of the parallelogram constructed
on the vectors AB = (x2—xi, y 3 —y\) and AC = (x3—xi, y 3 —yf).
Hence, the area of the triangle ABC is equal to
-V-.-.Y, 7 2 -/1 ! |
-Y a-.Y t I
This equality can also be written in the form
v, y x 1
S = \ x> y 2 1
v3 y 3 1
This determinant of the third order is equal to the second-order
determinant written above. To make sure that it is so, it suffices to
multiply the first row of the third-order determinant by (—1) and
to add to the second and third rows and then to expand the deter­
minant in terms o f the elements of the third row. As an example,
compute the area of the triangle ABC with vertices A = (1, 2),
B = ( 2 , - 1 ) , C = (0, 1).
271. Check to see whether the below vectors are coplanar:
(a) a = (2 ,3 , - 1 ) , b = (1, - 1 , 3 ) , c = (1, 9, -1 1 ) ;
(b) a = (1 ,1 ,0 ), b = (0 ,1 ,0 ) , c = (1 ,1 ,1 ).
272. Prove that the four points A = (1, 2, —1), B = (0, 1, 5),
C = (— 1, 2, 1), D = (2, 1, 3) lie in one plane.
Remark 2. The Mutual Positions of Two Straight Lines.
Let us be given two straight lines
* -* i _ y - y i _ {Ly)
Pi Vi ’
x — X» v—Vo z — Zo
(M
*2 ' = '~Jtr =
40 A Collection o f Problems

where a?+/??-f}f = 1 (/ = 1, 2). We introduce (unit) vectors


a1 = (oci, /Si, yi), a2 = (a2, /S2, y 2),
and points
= ( V], y u zj ) 9 Ao = (.v2, ) ' 2 , ^2).
77i6> distance d between two straight lines L\ and L >is defined as
the minimum of distances between two arbitrary points AC L\ and
BCLi.
. Three cases are possible here:
I. Li and L 2 intersect at some point. Jn this case, obviously,
d = 0.
II. Li and L 2 are sfcevv lines, i.e. they do not intersect and are
not parallel to each other. In this case the vectors a1 and a2 are
noncollinear and the distance d between L\ and L 2 is computed by
the formula
X 2— .V|1 y -i - y 1 z 2— .
(a^xa2) ___ 1
d= A\AA «| y 1 * ( 1)
i a1X a21 |a ] x a 2|
a2 ya
Indeed, let Hi be the plane passing through L\ parallel to L 2
and let 772, be the plane passing through L 2 parallel to Li. It is
obvious that the planes Hi and 772 are parallel to each other and
perpendicular to the vector a1Xa2. Therefore, the distance between
the straight lines L\ and L 2 is equal to the distance between the
planes 77i and 772. Since the point AiCL iC 77i, and the point
A 2 ClL2C772, the distance d between TZi and II 2 is, obviously equal to
the absolute value of the projection of the vector A\A 2 on the
vector a1Xa2:
d= A±A2\,
which equals the right-hand member
of (1) (see [2], Sec. 5, p. 39).
III. L± and L 2 are parallel. In this
case we may regard that a1 = a2
(changing, if necassary, the sign in
the equations of L2). The distance d
between L\ and L> is computed by
the formula
Fig. 16.

d= = V l^ T -U ^ a ')2
V (X2 -X1)2 +(y-i-yd*+(z2- + (v 2 -yi)Pi+ (z2 -- zi)Vi]2
=

(see Fig. 16).


<«i+/*f+yi = i)-
z.)2 - 1(*2 -
Ch. 3. Fundamentals o f Linear Algebra and Analytical Geometry 41

Remark 3. Two Straight Lines Belonging to One Plane.


Let us show that for two straight lines Lx and L 2 to belong to
some plane, it is necessary and sufficient that the equality
xo-xi y2-y\ Z2- Z 1
«1 Pi yx = 0 ( 2)
or.2 P2 y2

be fulfilled.
Indeed, this equality can be written in vector form in the fol­
lowing way: ^ ^ 2(a1Xa2) = 0 . But this is just the condition of
belonging o f the three vectors AxA2 , a1, a2 to one plane (see [2],
Sec. 13) and, hence, the condition of belonging of the straight lines
Lx and L 2 to one plane.
Remark 4. In Cases I and III the lines Lx and L 2 are, obviously,
found in one plane. Therefore, for them equality (2) is fulfilled.
As far as Case II is concerned, it is obvious that Lx and L 2 do not
belong to any one plane and, therefore, in this case equality (2)
is not fulfilled.
Given below are some exercises in finding the distance between
the indicated pairs of straight lines:
—1 _ y - 2
X z —6 x -l y- 2 z —6
(a) 4 5 ” 4 ’ 5 “ 6 “ 3 ’
x —1 y-2 z —6 X y - 1 r —2
(b) 4 5 “ T ’ T 2 T ' ;
x —1 y-2 z — 1 x-5 y- 3 z+1 .
(c) 4 5 ~y~’ 8 10 6 ’
x —2 y+ i z x-2 z +! .
(d) 1 2 “ T ’ 2 4 6 ’
X y - 1 z +2 x -l ^+2 z —6
(e) T 5 = T ’ 1 “ 2 " "T“ ;
X + l y+1 z —1 JC+1 ^+1 2—1
(0 l 2 “ T ’ 2 3 ~T“ ;

(answers: (a) 0; (b) 3 /^ /2 ; (c) \/3 3 /2 ;(d ) y / 3 / l 4 \ (e) 7 /V 2 ; (f) o).


Let us consider the solution of Problem (b). In this example
Ax = (1, 2, 6), A 2 = (0, 1, 2). The vectors (4, 5, 4) and (1, 2, 1)
are not unit vectors. Multiplying the equations o f the given lines
by the moduli o f these vectors, we obtain the equations of the lines
in the desired form:
x —1 _ y —2 _ z —6 x __ y —1 _ z —2
4 /\/5 7 _ 5 /\/5 7 ~ 4 /\/5 7 ’ l / \ / 6 ~ 2/-\/6 ~~ l / V ^ ’
42 A Collection o f Problems

i.e. a , = 4 /V 5 7 , 0, = 5 /V 5 7 , y, = 4 /V 5 7 ; a 2 = l / y / 6 , /?s =
= 2/ \ / 6 , y 2 = i / y / 6 . It is easy to check that condition (2) is
not fulfilled, i.e. our lines are skew lines. Therefore, the required
distance will be found by formula (1). Let us find the vector
product of the unit vectors a1 = (oci, /?i, yi), a2 = (a2, /L, >'2):

i j k
1
a 'X a 2 = Pi r1 [—3i + 3k].
V<3-57
a 2 fi* Y2

Hence, |a1Xa2| = ] / \ / \ 9 . Now by formula (1), we obtain

-1 -1 - 4
1
d= y/\9 4 5 4
\/57-6 \/2 ‘
1 2 1I

Remark 5. The Volume of Tetrahedron.


Let in space there be given a tetrahedron (triangular pyramid)
ABCD with vertices A = (xu yu z.i), B = (x2, y 2, z2), C =
= (*3, J>3, 2T3), D = (* 4, ^4, ^4).
—. It is required to find the volume
j of this tetrahedron (Fig. 17).
I As is seen from the figure, the
' volume of the tetrahedron ABCD
is equal to 1/6 the volume of the
parallelepiped constructed on the
vectors AB, AC , M). But we
know (see [2], Sec. 13) that the
volume o f this parallelepiped is
equal to the absolute value of
the triple scalar product o f the
vectors AB, AC , AD. Therefore,
the volume V of the triangular
Fig. 17.
pyramid ABCD is equal to

V = - ( ABxAC)AD

Xi yi z 1 1
x2 -X ! y 2~ y i z2 - z x
1 1 x 2 y 2 z2 1
* 3 — *1 73 -^ 1 ^ 3-^ 1
6 6 *3 ys z3 l
■v4-*i y4 y 1 Zi-zi
v4 Vi Zi 1
Ch. 3. Fundamentals o f Linear Algebra and Analytical Geometry 43

Problems. Find the volume of the tetrahedron given by its ver-


tices:
(a) A = (0, 0, 0), B = ( 1 ,1 , 0 ) , C = (2 ,1 ,0 ), Z) = (0 ,0 ,6 );
(b) A = (0, 0, 0), B = ( 4 ,1 , 1 ) , C = (1 ,1 ,0 ), D — (0, 0, 8 );
(answers: (a) 1; (b) 4).

Sec. 3.9. Dependent and Independent Systems of Vectors


(See [2], Sec. 14)
273. Find out whether the vectors
(a) a1 = ( 1, 1, 1, 1), a2 = ( 1, 2 , 1, 2 ),
a* = (3,1, 3,1), a4 = ( 0 , 1, 1, 0);
(b) a1 = ( 1, 0 , 1), a* = ( 1, 1, 2 ), a^ = ( 2 , 1, 2 )
are linearly dependent or linearly independent.
274. Prove that a system of vectors containing two equal vectors
are linearly dependent.
275. Find all the values of A for which the vector b = (7, —2, A)
is linearly expressed in terms of the vectors a1 = (2, 3, 5), a2 =
= (3, 7, 8 ), a « = ( 1, - 6 , 1).

Sec. 3.10. Linear O perators. The Basis


(See [2], Secs. 15-17)
276. Compute the product of the matrices AB and BA :

277. Compute the below expressions:


44 A Collection o f Problems

278. Find all the matrices B permutational


(commutative) with the matrix A :

(a) ^ = ( ; J ), ,b) ^ = ( i ?).

279. Find the inverse to the given matrix:

(a) A = (3 (h) A =

( cos a —sin a\
sin a a/
cos <
280. Solve the matrix equation

C^ = ( 5 »)•where *=(* :)•


Solution. Let us find the inverse of the matrix A =
= = 4, A\o = —3, A 2 1 = —2, A 22 — 1 ; A =

-G 4 ) = -
A~* =

Multiplying both sides of the equation by A ~ \ we get


(A - 1 A = E):

— GH r 5)-
281. Solve the matrix equation

* « - ( = ; 9-
To find the inverse A 1 of the matrix A , we have to solve
the system y = Ax with respect to x. Let x = By. Then A “ 1 = B ,
since ABy =* Ax = y, i.e. AB = is; iL4x = By = x, i.e. iL4 = is,
where is is a unit matrix, i.e. AB = BA = E.
Ch. 3. Fundamentals o f Linear Algebra and Analytical Geometry 45

Let us apply this method to find the inverse of the matrix A =

= (3 4) * ^orm ^near system

. [ *1+2*2 = yu
Ax = y or \
[ 3 * i+ 4*2 = y 2.
Solving this system we obtain
*1 = - 2 ^ i + j 2,
3 1
*2 = 2 + 1 - y ^ .

Hence,

282. Using the indicated method, find A~x for the given matrices:

I v /I 1 0
(a ) A = ( 2 ); (b)A = \ 0 1 1
x 7 \1 0 1
283. Find out which of the operators in Ax is a linear one, and
for a linear operator find its matrix:
(a) Ax = (*2+ * 3, 2 x i+ x 3, 3 * i - x 2+ * 3);
(b) Ax = (xi, * 2+ 1, * 3 + 1).
284. Let in the basis i1, i2, i3 there be given linearly independent
vectors a1, a2, a3. Find the linear transformation transferring the
vectors a1, a2, a3 into b1, b2, b3 respectively if
a1 = (2, 3, 5), a2 = (0, 1, 2), a3 = (1, 0, 0);
b1 = (1, 1, 1), b2 = (1, 1, - 1 ) , b3 = (2 , 1, 2).

Hint (see [2], Sec. 16). If we are given the systems of vectors
a1 = (an, a 2 1, # 31)* a2 = (^12, 022, 032)5 a3 = (tfi3, # 23?033);
b1 = (611, 621, 631)5 b 2 = (612, 6225 632), b3 = ( 6 i3, 6235 633)5
then the linear operator generated by the matrix
46 A Collection o f Problems

maps the basis F, i2, i3 into a1, a2, a3, respectively. Consequently,
A~x maps a1, a2, a3 into i1, i2, i3, respectively. Further, the operator
B generated by the matrix
(b n bi 2 &13
B = \ b 2i &22 ^23
\ h i ^32
maps i1, i2, i3 into b1, b2, b3, respectively. Hence, BA~X maps
a1, a2, a3 into b1, b2, b3, respectively.
285. Find the linear transformation mapping the vectors
a1 = (2, 0, 3), a2 = (4, 1, 5), a3 = (3, 1, 2)
into the respective vectors
b1 = (1, 2, —1), b2 = (4, 5 , - 2 ) , b* = ( l , - 1 ,1 ) .
286. A linear transformation A has the matrix
l 2 0 r
3 0 -1 2
2 5 3 1
,1 2 1 3,
in the basis i1, i2, i3, i4. Find the matrix of the same transformation
in the basis: (a) i1, i3, i2, i4; (b) i1, F + i2, F + i2+ i3, F + i2-f-i3+ i 4.
287. In the basis a1 = (1, 2), a2 = ( —1, 1) a linear transforma­
tion A has the matrix

Find the matrix o f this transformation in the basis b1 = (1, —2),


b2 = (3, —1).
288. Let transformation A have the matrix A = ^ ^ in the
basis a1, a2 (see Problem 287). Find the matrix of this transforma­
tion in the basis b1, b2.
289. Which o f the given pairs of vectors are orthogonal:
(a) x = (1, 2, 3), y = (0, - 3 ,2 ) ;
(b) x = (1, 2, 1), y = (0, 1, 2);
(c) x = (1, 0, 1), y = (0, 2, 1).
290. Show that the system of vectors
e1 = (1, 2, 3), e2 = ( 0 , - 3 ,2 ) , e3 = (13, - 2 , - 3 )
is an orthogonal basis in R$. Find the coordinates of the vector
X = (1, 0, 0) in this basis.
Ch. 3. Fundamentals o f Linear Algebra and Analytical Geometry 47

291. Supplement an orthonormal system o f the vectors


x = (l/y/2, 0, 1/V2, o), y = (0, - 1/a/2, 0, l/y/2)
with the vectors z and t to complete an orthonormal basis in R4 .
292. Find out the orientation of the orthogonal basis with
respect to the main basis i1 = ( 1, 0 , 0 ), i2 = (0 , 1, 0 ), i3 = ( 0 , 0 , 1):

1 V 2\
2 ’ 2 /’

293. Let a new orthogonal basis b1, b2 be given by the orthogonal


matrix
V 3 /2 l/2 \
—1/2 y/3/2)
Write the formulas relating the coordinates (xi, x2) of the vector a
in the old basis with its coordinates (x^, x£) in the new basis.
294. Let the basis a1, a2 be given by the matrix A =
Write the transformation formulas for passing from the coordinates
of the vector a in the old basis to the coordinates in the new basis,
and vice versa.

Sec. 3.11. Linear Subspaces


(See [2], Sec. 20)
295. Is the following set of vectors a linear subspace:
(a) having odd integral coordinates;
(b) having even integral coordinates;
(c) lying on a straight line passing through the origili;
(d) lying on the x- or 7 -axis;
(e) whose terminal points lie in the first quadrant of the coordi­
nate system (the initial points of the vectors are supposed to be
coincident with the origin);
(f) whose terminal points lie on a given straight line;
(g) whose terminal and initial points lie on a given line;
(h) which are all possible linear combinations of the vectors
x \ x2, . . . , xk in Rn (k ^
48 A Collection o f Problems

296. Enumerate all linear subspaces o f R 2 .


29T. Let L be a subspace of R 2 (that is, the set of vectors lying on
the straight line x 2 = kx 1). Find the subspace L' which is ortho­
gonal to the given one.
298. Find the dimension and basis of linear subspaces which are
linear combinations of the indicated vectors (or, as we say, spanned
on a given system of vectors):
(a) a1 = (1, 0, 0, - 1), a2 = (2, 1, 1, 0), a2 = (1,1, 1, 1),
a4 = (1, 2, 3, 4), a5 = (0, 1, 2, 3);
(b) a1 = (1, 0, 1), a2 = (1, 1, 1,) a* = (2, 1, 2), a4 = (3, 2, 3).
299. Let L be a subspace in jR4spanned on the vectors
a1 = (1, 0, 0, —1), a2 = (2, 1 ,1,0).
Find the subspace L' orthogonal to L. Let the vector a be orthogonal
to L '. Prove that this vector is a linear combination of the vectors
a1, a2 (a = a a ^ /fo 2).
300. Let e1, e2 be an orthonormal basis in a plane and linear
operator A has the matrix in the basis f 1 = e1, f 2 =
= e1-}-©2. Find the matrix of the adjoint operator A* in the same
basis f 1, f 2.
Solution. In the basis e1, e2 the matrix of operator A* is a trans­
pose o f the matrix of operator A.
Let us first find the matrix of operator A in the basis e \ e2.
We have
A(eL) = A ii1) = f x+ f 2 = 2e1+ e 2;
A(e2) = ^ (f2- f x) = A ( P ) - A ( f 1) = f 1—2f2 = —e1—2e2.
Thus, the matrix of operator A in the basis e1, e2 has the form

^=e - 2)*
Now we find the value o f operator A* on the vectors f 1, f 2:
A*f 1 = A*ax = 2e1- e 2 = 3f
A*f 2 = y4*(e1+ e 2) = A*el +A*e* = 3f1- f 2+ e 1- 2 e 2 = 6f1—312.
Hence, the matrix o f A* in the basis f 1, f 2 has the form
Ch. 3. Fundamentals o f Linear Algebra and Analytical Geometry 49

301. Let in Problem 300 the matrix of operator A in the basis


matrix of A* in the same
basis.
302. A linear operator A in the basis f 1 = (1, 2, 1), f 2 = (1, 1, 2),
f 3 = (1, 1, 0) is given by the matrix

Find the matrix of A* in the same basis regarding that the coordi­
nates o f the vectors are given in some orthonormal basis (for in­
stance, f 1 = e1+2e2+ e 3).

Sec. 3.12. Self-adjoint O perators. Q uadratic Forms


(See [2], Secs. 22, 23)
303. Find the greatest eigenvalue of the self-adjoint operator
defined by the matrix:

304. Using Sylvester’s theorem, find out whether the given


quadratic form is strictly positive:
(a) X^-\~X2 4-3X^+4X1 X2 +2X 1 X3 +2X 2 X3;
(b) 2xix 2 4- 2x±xz + 2xix4+ 2x 2 x 3 4- 2x2x44- 2x3x4;
(c) 2*14- 4- 3x§ 4- 2x1X24- 2 xix 3+ 2x2x 3.
305. By finding the eigenvalues, find out the type of the quadratic
form:
(a) / = x 2 + 4 x y —y 2;
(b) / = x2+ 2 6 y 2+10xj>;
(c) / = x2+ 3 j 2+ 2 \/3xy.
306. Reduce the indicated quadratic form to the canonical form:
(a) 3 x l+ 3 x § + 4 x ix 2 + 4 x ix 3—2x2x 3;
(b) 7*i 4- 7x14- 1x3 + 2 x 1 x 2 + 2x \x 3 —2x 2 x3.
307. Find the orthogonal transformation which reduces the fol­
lowing-forms to the canonical form. Write this canonical form:
(a) 6 x i+ 5 x l+ 7 x 3—4 x ix 2+ 4 x ix 3;
(b) 17x?+14*1+ 14x3—4 x ix 2^-4xix3—8x2x3.
50 A Collection o f Problems

Sec. 3.13. Second-order Curves


(See [2], Sec. 24)
The general equation of a second-order curve is written as
Ax 2 +2 Bx y+ Cy 2 + D x + E y + F = 0, (1)
where A , B , C are not all zero.
We assume that B ^ O . Equation (1) can also be reduced to this by
setting x = x \ y = —y'. An appropriate orthogonal transfor­
mation reduces equation (1) to the form
Ail2+A 2r?2+ 2 c /|+ 2 ^ + g = 0, (2)
where Ax, A2 are the eigenvalues of the linear self-adjoint operator
generated by the quadratic form
Ax 2 +2Bx y+ Cy 2 (3)
and d 9 e , g are certain numbers. For B = 0 equation (1) has form
(2). Therefore, further on we assume that B > 0.
The coordinates £, r\ are considered in a new rectangular system
(new orthonormal basis) whose unit vectors are the eigenvectors o f
the indicated self-adjoint operator. In this case, if the first eigen­
vector x1 (unit vector), corresponding to the eigenvalue Ai, has the
coordinates (jc0, jo)» then for the second eigenvector (for 2? > 0) we
take the vector ( —y0, x0) or (y0> —xo). Note that the vectors
(x0, yo% ( —Jo, *o) are oriented in the same way as the initial basis
i = (1, 0), j = (0, 1), whereas the vectors (x0, yo)? (yo? —*o) are
x0 yo
oriented in the opposite way | A = = - i < o j.
yo - x 0
Thus, if B > 0, then the coordinates are transformed to the form
* = xot+yor},
y = yo£±x 0 r}.
In the two-dimensional case the eigenvalues and eigenvectors can
be computed by the formulas obtained in [2], Sec. 24:

Ai = ^ C f,

A2 = — ~ - \ / 4 B 2 + ( A - C ) 2;

_ if 1 . A~ c
X° ]/ 2 + 2a/4B2+ (^ -C )2’
1 A -C
2 2 V 4#2+ (A —C)2
Ch. 3. Fundamentals o f Linear Algebra and Analytical Geometry 51

Of course, each time the eigenvalues can be found as the roots


o f the characteristic equation
A —X B

and the coordinates of the eigenvector x 1 as the solution of the


system
| ( A - X 1 )xo+ By 0 = 0,
\ Bx o + ( C - A i )^0 = 0.
If both eigenvalues and A2 are not equal to zero, then equation
(2 ) can be written in the following way:

w - * y + W v - P r = Y, « = - £ , /* = - • £ > w
where y is a constant. Whence, setting u = a, v = rj—(J9 we get
X1 u2 +X 2 v2 = y. (5)
If AC—B2> 0, then AiA2 > 0 , and (5) directly yields the
canonical equation of an ellipse (real or imaginary) or a point.
If AC—B2 < 0, then X±X2 < 0 and (5) readily yields the
canonical equation of a hyperbola or a pair of intersecting straight
lines.
And if AC—B1 = 0, then X\X2 = 0 . But if one o f the eigenvalues,
say Ai, is different from zero, then equation ( 2 ) is written in the form
A i(!-a ) 2+ f y = co. (6 )
If 6 = 0, then equation (6 ) defines a pair of straight lines (real or
imaginary). But if d 0, then, setting u — a, v = y — and,
possibly, v = —v \ we obtain the canonical equation of parabola.
308. Find out the type o f the following curves and reduce their
equations to the canonical form:
(a) 3x2 + 3 y 2 —6 x —12y+3 = 0 ;
(b) 3x 2 -\-2y2—6 x —12y+15 = 0;
(c) x 2 —2y 2 + 4 y —4 = 0;
(d) 3x2—6x+3y2—12y+15 = 0;
(e) x 2 —2y 2 ~h4y—2 = 0 ;
(f) 4 x - 3 y 2 + 1 2 y - 1 2 = 0 ;
(g) 3x2 + 2y 2 —6x—12y+22 5= 0 .
52 A Collection o f Problems

309. Determine the type of the following curves and reduce


their equations to the canonical form. Write the transformations
o f the coordinate system. Represent the coordinate systems and
the curves.
(a) 3x2 -\-10xy+3y 2 —2x—\4y—\3 = 0;
(b) 25x2—I4xy+25y2+6 4x —64y—224 = 0;
(c) 9x2—24xy + 16y2—20x+ llOy—50 = 0.
310. Given the equation of a curve: 4x 2 —4 xy + y 2 + 6 x + 1 = 0.
Determine for what values of k the straight line y = kx
(a) has one common point with the curve;
(b) intersects the curve at two points;
(c) has no common points with the curve.
311. For what values of k does the straight line^y = kx touch the
curve (x + j >)2+ 2 = \ / 2 (y—x).
312. Write the equation of the second-order curve passing through
the following points: ( 0 , 0 ), (1, 0), (1, 1), ( —2, 1), (0, 3).

Sec. 3.14. Second-order Surfaces


(See [2], Sec. 25)
The general equation of a second-order surface has the form
0 1 1 * 1 + # 2 2 * 2 4 * 0 3 3 * 3 “I" 2 f l i 2* 1 * 2 + 3X 1 X 3 + 2 0 2 3 * 2 * 3

-f-2^4] Xi+ 2 ^ 2* 24- 2 ^ 3X3+ B = 0 . ( 1)


If the mixed products of the variables are absent from equation
(1) (i.e. 012 = 013 = 023 = 0), then the equation of a surface is
reduced to the canonical form by separating perfect squares with
respect to the variables xi, X2, *3 of the form
0 1 l ( * l — a i ) 2 + 0 2 2 ( * 2 — <*2) 2 + 0 3 3 ( * 3 _ a s ) 2

and translating the origin to the point (aT, a 2, a3).


But if the mixed products are present, then we first reduce the
3 3
symmetric quadratic form Y Y akixkx\ to the canonical form.
k= 1 /=i
The eigenvalues of the operator generated by this quadratic
form are found as the roots of the characteristic equation
| 011— A 012 013 |

i 021 022 ^ 023 | ^ 0,


| 031 032 033 ^ |
Ch. 3. Fundamentals o f Linear Algebra and Analytical Geometry 53

and the coordinates of the eigenvectors xk (k = 1 ,2 , 3) are found


from the systems
(tfll—%k)Xl+ 012*2 + 013*3 = 0,
021*1 + ( 0 2 2 - 4 ) * 2 + 023*3 = 0,
03l*| + 032*2 +(033 ~’Ajfe)*3 — 0,
which, as it is shown in [2], Sec. 25, always have solutions (three
pairwise orthogonal vectors xl , x2, x:t).
313. Reduce the indicated equation to the canonical form and
name the surface defined by this equation:
(a) x2+ y 2+ z2+ 2 x + 4 y —4 = 0;
(b) x2+ 2 y 2-h z2+ 2 x + 4 y —1 = 0;
(c) x2+ 2 y 2— z2~h2x-h4y—1 = 0;
(d) x2+ 2 y 2+ 2 x + 4 y —2 z +3 = 0;
(e) x2- 4 y 2- z2+ 8 y —2 z —9 = 0;
(f) x2+ 2y 2—2x—4y—1 = 0.
314. Reduce the given equations of surfaces to the canonical
form and indicate the appropriate transformations of the coordi­
nates
(a) llx f+ 5 x f+ 2 x 3 + 16x1x2+ 4x1x3—20x2x2+ 2 x 1+ 2 x 2-h2xs
+ 1 = 0;
(b) 3*2+3 a| + 4 x1X2+4 x1x3—2 x2*3+ 4 * 1+ 1 = 0.
315. Find the curve along which the plane x = 2 cuts the ellip-
V-2 y2 ^2
Soid J6 + h + ~4 = l -
316. Write the canonical equation of the hyperboloid of one
sheet passing through the points (1, 0, 0), (0 ,4 , 0), (1,1,1).
317. Write the equations of the projections (on the coordinate
planes) o f the section of the elliptic paraboloid / 2+ z 2 = x by the
plane x + 2 y —z — 0.
318. What line is defined by the equations

* - 2 ^ + 2 = 0?
319. Write the equation of the tangent plane to the hyperboloid
of one sheet ^ —-~ + ~ = 1 at the point (0, 0, c).
54 A Collection o f Problems

320. Write the equation of the surface generated by revolving the


ellipse

o2
z= 0
about the x-axis.
Solution. Let us take an arbitrary
point P = (x, y , 0) on the given
ellipse. During rotation of the ellipse
about the x-axis the point P describes
a circle o f radius y. Let M = ( x , y, z)
be an arbitrary point on this circle
(and, consequently, on the desired
surface). It is obvious that CP =
= \y\ = CM = V y 2+ z2> x = x
(see Fig. 18). Since the point P lies
on the ellipse, we have

Replacing y and x by their values, we obtain


+* 1
Z>2 1#
This is just the sought-for equation of the generated surface.
Remark 1. Since the curve / ( * , y) = 0 is revolved about the
x-axis, the jz-coordinate is replaced by \ / y 2 +z* in the equation of
this curve. As a result, we get the equation of a surface of revolution
about the x-axis:
f i x , v y + z 2) = o.
It is also obvious that
f i y / j p + z2, y) = 0
is the equation o f a surface of revolution about the jz-axis.
321. Write the equation of the surface of revolution generated by
rotating the curve

(a) (b)
{ y = o
about the z-axis.
Ch. 3. Fundamentals o f Linear Algebra and Analytical Geometry $5

322. Find the points of intersection of the folloving surface and


straight line:
X2 V2 z2 x -3 _ y - 4 _ z+2 #
<a> £ + & + - * - 3~ “ ' - 6 “ 4“ ’
O *}
a- -|-1 _ v —2 _ z | 3
(b) JJ - + - V = * 2~ “ -I " -2 •

323. Write the equation of the cone with vertex at the origin
whose elements touch the sphere

( x + 2 ) 2+ 0 > — l ) 2+ ( z — 3 ) 2 = 9.

Solution. The given sphere is of


radius 3, its centre C having the
coordinates C = ( —2, 1, 3). Hence,
the xy-plane touches the sphere at
the point P — ( - 2 , 1, 0) (Fig. 19).
The ray OP is the generatrix of the
cone. The directrix of the cone lies
on the sphere and in the plane pass­
ing through the point P perpendicu­
lar to the straight line OC. The
equation o f the straight line OC is
X _ Y _ Z
\ ~ 3 *

The equation o f the plane passing through the point P perpendicu­


lar to OC is
—2 ( x + 2 ) + ( y —l)+ 3 z = 0.

Therefore, the directrix of the cone can be defined by the system

f (x+2) 2 + ( y —\) 2 + ( z —3) 2 = 9,


{ —2 x + y + 3 z —5 = 0.

Let now (x, y , z) be an arbitrary point on the directrix and let


(X , T, Z) be an arbitrary point o f the generatrix (and, consequently,
a point belonging to the cone). The equation of the generatrix can
be written as the equation of the straight line passing through the
points (0, 0, 0) and (x, y 9 z ) :
X _ Y _ Z
.y y z
56 A Collection o f Problems

Eliminating x, y , z from the system and from the last three equa­
tions, we obtain the equation of the cone. We then fix z = c and
express x and y in terms of X, Y, Z :

Substituting these values into the system and eliminating the para­
meter c, after elementary transformations we obtain the required
equation o f the cone:
X2+ 4 Y 2- 4 Z 2+ 4 X Y + \ 2 X Z - 6 Y Z = 0.
324. Write the equation of the cone with vertex S = (5, 0, 0)
whose elements touch the sphere x2+ y 2~hz2 = 9.
C hapter 4
FUNCTIONS OF SEVERAL VARIABLES
(See [1], Chapter 8)

Sec. 4.1. Basic Concepts


325. Find and represent the domains of existence of the following
functions:

(a) u = \ / l - x 2- 4 y 2 ; (b) w = —+ ;

(c) u = V y 2—4x; (d) u = ;

(e) u = In (x-bj>); ( f ) u = arc sin y .

326. Find and represent the domains of existence of the following


functions o f three variables:

(e) u = arc sin x + a rc sin y +arc sin z.


327. Evaluate the function

f(x,y) = x*+±

at the points (1, 0), (1, 1), (2, 1).


328. Find f ( x , y ) if f ( x + 2 y , x —2y) = xy.
329. The contour (or level) line of the function u = / ( x , y) is
defined as the set of points belonging to the domain of its definition
at which it attains the prescribed constant value: / ( x , y) = c.
The latter is thus the equation of the contour line.
Geometrically, it means that we cut the surface defined by this
function (i.e. its graph) by the plane u = c and projected the line
thus obtained on the xy-plane. This projection of the section is just
the contour line.
Find the contour lines of the given functions:
58 A Collection o f Problems ^

330. Find the distance q between the points (1,0, 1) and (2, 1, 0)
in space R&.
331. Find the limit of the sequence o f points

* * = (r h -* T i)
332. Let the set E — {|.v| < 1, \y\ ^ 1}. Which points of this
set are interior?
333. Are the following sets connected on es:

(a ) E = {\x\ + \ y \ ^ 1}; (b ) £ = { * - £ - £ ^ l} ;

(c) E = {x*+y* * 1}?

Sec. 4.2. Limit of a Function. Continuity


(See [1], Secs. 8.3, 8.4)
A function/has a limit at a point x° equal to A if it is defined in
some neighbourhood of the point x° except, possibly, at the point
x° itself, and if
lim f { x k) = A,
X*-KX0
X*^X°
whatever the sequence of points xk from the mentioned neighbour­
hood (different from x°) tending to x°.
But there are cases when a function / is defined not in the entire
neighbourhood, but only in one of its subsets E. In this case there
arises the notion o f the limit of a function at the point x° with res­
pect to the set E.
The number A is said to be the limit of the function/ at the point
x° 6 E (E is the closure of E , see [1], Sec. 8.11) with respect to the
set E if
lim f ( x k) = A ,
x^—►X®
x* € E,

whatever the sequence o f points xk £E, converging to x°. This


definition is equivalent to the following: the number A is called the
limit o f a function / at point x° 6 E with respect to the set E if
V e > 0, 35 = 5(e, x°) > 0 such that
|/ ( x ) —A \ < e, \jx£E, 0 < | x —x°| < 5.
Example 1. The function
f ix ) = f i x I, x.,) = V - r H 4 sin -jr-- *
i I.,
Ch. 4. Functions o f Several Variables 59

is defined in the entire plane R >except the origin. It is obvious that


in any neighbourhood of the origin the function / satisfies the in­
equality

l/(x)| = x/x\-Vxl sin


V L “ .vf KY$
y /x \+ x \
— IX I = |x~~0|-^ F (x ^ 0)
provided that |x | < b — e. Hence, at the point 0 = (0, 0) an or­
dinary limit exists and is equal to zero:
lim f ( \ ) = 0.
jc—►0
x^O
Example 2. The function

/(*i> * 2) = sin —
xlx2
is defined on the set E representing the plane R2 without the coordi­
nate axes. The function / has no ordinary limit at the point 0 =
= (0, 0), but the limit o f/ at this point with respect to E exists and
is equal to zero
lim f (x u x 2) = 0.
Xj ->■0, x%-+■0
C*i>*2) €E
Problems. Consider the existence of the limit of the given func­
tion at the indicated point:

(a) f{x , y) = sin/ + y ■ at the point (0, 0);

(b) f i x , j>) = 1 2) at the point (0, 0);


(c) f { x , y, z) = exp ( - 1/(x2+ y 2+ z2))/(x4+ y * + z4)
at the point (0, 0, 0).
334. Find the limit o f the given function:
(a) lim t o ( , * 0); (b) lim t o .
x—
*o x x-+o x
y-+ 2 y 0

335. For what c will the function

V l - x 2- 4 y \ x2+ 4 y2 ^ 1,
f i x , y) =
c. x2+ 4 v 2 > 1
be continuous in the entire .vy-plane?
60 A Collection o f Problems

336. Find out: (a) whether the function


2xy
x 2+ y2 ’
x2+ y 2 ^ 0,
f i x , y) =
0, x = y = 0
is continuous at the point (0, 0);
(b) whether it is continuous on the ray in the direction of any
vector co ^ 0, emanating from the origin.
337. Prove that the set of points (x, y) satisfying the inequality
1 —x2—y2 > c is an open set.

Sec. 4.3. Partial Derivatives. Differentials


(See [1], Secs. 8.5 and 8.6)
In Problems 338 to 341 find the partial derivatives of the given
functions and their total differentials.
338. u = x3+ y 2—2xy. 339. u = x2y 3.
340. u = In (* + \ / x2+ y 2).
341. (a) u = arc tan (ylx ); (b) u = x y + — ;

(c) u = xy \ (d) u = sinh (x+y); (e) u = cosh (jt2y + sin h y ).


342. Compute the determinant
0JC dx
dr d<p
dy dy
dr d(p
if: (a) x = r cos y , y = r sin <p; (b) x = r2+<p, y — r+cp2.
343. Find the partial derivatives of the indicated composite
functions with respect to the variables t and r :
(a) u = \/x - h y , where x = et+r, y = In t\
(b) u = xy, where x = cos (f+ r), y = sin (t—r).
344. Find and construct the gradient of the given functions at the
point P = (1, 1):
(a) u = x2y; (b) u = 2x2—3y2.
345. Find at the point P = (1, 1) the derivatives of the functions
u in the direction o f the vector n = ( \/3 /2 , 1/2):
(a) u = In V ^ + 7 2; (b) u = 2x2- 3 y 2.
Ch. 4. Functions o f Several Variables 61

346. Find the derivative of the function u = 2 x 2— 3y2 at the point


P = (1, 1) in the direction of the gradient.
347. Find the angles made by the gradient of the given function
at the point P = (1, 1) with the coordinate axes:
(a) u = x ^ 3+ y ; (b) u = x + y ^ 2.

Sec. 4.4. Partial Derivatives and Differentials


of Higher O rders
(See [1], Secs. 8.5 and 8.9)
348. Find the partial derivatives and second-order differentials o f
the following functions:
(a) u = In (x2-\-y); (b) u = \ / 2 x y + y 2.
349. Show that the indicated functions
(a) u = arc tan 0 7*); (b) u ——In \ / { x —a)2+ ( y —b)2 satisfy
Laplace’s equation
d2u & u_
dx2 + dy2 ” U*
350. Show that the function
u = (p{x—at)+y)(x+at).
where cp, ^ have derivatives up to the second order, satisfies the
. 02W o 02M
equation = a2 .
351. Find the derivatives and differentials of the second order of
the composite functions (*, y are independent variables):
(a) u = / ( | , rj), £ = ax, r\ = by;
(b) u = /( £ , rj), | = x+y, rj = x - y .
It is supposed that / ( I , rj) has derivatives up to the second order
(inclusive) with respect to all variables.

Sec. 4.5. Tangent Plane and a Normal to a Surface


(See [1], Sec. 8.7)
352. Write the equation o f the tangent plane and the equation of
the normal to the given surface at the indicated point:
(a) to the paraboloid o f revolution u = x2+ y 2 at the point (1,
2, 5);
2
.XT
(b) to the surface u = — y 2 at the point (2, —1, 1).
62 A Collection o f Problems

Sec. 4.6. Taylor's Formula


(See [1], Sec. 8.10)
353. Find the increment in the function:
(a) u = x2—y2+ x y when passing from the values x = 1, y = 2
to the values xi = 1+ h, yi = 2 + k \
(b) u = x2y when passing from the values * = 1, y = 1 to the
values jci = 1 + h9yi = 1+ k.
354. Using Taylor’s formula, expand the function f ( x 9y) =
= ex sin y in the neighbourhood of the point (0, 0) up to the terms
o f the third order inclusive.
355. Using Taylor’s formula, expand the function f ( x 9y) =
= exp (x-h y) up to the terms of the third order (inclusive) in the
neighbourhood of the point (1, —1).
356. Find the value of the parameter 6 in Lagrange’s formula for
functions o f two variables (see fl], Sec. 8.10):

■i/xO+0(x-xO)

(x2-* g ) (0 < 6 < 1);


2 / X ° + 0 (X — X °)

(a) / ( x ) = x j+ x| with respect to the points x° = (0, 0), x =


= a , D;
(b) / ( x ) = x j+ x f with respect to the same points.

Sec. 4.7. Extrem a


(See [1], Sec. 8.13)
In Problems 357 to 361 investigate the given functions for extre­
mum. ,
357. z = (x —2)2-f 2y2. 358. z = (x -2 )2- 2 y 2.
359. z = x4+ 4 x y - 2 y 2.
360. z = xi + y i —2x2+ 4 x y —2y2.
361. u — x2+ j;2+ z2—xy-fx—2z.
362. Find out whether the given function has the greatest value
within the indicated intervals. Find it if any:
| 1, 0 =s x =s 1, 0«sj>=sl,
(a) z = |
i 2 ( 2 - y), 1 < j2

I*6 1 ’ 1^1 ^ ’>-


Ch. 4. Functions o f Several Variables 63

Sec. 4.8. Implicit Functions. Conditional Extrem um


(See [1], Secs. 8.15-8.17)
363. Find the derivatives y X9 y xi of the implicit function y(x )
given by the equation

364. Find — and — , if


dx dy

x cosy-by cos z-bzcos x 1.


365. F(x, y, z) = 0. Prove that
dx dy _ - dy dz dx
~ dy dx ’ dz dx dy

366. Functions w, v of the variables x9y are represented implicitly


by the system o f equations
x —(p(u, v) = 0,
y - \p(u, v) = 0.
du_ du dv dv
Find
dx dy dx dy"
d z_ dz
367. Find if x = u cos v, y = u sin v , z = cv.
dx ~dy

368. Write the equation of the tangent plane to the indicated


surface:
(a) x2-h2y2+ 3 z 2 = 21 at the point ( \ / 3 , 0, 6);
X% i;2 *r2
(b) to the ellipsoid + 1 at its point (x0, y 0, z0).
369. To the surface x2+ 2 y 2+ 3 z 2 = 21 draw the tangent planes
parallel to the plane x + 4 y + 6 z = 0.
370. Write the equations of the tangent plane and the normal to
the surface 3xyz—z3 = a3 at its point (0, a , —a).
371. Find the rectangle of the greatest area having the given
perimeter /.
372. Find the axes of the ellipse 5x2+ $ x y + 5 y 2 = 9.
373. Of all the triangles o f the given perimeter 21 find the one
having the greatest area.
C hapter 5
SERIES
(See [1], C hapter 9)

Sec. 5.1. N um ber Series


(See [1], Sec. 9.1)
374. Using the definition, consider the convergence of the given
series and find their sum s:
1 , 1
^
-f;
1-2 + 2-3 + 3 - 4 + •** ' n ( n + 1)
1
+ (n+l)(n+4) + • • •;
o
J
(C) I

375. Prove that the harmonic series £ — is divergent, applying


n= 1 n
the integral test and Cauchy’s criterion.
376. Using the integral test for convergence of a series, find out
for what a > 0 the series n~a is convergent. Prove that
n—1

sk= X T = 0 (ln (N + iy ),
k=1
N s* 2; (1)
N
S% = Z 4 = 0 [ ( N + l f - % 0<«<1, N ^ l; (2)
k=1

Kr = E i = O r - ) , «>1, JV^l. (3)

Solution. Since the function / ( x ) = x ~ a (a > 0) is monotonically


decreasing to zero on (0, ~ ) as x — the series and the

improper integral J x~ adx are simultaneously convergent or


1
divergent.
As is known, this improper integral converges for a > 0 and
diverges for 0 < a 1, hence, the series £ k~a also converges for
a > 1 and diverges for 0 < a ^ 1.
Let us now estimate the growth of S% (0 < a < 1). Let a = 1;
Ch. 5. Series 65

then
N+l N k+1 N
dx
In E
— <
E ■
*=1 f *
*=1
1 k
N+l N k+ 1 N
dx
__
E j E -
/c=l A=1
k
N+l
1 1
I ~k == s & - -1 + N + l *
k= 2
Hence,
In (7V+1) ^ Sh ^ 1+ In (N+ 1) ^ 2 In ( N + 1), N ^ 2.
The inequality |5jy| ^ 2 In (iVH-1) just proves property (1).
Using the equality (0 < a < 1)
N+l k+ 1
+ iiyy~a
(N+ i _ r dx _ y c dx
1 -a 1- a “ J xa ~ J xa ’

we get (2) in a similar manner. Note that the constants entering into
the symbol O^N1-*) depend on a.
Let us now estimate the remainder a > 1):
oo k+1
* _ f _ y f ^ y 1 _ Da.
(a- k—N
N

o° °° I 1
1____ > y _ y 1 _ pa___
(a -l)/V a_1 ^ (* + &!)*
=N“+ 1 * - 4 k i *a *’
a
^ ^ + TV* ^ a -1 N± (N ^
i.e. property (3) takes place.
Note that in fact we have proved more than this:
_______I______ Dot a 1
(a-OiV”- 1 N a-1 jV*"1 *

In Problems 377 to 380 investigate the given series applying


comparison tests and a necessary test:
377. 1 + 4 + . . . + ^ + . . . .

378. £
n=lV/«2+ 2«'
66 A Collection o f Problems

W I.F T T -

380. (a) £ l n ( l + ?-); (b) £ In ( l + ~ ) ;


k= 1 x 7 *=1 ' 7

<c) ,£
In Problems 381 to 384 investigate the indicated series for con­
vergence with the aid of D ’Alembert’s test or Cauchy’s test.
1 4 n2 « w2
382. - - + -*
Cx
+ ... + —
3n
+ ... = Y ' — .
-in On
3n

Z l / kk-+
\ -\\k
\\k ~ / \ 2 /1 - 1
3*3- I.G t -',) • *4. I f , - , ) .

In Problems 385 to 387 investigate the given series with the aid of
the integral test.

385. y — — — . 386. Y v . \ . - (e > 0).


n= 2
n In nAn In n ,^-l
k= 2 k \n e k v '

387. y
fcx 12
388. Investigate for convergence Ihc series with the general term:
l/n _ 1 In
, x [ 's/xdx x f sm* x dx
<a) w" = J Y * + T ; (b) u„ = J -T O - .
0 0
In Problems 389 and 390 investigate the given series for absolute
and conditional convergence.
r_n n + i
389. 1 - y + y + V2 n^—r1- + ••• • 390. y i - 4n2—
/l=l

391. Show that the series y akbk is absolutely convergent if


A= 1
the series Y a*, Y are convergent.
A:= 1 A= 1
Ch. 5. Series 67

Sec. 5.2. Functional Series


(See [1], Secs. 9.8 and 9.9)
392. Find the domain of convergence of the indicated series:

(a)f-; ( b ) I ( - i ) ”- ;
n= 1 n—1

(C) I ; (d) I *" -2.


k= 1 n=l

393. Investigate the given sequences for uniform convergence:

(a) fn(x) = — , 0 < X < oo ;

(b) f n(x) = x", 0^ x ^ --;

(c) /„(x) = xn—xn+1, 0 ^ x ^ 1;


(d) f n(x) = xn- x 2n, 0 < x < 1.
394. Make sure that the given series are uniformly convergent
throughout the x-axis:
sin kx cos A:jc °° 1
(a) £ ~k\~~ ' (b) £
k=0 k=1 ’ ^ «2[1 +(n*)2]'
395. Applying termwise differentiation and integration, find the
sum of the indicated series:
y2 ytl
(a) ,v+ 2 + . . . + - - - + . • •;
(b) 1 + 2 x 4 - . . . + (az+ 1 ) x " + . . . ;
(c) l - 3 x 2+ 5 x 4- . . . + ( - l ) « - 1( 2 « - l ) x 2» -2+ . . . .

Sec. 5.3. Power Series


(See [1], Secs. 9.11 and 9.12)
396. Determine the radius and the interval of convergence of the
given series and investigate the convergence at the boundary points
o f the interval o f convergence;

(a) E 7+ (b) £ ( « * ) " ; (c) E (”- 1)3"-1 x”~1-


n= 1 n —1 n=2

397. Write the first two nonzero terms of the expansion into a
series in powers o f x for the function:
(a) tan x; (b) tanh x; (c) exp (cos x).
68 A Collection o f Problems

398. Express in the form of series the following integrals:

(a) j exp ( - fi) d r , (b) f arCt7 ' ^ .


0 0
399. Evaluate to within 0.001 the integral
0.2

o.i
400. Expand the function ex in powers of (x ± 2 ).
C hapter 6
DIFFERENTIAL EQUATIONS
(See [3], C hapter 1)

Sec. 6.1. General Concepts


(See [3], Secs. 1.1 and 1.2)
401. Write the differential equation for the indicated family of
curves:
(a) y 2 = 2Cx; (b) y = C i* + C 2; (c) y = Cex\
(d) x2-hy2 = C2; (e) = C\e2x+ C 2 e~x.
402. Construct the isoclinic lines of the given differential equations
and draft the integral curves:
(a) / = x; (b) y = 1 + / ; (c) y = - x .

Sec. 6.2. First-order Equations


(See [3], Sec. 1.3)
In Problems 403 to 408 solve the indicated equations with varia­
bles separable:
403. xy d x + (x + 1) dy = 0.
404. '\/y2+ l d x —x y d y = 0 .

405. « - ' ( l + $ ) = 1- 406. y ' - x y 2 = 2xy.


407. y' = 3j>2/3, y(2) = 0. 408. y' cot x + y = 2, ^(0) = 1.
409. Find the curves in which the point of intersection of any
tangent with the axis of abscissas has the abscissa equal to 2/3 the
abscissa o f the point o f tangency (Fig. 20).
410. A tank is filled with 100 litres of
a solution containing 10 kg o f salt. The
tank is continuously supplied with
water (5 litres per minute) which is
mixed up with the solution. The mixture
flows out at the same rate. How much
salt will remain in the tank in one hour?
411. A body cools from 100° to 60°
during 10 minutes. The ambient temper­
ature is maintained at 20°. When will
the body cool down to 25° ? Fig. 20.
70 A Collection o f Problems
\

In Problems 412 lo 417 solve the equations reducible to the form

y = * -y (£ ) (i)
(for a = 1 we have a homogeneous equation).
412. (xA 2y)dx-~xdy = 0. Hint. Use the substitution y = lx.
413. (y2 - 2xy) dx y x2 dy — 0. 414. y2A-x2y' — at/ .

415. x4 tly = y 2 dx. 416. a*4 dy = ^ dx.


417. x d y = ^x2 cos2 ~A-2yj dx.

418. Find the curve the tangent to which is found at a distance


equal to the modulus of the abscissa of the point of tangency.
An equation of the form
y'+a(x)y+b(x)y2 = c(x)
is called the Riccati equation. In the general case it is not solved in
quadratures. Some of Riccati equations are equations of type (1).
In Problems 419 and 420 solve the given Riccati equations:
419. x2y '+ xyA-x2y 2 = 4.
420. (a) 3 / = - y 2- - . , ; (b) / = £ .
1= 1

In Problems 421 to 426 solve the indicated linear equations:


421. y'A-2y = 4x. 422. xy'—2y = 2xA.
423. x(y'—y) = ex. 424. x y '+ y = exy j>(1) = 1.
425. y = x(y'—x co s x). 426. (sin2 j>+Jccot<y)y/ = 1.
In Problems 427 to 430 solve the given Bernoulli equations:

427. y = y 4 cos x + y tan x. 428. y' = — + —-.

429. xy'—2x2 \ / y = 4y. 430. = —xy2.

Sec. 6.3. Metric Spaces. Contraction O perators.


Solution Existence Theorem
(See [3], Secs. 1.4-1.7)
431. Will the ^-dimensional space Rn be a metric space if the
distance between the points x = (xi, . . . , x„) and y = (yi, . . . , y „)
is determined by the equality:
(a) e(x, y) = max {| .v, - yt \};
1=1.........n
Ch. 6. Differential Equations 71

1, X ^ y,
(b) e(x, y) =
0, x = y?
432. Find out whether the set of all continuous functions defined
on [a , b] is a metric space if

433. Let
0 < x < 1In, a > 0,
1In ^ x ^ 1.
For what a does the sequence /„(x) converge to zero in the sense of
the metric from Problem 432?
434. Will the metric space M = [2, 3) with the metric p(x, y) =
= \x—y\ be a complete metric space?
435. Will the function F(x) = x2 be a contraction operator:
(a) in the complete metric space M = [ - 1 /3 , 1/3];
(b) in the complete metric space M = f —1, 1] ?
In both cases the metric is g(x, y) = | x —y |.
436. Construct an iterated sequence for the operator F(x) = x2
if xo = 1/2.
437. (a) Find the fixed points of the operator F(x) = 1/(1 + x) on
[1/2, 1]. Will the operator F(x) be contracting on [1/2, 1]?
(b) Let the operator F(x) (x — (xi, x 2)) operate in the two-
dimensional metric space F 2 according to the law
F(x) = (xi, x 2)
(mirror image about the axis x 2 = 0). What points are fixed for
this operator?
(c) Let F(x) = (*!, x|), x 6 F 2. What points of the plane R2 are
the fixed points o f the operator F?
438. On the basis of the theorem on existence of a solution of a
differential equation, investigate in what interval [x0—5, x0+ 5 ]
the existence o f a solution of the equation y' = / ( x , y) is guaran­
teed if
(a) xo = 1, yu = y{ 1) = 2, / ( x , y) = 2xy2 on the set
11 A Collection o f Problems

(b) .vo = 0, i’o = v(0) = 1, f (x , v) = 2xy- on the set


| |x | ^ V 2 /4 =
D =
1\y - \ 1 1= b
1'
439. For the equation y' = y x y , y(O) = 1, evaluate approx­
imately y( 1) applying Euler’s method. Jake h = 0.1 for the com­
pulation step.
440. For the equation y f — x + y evaluate approximately y(2)
if y (l) = 1. Take h = 0.1 for the computation step.

Sec. 6.4. Equations N ot Resolved with Respect to Derivative.


Singular Solutions
(See [3], Secs. 1.8-1.10)
441. Find all the solutions o f the given equations; single out
singular solutions if any; give a pictorial representation for each
case:
(a) y'2—y 2 = 0; (b) y'2—4 / = 0;
(c) jW * + 1) = 1; (d) / 2 = 4 /( 1 - y ) ;
(e) xy'2~ 2 y y '+ x = 0; (f) y ( x y '-y ) 2 = y - 2 x y ' .
442. Solve the given equations by introducing a parameter:
(a) x = / 3+ y '; (b) y = y'2+2y'3; (c) x = y' a/1 + / 2-
443. An equation of the form
y = x<p(y')+v>(y%
where cp, y) are some functions, is called Lagrange*s differential
equation. In particular, if cp(y') = y', then it is called Clairaufs
differential equation. These equations are also solved by introducing
a parameter:
y = P, dy = p dx\ y = x<p(p)+y)(p);
dy = (p(p) dx + x(p'(p) dp + y/(p) dp,
or, taking into account that dy = p dx, we get
[p-<p(p)]dx= [x<p'(p)+y)'(p)]dp.
The latter equation is linear with respect to x. We know how to
solve it (if p ^ (p{p)): x = Cf(p)-bg(p), where f , g are known
functions. The system
( x = Cf(p)+g(p),
1y = x<
p(p)+ v (p)
yields a parametric representation of the solution.
Ch. 6. Differential Equations 73

And if p = </ (p) (in this case we have a Clairaut’s equation), then
[x<p'(p)+y'(p)\dp = 0,
whence:
(1) dp = 0, p — C and y = xcp(C)+y)(C) = xC+y)(C) is the
general solution of a Lagrange’s (Clairaut’s) equation. This is a
family of straight lines. Formally, the general solution is obtained
by replacing y' by an arbitrary constant C;
(2) x(])'(p)-\-y(p) = 0. Then from the system
| y = *f(p)+f(p\
I 0 = x<p'(p)+f'(p)
by eliminating the parameter p , we obtain y = %{x). If this func­
tion is a solution of Lagrange’s equation and the uniqueness of
solution is violated, then it is a singular solution of Lagrange’s
(Clairaut’s) equation.
444. Solve the Clairaut’s equation

y = x y —j y'2-
445. Solve the equation
y = x y ' + ^ Y + y 2.

Sec. 6.5. Reducing th e O rd er of a Differential Equation


(See [3], Sec. 1.14)
446. Solve the given equations: (a) y" = cos x; (b) y " ' = x.
In Problems 447 to 457 solve the given equations.
447. x Y ' = y'~. 448. / " = 2 ( / ' - 1 ) cot x.
449. y"' = y". 450. / ' - 2yy’.
451. xy" = 2y y '- y ' . 452. y y" + y'2 = 1.
453. 2yy" = y'2+ y \ 454. yy" = (y')\
455. yy" = y'2+ y Y - 456. xyy"—xy'2—yy' = 0.
457. x2yy" = (y —xy')2-

Sec. 6.6. Linear Equations with Constant Coefficients


(See [3], Sec. 1.16)
In Problems 458 to 462 solve the following equations:
458. y " - 2 / - 3v = 0. 459. 5y " + 6 / = 0.
460. y W - y = 0 . 461. 5 y " - 8 y ' - 4 y = 0.
74 A Collection o f Problems

462. (a) y (A)J\-2y"+y = 0; (b) y " + 3 y '—4y = 0.


463. Prove that the Wronskian
yi(t) ••• y n(t)
//A //A

’*(/) •••
o f the system of solutions y M , . . y„(t) of the equation
y ("\t)+Pi(t)y(n~1)U)+ ■■■+P»(0y(t) = 0,
witty continuous on (a, b) coefficients pi(t), . . . , p„(t) satisfies the
equation

The given equation is an equation with variables separable,


therefore, integrating it within the limits from x0 to x, we obtain

The last formula is known as the Ostrogradski-Liouville formula.


[t implies, among other things, that if fV(x0) ^ 0, then the Wrons-
Idan is not equal to zero at all points belonging to («a, b). In this
:ase, as we know, the solutions yi(t)> . . . y,Sf) are linearly indepen­
dent on (<a, b).
Hint. See Problem 86, (b). Take advantage of the fact that
jfH O = ~/>i(0 ^ B-1)( 0 - • • • -P»(t)yk(t) (k = i , . . n).
464. Solve the following nonhomogeneous equations
y"'—5y"+8y'—4y = f(x ), where
(a) f(x ) = 2e3x; ( b ) /( x ) = 4ex; (c) / ( x ) = 3e2x.
465. Find the particular solution o f the following equations
yW + 2y " + y = f (x), where
(a) / (x) = ex; (b) / ( x ) = sin x;
(c) /(.v) = sin .y + cos ,v; (d )/(.v ) = sin 2.v.
Ch. 6. Differential Equations 75

466. Solve the equation y"+2y' - 3 y = sin x.


467. Write the form of the particular solution of the indicated
nonhomogeneous equations:
y'" —5y" + 6 v' = / (x),
where
(a)/(.v) = e x\ (b)/(.v) = xc-'\ (c)/(.v) = sin v;
(d )/(x ) = .v V v; (c)/(.v)-= v - + . v + 1 ; ( f ) /( v ) = .ve-Ysin x.

Sec. 6.7. Euler’s Equation.


Equations with Variable Coefficients
(See [3], Secs. 1.15 and 1.16) v
468. Find out which of the given systems of functions are line­
arly independent on fO, 1]:
(a) 1, sin2 x, cos2x; (b) 4 —x, 2x+3, 6x+8;
(c )x + 2 , (x + 2 )2; (d) x2, x3, x4.
469. Solve Euler’s equations:
(a) x2y " - 4 x y ' + 6 y = 0;
(b) x2y " - 3 x y ' + 3 y = 0;
(c) x2y " —3xy'+4y = 0 ; (d) x2y'ff—2y' = 0.
470. Solve the given equation (a particular case of Bessel’s
differential equation for v — 1/4, see [3], Sec. 1.24)

= o.

Solution. Let us introduce the substitution y — a(x)z and choose


the function a(x) so as to eliminate the term containing the first
derivative z'. We have
y* = a'z+az', y" = a"z+2a'z'+az";
x2<x.z"+z'[2oc/x 2+ a x l + z j a '^ + a 'x + a x 2——J = 0;
3
2 a '-f— = 0, a = ~r, a ' = ------— , a"
x y /x 2x v x 4x2\ / x ’
O // +. xa / +i x o2a -----=
x~a a X2 .
4 y /x
We Anally obtain
x 2i r2
—~j=.z"“h z = 0, z"+ z = 0.
Vx VX
A Collection o f Problems

The last equation is with constant coefficients, its general solution


being
z = Ci cos x + C2 sin x.
The general solution o f the original equation is

y — ■ (Ci cos xT C >sin x).


V*
471. Solve the equation
y " —2xy' + x2y = 0.
Hint. See Problem 470: a(x) = exp (x2/2).
472. Solve the equation x?y"+xy'—y —/( x ) , where
(a ) / ( * ) = x2; ( b ) / ( i ) = *">.

Sec. 6.8. The Method of Variation of Constants


(See [3], Sec. 1.17)
473. Solve the following equations:

(b) y " + 4 y = 2 tan x; (c) / " + / =

( d ) / ' + 2 / + y = ± e-* .

Sec. 6.9. Systems of Differential Equations


(See [3], Secs. 1.19-1.22)
474. Solve the given system by reducing it to one differential
equation:

(a)

(c) (d)
dz
i x + y = x■
Ch. 6. Differential Equations 77

475. Solve the following homogeneous systems without passing


to one differential equation:

(a) — 2v i +>’2,
I y 2 = 3 y i+ 4 y s;
(b)
x-y,
y-4x;
x(t) = y(t),
(c) i = y +5z'
dz
= -y-3 z;
(d) y( 0 = z(t),
m = x(t).
dx
476. Solve the following nonhomogeneous systems:
, . f x = x - y + 1, f jc = x - y + e > ,
(a) i . . , , (b) \ .
[ y —^ -4 x 4 -^ ; [ y = x —4 y + e 3t.

Sec. 6.10. Solving Equations with the Aid of Power Series


(See [3], Sec. 1.24)
477. / ' + / - x f = 0, y( 0) = 2, / ( 0 ) = 1.
478. y " + x y = 0, y( 0) = 1, / ( 0 ) = 0.
Solution. We shall seek for the solution of y(x) in the form of
the power series
y(x) = a o + 0 iX + 0 2*2-f- . . . .
F in d in g '/, xy and substituting them into the equation, we
obtain a number of relations connecting the coefficients a, (do = 1,
0 i = 0) wherefrom we just find the the values o f these coefficients.
But we may also reason in the following way. Since the power
series is at the same time the Taylor series of the function y ( x \
me may write
A x ) = _ K 0 ) + /( 0 ) * + ^ f * * + * » + ... .

The values j/(0) = 1, / ( 0 ) = 0 are known by hypothesis. The values


of other derivatives obtained from the solution of y(x) at the point
x = 0 can be found with the aid pf the differential equation. From
the equation we have /'(()) = —0 •y(0) = 0. Differentiating the
equation, we obtain
y '" + y + x y ' = 0,
whence
y '" (0 )= -y (0 )= -l.
Analogously,
y W + 2 / + xy" = 0, / 4>(0) = - 2 / ( 0 ) = 0,
/ 5>+ 3 / ' + * / " = 0, / 5>(0) = —3y"(0) = 0,
78 A Collection o f Problems

yM +4/"+x)F > = 0, / 6>(0) = - 4 f ( 0 ) = 4,

yW +(n —2)y(n- v + xy(n~2) = 0, .y(,,)(0) = —(n—2)^(w~a)(0),

Substituting the value >»(/,)(0), we get


, v t x3 , 1-4 . 1-4-7 0,
* * ) = 1- 3 T + _6 T ^ ---- 9 T * + - ” •
The obtained series converges throughout the axis uniformly and
absolutely.
479. y"+ye* = 0, y( 0) = 2, / ( 0 ) = 1.
480. y' = y 4- xe*, y(0) = 0. Find the first three terms of the series.

Sec. 6.11. Stability in th e Sense of Lyapunov


(See [3], Secs. 1.25 and 1.26)
481. Investigate the trivial solution of the indicated system for
stability with the aid of Lyapunov function:
| ± = - x 3- y , x = 2y3—x5,
(a) (b)
I y = y = - x - y 3+ y 5;
f x =
(c)
1 y = *+r-
Remark. If there exists a function v(x , y) such that in a sufficiently
small neighbourhood of the origin there is a region where v > 0
and v = 0 on a part of the boundary of the region (v > 0) and
dv dv dx , 0v dy n ~ , .•
5F = dx dt + 0^ > ® m t^ie re8lon ^ > 0, then the stationary
point is unstable (Cetaev’s Instability theorem).
482. Investigate the trivial solution for stability in the following
systems with a symmetric matrix:
{ X = X -2 y .
(a)
C = ~ lx + y'
x-y;
483.. Investigate
Inv
(b>,
\ yy==- 2- x + 4 y ;
the given systems for stability:
(c)
x = x4- *3y,
y = 3x4-8 y.

| x = 3x, x = x,
(a) (b) ( * = * + 3 * (c)
I y = 2x + y ; \y= ~ 6 x -5 y , y = 2x - y ;

(d)
= —2x—5y,
2x+2y;
(e) f x = x+y,
\ y = y-
C hapter 7
MULTIPLE INTEGRALS
(See [3], C hapter 2)

Sec. 7.1. Integrals Depending on a Param eter


(See [3], Sec. 2.4)
484. Find the domain of definition of the function
2n
FW _

(i.e. the set of values of x where the integral exists).


Investigate the given function for continuity and differentiabi­
lity.
485. Proceeding from the equality
b
C dx 1 4 b
= — arc tan — ,
J + a a

e v a lu a te th e in tegral

f ,d x , , .
J (.V-■+■«-)-

486. Compute the derivative F\x) if

(a ) F(x) = J e-**'1 dy; (b ) F(x) = J - - dy (x > 0 );

(c ) F(x) = j f ( y + x , y - x ) dy.

where /(w , v) is continuous together with its partial derivative f ' .


487. Find the integral of the function F(x) on [0, 1] if
1 i
(a) F(x) = J ( y - 2 x ) d y ; (b) F(x) = | (x+y)dy.
X% X
80 A Collection o f Problems

Sec. 7.2. Multiple Integrals


(See [3], Secs. 2.1-2.5)
In Problems 488 to 490 compute the given double integrals.

488, JJ ’ where D = <3 4>i 2}.


rr f y 2— 4 =*S X < 51
489 (x+2y) dx dy, where D = I >.
V l — 3 “s j 3J

rr n n « I 0 = ^ r < 3 cos w }
490. r2 sin2 w dr dcp, where D = \ L

In Problems 491 to 495 draw the range of integration and change


the order o f integration of the given double integrals.
2 2-y

4 9 1 . 7 = J dy J / ( x , y) dx.
-6 0>2—4)/4
3 2x

492. 1 = J dx j f (x , y)dy.
0 jc/3
2 X 1*2

493. / = J dx J f ( x , y) dy. 494. / = J dx J f (x , y) dy.


1 1/x 0 x*

cV 2 V * ~ £
495 . 1 = J dy J f(x ,y ) d x .
0 yV2c*
In Problems 496 and 497 change the order of integration and
evaluate the double integral.
l x 2 2 —x

496. 7 = ^ dx f (.x + y 2) d y + j dx j (x-\-y2)dy.


0 0 1 0
1 .V

497. 7 = J dx J xy2dy.
o *2
In Problems 498 to 500 evaluate the given triple integrals.
a b c 1 x y

498. J dx j dy j ( x + y + z ) d z . 499. J dx J dy j xyz dz.


o o o o o o
Ch. 7 Multiple Integrals 81

500. j dz j j xyz dx dy , D = {x2-\-y2 < 4 , x > 0, y > 0}. *


o D

501. Set the limits of integration in the triple integral

J JJ f (x , y, z) dx dy dz,
V
where
(a) V is a common part o f the paraboloid 2 az ^ x2+ y 2 and the
sphere x2+ y 2+ z 2 < 3a2 (a > 0).
(b) F is a common part o f the spheres x2+ y 2-hz2 < R2, x2+ y 2+
-J-z2 <= 2Rz.

Sec. 7.3. Change of Variables in a Multiple Integral


(See [3], Secs. 2.6-2.10)
502. Passing to polar coordinates, evaluate the double integral

J J '\/a2—x2—y 2 dxdy,
s
where S = {x2+ y 2 ^ a2, y > 0}.
503. Compute the triple integral

K
where F is an ellipsoid + ^ ^ 1-
504. Compute the double integral
a 's/a2—x2
j dx J \ / x 2+ y 2 dy.
o o
505. Compute the double integral

ls l V '- s - i * *
y2
where 5 is an ellipse -^- + ~ =s 1.
506. Passing to spherical coordinates, evaluate
JJJ Vx2+y2+z2dx dy dz,
v
where V is a sphere of radius R with centre at the origin.
82 A Collection o f Problems

507. Passing to cylindrical coordinates, evaluate the integral


2 y / 2 x —x 2 a

I= J dx J dy j z \ / x 2+ y 2 dz.
0 0 0

508. Compute the double integral

JJ V *2—y 2 dx dy,
s
where S is a circle of radius R with centre at the origin.

Sec. 7.4. Application of Multiple Integrals


(See [3], Secs. 2.11 and 2.12)
509. Compute the area of the figure bounded by the parabolas
y 2 = lO.x+25 and y 2 = —6x+ 9 and make the relevant drawing.
510. Draw the body whose volume is expressed by the double
integral
1 l-JC
j
dx j
(1 - x - y ) d y ,
0 0
and compute its volume V.
511. Find the volume V of the solid bounded by the cylinder
x 2 _i_ z 2 — a 2 an(j tfoe planes y = 0, z = 0, y = x. Make the relevant
drawing.
512. Compute the volume V of the part of the cylinder x2A-y2 =
= 2ax enclosed between the paraboloid x2+ y 2 = 2 az and the
plane z = 0. Make the relevant drawing.
513. Find the area of the portion of the plane

located in the first octant (x > 0, y > 0, z > 0).


514. Find th surface area of the part of the sphere x2+ y 2+ z2 =
= a2 cut away by the elliptical cylinder

515. Find the centre of gravity of the upper half of the ellipse

° = { £ + £ “ • j’ * 0}-
filled with mass of density q = 1.
Ch. 7. Multiple Integrals 83

516. Find the volume:


(a) o f the solid generated by revolving a half o f the ellipse D
(see Problem 515) about the x-axis;

(b) o f the ellipsoid + £ + — «s 1.

517. In the hemisphere


Q = {x2+ y 2+ z 2 a2, z 2* 0}
the density o f distribution o f masses is proportional to the distance
of a point from the centre of the sphere : q ( x , y ,z ) = c \ / x2+ y 24-z2.
Find the centre o f gravity o f this solid.
518. Find the centre of gravity of the homogeneous figure shown
in Fig. 21:
S = {0 ^ y ^ 4 —x2, —2 ^ x < 2}.
519. Find the volume o f the solid generated by rotating the
curvilinear trapezoid S (see the preceding problem) about the
x-axis.
520. Find the moment of inertia o f a homogeneous cylinder
(whose altitude is h and the radius of the base is a) about the axis
which serves as the diameter o f the base of the cylinder.

Solution. Let the z-axis be directed along the axis o f the cylinder,
the base o f the cylinder be found in the plane z = 0, and let the
centre o f the base coincide with the origin. We shall seek for the
moment o f inertia about the y -axis (i.e. about the planes x = 0,
z = 0) (Fig. 22):
Igl = J
G
(x2+ z2) dx dy dz ,

0*
84 A Collection o f Problems

where G is the cylinder under consideration. Computing the inte­


gral, we obtain
a ‘x / a t —x* h

J dx J dy j (x*+ 2? )d z =
-a - v V -*2 0
a
= 4h J -\/a2—x2 ^x2 + dx = (x = a sin t - )
o
n! 2

= 4 a2h J cos2 t[a2sin2 1+ -y-) dl =


o
nl2 n/2 \

1a2j I z S ^ d i + ^ j 1 ± ^ * J = ^ ( 3 ^ + 4 A * ) .

Sec. 7.5. Im proper Integrals


(See [3], Sec. 2.13)
521. Compute the following integrals:

w /J in S S r- <b>J J
0 0 0 0

oo oo

(c) J J xy exp ( —x2—y2) dx dy.


o o
In Problems 522 and 523 compute the given integrals by differen­
tiation with respect to a parameter:
oo

522 . f ± z £ ! dx = F ( y ) (y>-l).
0

OO
r i _ „- »**
523. j - dx = F(y) (y > 0).
o
524. Using the equality
i

o
Ch. 7. Multiple Integrals 85

evaluate the integral

/ t t t * « > > - ' • *=>-')•


0
525. Investigate for convergence the improper integral

/ / ,n v"x2jr y 2 dx d\\
s'
where S is a circle x2+ y 2 ^ 1.
526. Investigate the indicated integrals for uniform conver­
gence :
u OO

(a) F(y) = | dx ( - oo < y < «,);


0

(b) F(y) = j \ f y exp ( - yx2) dx (0 ^ y - <»).


0
C hapter 8
VECTOR ANALYSIS
(See [3], Chapter 3)

Sec. 8.1. Line Integrals of the First Kind


(See [3], Sec. 3.2)
In Problems 527 to 532 evaluate the given line integrals:
C ds
527. I , where r is a segment of the straight line connect-
r
ing the points A = (0, —2) and B = (4, 0).
528. J xy ds, where F is the contour of the triangle with vertices

A — ( —d, 0), B = (1, 0), C = (0, 1).

Solution. The equation of the straight


line AB: y = 0; of the straight line
BC: x + y = 1; of the straight line
AC: y - x = 1 (Fig. 23).

J xy ds = 0;
AB

J xy ds =
BC CB
j xy ds = J x(\ —x) y /2 dx = y /
o
2/6;

0
J xy ds = j xy ds = J x(\ + x) \ / 2 dx ——V ^ /6;
CA AC -1

jxyds=
r AB
J... +J... + f ••• =0+^-1? =°-
BC CA

529. J xy ds , where F is the contour of the rectangle with ver­

tices A = (0, 0), B = (4, 0), C = (4,2), D = (0,2).


530. J xy ds, where r is the part of an ellipse found in the first
r
X 2 y l
quadrant: + -2- = 1, x ^ 0, y ^ 0.
Ch. 8. Vector Analysis 87

531. J y ds , where r is the part of the parabola y = 2 \ / x ,


r
situated in the upper half-plane 0 ^ x ^ 1.

532. J (x —y) ds , where r is the circle: x2+ y 2 = 2ax.


r

533. Find the mass m of the portion of the ellipse x = a cos /,


y = 6 sin t situated in the first quadrant if the density at each of
its points is equal to the ordinate of this point.
534. Find the area S of the lateral surface of the parabolic
3
cylinder y — -g- x2 bounded by the planes z = 0, x = 0, z = x,
y = 6.
Solution. Geometrically, the line integral
\ f ix , y)ds,
r

where / (x, y) s* 0, can be in­


terpreted as the area o f a
cylindrical surface with gener­
atrix parallel to the z-axis
whose base is the contour of
integration r and whose alti­
tudes are equal to the values
o f the function / ( x , y). There­
fore the desired area (see
Fig. 24)

r
Jx ds.
3
where T is part of the parabola y = g- x2 (0 ^ x ^ 4). Computing,
we obtain
4 __________________________ 4

S = | x ^ \ J 1 + ^ x j 2 dx = JV 16 + 9x2rf(16 + 9x2)
0 0

= ,08 (16+ 9.v'J):i/2 [ - 1^- (10 v T O - 1).

r535. Compute the area of the lateral surface of a circular


cylinder found under the first turn of the helical line x = a cos /,
y = a sin t, z = lot 2n) and above the plane z = 0.
88 A Collection o f Problems

536. Find the coordinates of the centre o f gravity of a homoge­


neous half-arch o f the cycloid x = a(t—sin t)9 y = a(l — cos t)
(0 ==£ / =stz).
537. Find the moment of intertia about the z-axis (that is, with
respect to the planes x = 0, y = 0) of the first turn F of the helical
line x — a cos /, y = a sin /, z = bt (0 t ^ 2 ri).

Sec. 8.2. Integral of a Vector Along a Curve


(See [3], Sec. 3.3)
538. Evaluate the line integral of the second kind

J (y2 d x + x 2 dy ),
r
y
where J1 is the upper half o f the ellipse
b x —a cos /, y = b sin t traced clockwise
(Fig. 25).
Solution. When a point moves along
the curve F in the indicated direction,
Fig 25 the parameter t changes from n to 0.

Therefore

J (y2 dx+ x2 dy)


r
o
= J Ib2 sin2 t(—a sin t)+ a 2 cos2 t-b cos /] dt
n
n
= ab j [b sin3 t—a cos3 t] dt
o

= ab — J h(l —cos2 t)dcos t —J a(l —sin2 /)rfsin t ab2.


3

539. Compute

jx d y,
r
where r is the contour o f the triangle formed by the coordinate
axes and the straight line x + y = 2 traversed in the positive direc­
tion (i.e. anticlockwise).
Ch. 8. Vector Analysis 89

540. Compute the value of the integral

J (y d x + x2 2 dy),
r

where: (a) r is the arc of the parabola y = 4 —x 2 found in the upper


half-plane and traversed clockwise; (b) F is the polygonal line
connecting the points ( - 2, 0), (0, 4), (2, 0); (c ) F is the segment
[—2, 2] o f the x-axis.
541. Evaluate the integral

J (x d y + y dx ) 9

where (a) r is an arc o f the parabola


y = V x> (b) T is a segment o f a straight
line, (c) r is the arc of the parabola y = x 2
connecting the points (0, 0) and (1, 1) in
the direction indicated by the arrows (see
Fig. 26).

Sec. 8.3. Potential. The Curl of a Vector


(See [3], Sec. 3.4)
542. Find the gradient of the function
u = x2+ 2 y 2-h3z2+ x y —6z.
543. At what points in space is the gradient o f the field
u = x3-t~y3+ z 3—3xyz
(a) perpendicular to the z-axis? (b) equal to zero?
544. Find the curl o f the vector a = {x9 y 9 z}9 i.e. the radius
vector, o f the point (x9y 9 z).
545. Find the curl of the vector a = { z + y 9 x9y}.
546. Find out whether the vectors (a) a = x2i+ j^ j+ z 2k, (b) a =
= y n + x z } + x y k 9 (c) a = zi-\-xzj-hxyk have a potential in the
entire space R3 .
547. Find the potential function for the vector a = {x2, y29 z2}
in the space R3.
Solution. The curl of the vector a is equal to zero in R3. Besides,
the space R3 represents a simply connected domain. Therefore the
vector a has a potential which is found by the formula

U(x 9 y 9 z) = J (P dx+Q d y + R dz ) 9
90 A Collection o f Problems

where for the curve r we may


take the polygonal line (Fig. 27)
connecting the points (0 ,0 ,0 ), ( x,
0, 0), (x, y, 0), (x, y, z). In this
case the integral of the second
kind is independent of the inte­
gration path. Computing, we find

U(x,y, z) = y (A-Hy-Hz3).

Sec. 8.4. Exact Differential Equations


of First O rd er
(See [3], Sec. 3.5)
548. Which of the given equations are exact differential equa­
tions:
(a) (2x-\~3y) dx-\-(3x—Ay) dy = 0,

(b) — - = 0, (c) (2 - y ) dx+ x dy = 0?

In Problems 5 49 to 555 solve the given equations:


549. (2 x + 3 x2y) dx+(x*—3y2) dy = 0.
550. 2 xy d x+ (x2—y2) dy = 0.
551. e~y dx—(2y+xe~y) dy = 0.

552. ¥ + y d x - ^ P ^ d y = 0.
553. yz d x + x z dy-\-xy dz = 0.
554. (x+ z) d y + ( y + z ) d x + ( x + y ) dz = 0.
555. 2xv dx + (x 2-\~z2) dy-\-2yz dz = 0.

Sec. 8.5. Green's Formula


(See [3], Sec. 3.7)
In Problems 556 and 557 transform the given line integrals
about closed (positive oriented) contours JT into double integrals
over the regions Q bounded by these contours:

556. | ((1 - x2)y dx+ x(\ + y 2) dy).


r
Ch. 8. Vector Analysis 91

557. J ((exy
4-2x cos y) dx 4- (exy —x2 sin y) dy ).
r

558. Evaluate J ( (x y + x + y ) dx-\~{xy-\-x—y) dy), where r is


r

Ihe ellip se ^ 4 = 1.

559. Compute the difference between the integrals

h = J
rt
((*+.y)2 d x - ( x - y f dy),

h = J ((x+ ^ )2 d x - { x - y f dy),
rt
where A isanarcof the parabola y = .x2, and is the line segment
joining the points (0, 0), (1, 1).
560. Evaluate the integral

| ( —x2y d x + x y 2 dy),

where r is a positive oriented circle x24-j>2 = R2.


561. Find the area o f the figure Q
bounded by the astroid (Fig. 28):
x — a cos3 t, y = a sin3 t.
562. Show that the work done
by the force a = {2xy, x2} in dis­
placing a point o f mass m depends
only on its initial and final positions
and is independent of the shape of
the path. Compute the work A done
by the force when displacing the
given point from the position (1, 1)
to (2, 5).

Sec. 8.6. Surface Integral of the First Kind


(See [3], Secs. 2.11 and 3.8)
563. If a surface is represented parametrically
x = x(u, v), y = y(u, v), z = z(u, v)

o f in vector form
r(w, v) = x(u, v)lA-y(u, v)\+z(u, v)k.
92 A Collection o f Problems

where (w, v) £ Q and the functions x, y , z have continuous partial


derivatives on the closure o f £?, then the area S of this surface is
expressed by the double integral

S = j j ^/EG-F^du dv =
Q Q
JJ
\ / | r«l2 lru|2“ (r«, rv)2du dv ,

where

« - w - ( £ ) ,+® ),+© \
17_ / X _ 8 x 0 JC , 0^ , 0 Z
l “’ v> du dv~*~du S v ^ d ii d v '
Indeed, as we know (see [3], Sec. 2.11),

S = JJ | r„ X r„ | du dv,
£}
but
i j k

0A- dy dz
du du du

dx
dz dz
dv dv dv

therefore

Expanding the Jacobians under the radical sign, squaring them


and performing all necessary algebraic transformations, we obtain
the sought-for formula.
This formula for computing an area is convenient in a number of
cases, particularly, when the vectors ru and rv are orthogonal
(0*K> *v) = 0)«
In this case

S = JJh |r„|-|r„| dudv.

Find the area o f the surface:


x = u cos v , y = u sin v9 z = Av,
0 u ^ 3, 0 c v <= 71.
Clt. 8. Vector Analysis 93

In the present case


ru = (cos v , sin v , 0), rv = ( —u sin w cos v, 4);
(r„, r„) = 0, |r„|2 = 1 , |r„|2 = 16 + u 2.
Therefore
7T 3

S = J J 1 • V l6 + w 2rfw dv = j dv j y / \ 6 + u 2du
Q 0 0
3

= J* \ / 164"w2 / y/16+w2+8 In (w+ 16+w2)J


tc
o ^ 0
= --(1 5 + 1 6 In2).

564. Evaluate J (x2+ y 2) ds , where S is a sphere x2+ y 2+ z 2 = d1.


s
565. Find the mass of the portion o f the sphere x2+ y 2+ z 2 = a2
found in the first octant (x ^ 0, y ^ 0, z ^ 0) if the density o f mass
distribution over the sphere is equal to V x2+ y 2.

566. Compute J z dS , where S' is part of the surface of the heli-

coid
x — u cos v , j = u sin v, z = v ( 0 ^ u ^ a 90 ^ v ^ 2 n).

Sec. 8.7. A Vector Flux Through an O riented Surface


(Surface Integral of th e Second Kind)
(See [3], Sec. 3.12)
The flux of the vector a = (P ,Q ,R ) through an oriented surface
5*

J (a, dS*) sJ (a, n) dS


=
s*
J (P (n, x)+ Q
= cos cos (n, y) + R cos (n, z)) dS
s
is equal to the surface integral o f the first kind of the scalar product
(a, n), where n is the unit normal defining the orientation of S*.
If a surface is given by the equation
r(t/, v) = x(u, v)\-by(u9 v)j + z(w, v)k ((w, v) € &),
94 A Collection o f Problems

then

dS = \rux r v\du dv, n = ± —~


I»ii A rw
therefore

J (a, dS*) = ± QJ ((a, rHXr„) du dv.


s*
Hence it is seen that to the different sides of the surface S there
respond the surface integrals o f the second kind o f the vector a
differing in sign.
If the surface S is defined by an implicit equation F(x, y, z) = 0,
then the direction cosines of the normal are determined by the
formulas

where
D = ± | grad F\ = ± ^ F * + F * + F j.
The sign before the radical must be made consistent with the
appropriate side of the surface S.
Surface integral of the second kind is also designated by the
symbol

J (a, t/S*) = J ( P dy dz+Q dx dz+ R dx dy).


s* s*
This notation is suitable for the case of explicit representation of a
surface. If the surface S is simultaneously defined by the equations
x= fi(y,z), (y , z ) e Q u
y = f 2(x, z), (x, z ) € 0 2;
z= M x,y), ( x , y ) £ Q s,
then
J (Pdy dz+ Q dx dz+ R dx dy)
s*
± JJ P( fi(y, z), y, z) dy dz± JJ Q(x, f 2(x, z), z) dx dz
Ch. 8. Vector Analysis 95

where sign (plus or minus) is taken depending on orientation of the


surface (for instance, the first integral is taken with the plus or
minus sign depending on what angle (acute or obtuse) is formed by
the normal to S with the x-axis).

567. Compute | z dx dy , where S* is the exterior side of the


s*
e l l i p s o i d - + J + ^ = 1.

Solution. Method 1. The surface S is given by the explicit equa­


tion

The cosine o f the acute angle formed by the outward normal with
the z-axis for the upper half of the ellipsoid is determined by the
formula
cos (n, z) = l / - \ / l + zl + z5
(and the lower half must be taken with the minus sign). Therefore
we take the plus sign in the corresponding formula for the upper
half St of the ellipsoid:

5.
Analogically, for the lower half S t :

J zdxdy = 1
-^ r-jfd x d y

thus,

where Q = {^ 2"+ ^ ^ l} . Computing the last integral, we obtain


96 A Collection o f Problems

Method 2. Let us pass to parametric representation of the ellip­


soid:
x = a cos u cos v9 0 ^ v 2jz
• y = b cos u sin v, A= ■ w w
—— <= W —~
. z = c sin h, I 2 2
Here
r„ = {—a sin w cos v, —6 sin asm v, ccosw},
rw = { —a cos u sin v, fc cos u cos v, 0}.
In the right-handed system of coordinates the outward normal to
the ellipsoid is determined by the equality
r«Xr„
n
|r„XrJ
For example, let us take the point x = a,
y = 0, z = 0 on the ellipsoid. This point
corresponds to the parameters u = v = 0.
At this point ru = {0, 0, c}, rv = {0, fe, 0}
(Fig. 29). The vector product r„Xr„ to­
gether with the vectors ruand rv must form
a right-handed triple (that is, be oriented in
the same way as the coordinate system),
i.e. the vector r „ x r r is in the direction
o f the negative x-axis (see Fig. 29).
Hence,
j z dx dy = — JJ (a, ruX rv) du dv
5* A
Sx dy
, D(x, y) du du
D{u, «)
dudv = —
fJZ dx
A 0]y
du dv
dv dv
n!2
sin2 u cos u du dv - 2nabc J sin2u cos u du
-n il
7l/2
Anabc J sin2ud sin u = --nabc.

568. Compute
J (x2 dy d z + y 2 dx dz -f z2 dx dy),
s*
where S* is the outer side of the hemisphere x2+ y 2+ z 2 = a2
(z ^ 0 ).
Ch. 8. Vector Analysis 97

569. Compute

J (yz dy d z+ xz dx dz+ xy dx d y \
s*
where S* is the outer side of the te­
trahedron bounded by the planes
x = 0, y = 0, z = 0, x + y + z = a y
(Fig. 30).

Fig. 30.

Sec. 8.8. Gauss-Ostrogradski Formula


(See [3], Sec. 3.13)
570. Find the divergence o f the vector a = (x3, y3, z3).
571. Find the divergence o f the vector a = / ( r ) — , where r =
= | r | , r = x i+ j> j+ z k ,/is a differentiable function.
572. Let u = x2+ y 2+ z 2. Find div (grad u).
573. Compute curl a if : (a) a = r, (b) a = / (r) c, where c = c ii+
+ C2j+C3k is a constant vector, r = |r| = |xi+}>j+zk|.
574. Using the Ostrogradski-Gauss formula

JJJdiva dG J(a, n) dS,


=
G S

where n is the unit vector o f the outward normal to the surface S


which is the boundary o f the solid G, transform the following sur­
face integrals o f the second kind:

(a) J (xy dx d y + y z dy dz+ xz dx dz);


s
(b) J dy d z + y 1 dx d z + z 2 dx dy);
(x2
s
, x f x cos a cos @+ 2 cos V
w J S
V**+y*+*

(d) J (— COS a + | - cos £ + - - cos yj dS.


s
In Problems 575 to 577 evaluate the indicated surface integrals
with the aid o f the Ostrogradski-Gauss formula.
98 A Collection o f Problems

575.
J (x dy d z + y dx dz-\-z dx dy),
s
where S is the outer side of the pyramid bounded by the planes
x = 0, y = 0, z = 0, x-hy-hz = 1.
576. j z 2 dx dy , where S is the outer side of the ellipsoid
s

577. J z^dxdy , where S is the outer side of the ellipsoid from


s
Problem 576.

Sec. 8.9. Stokes’ Formula


(See [3], Sec. 3.15)
Stokes’ formula, establishing relation between the circulation of
aa
the vector = ( P , Q, R) around a contour r and the flux of the
vector curl through an oriented surface S* (with the rim r ), can
be written in the following expanded form:

J (a c/e) J (P dx+ Q d y + R dz)


=
r r
cos a cos/? cos y

- \
0
'0jc
0
0v
0
dz
dS = J (n, a) dS,
curl

P Q R
where cos a, cos /?, cos y are the direction cosines of the normal n
to the surface S. The contour r is oriented in accordance with the
orientation of S* (Fig. 31).

Fig. 31. Fig. 32.


Ch. 8. Vector Analysis 99

578. Applying Stokes’ formula, compute the line integral (circu­


lation)

J (y d x + z d y + x dz),
r

where jT is a circle x2+ y 2+ z 2 = b2, x + y + z = 0 traced anti­


clockwise when viewed from the side of the positive x-axis.
Solution. This tracing o f the contour r corresponds to the ori­
entation o f the piece of the plane x + y + z = 0 lying inside the
sphere x2+ y 2+ z 2 = b2 by the normal directed upward to the right
(as is shown in Fig. 32):
cos a = cos ft = cos y = 1/ \ / 3
(F(x,y, z) - x + y + z = 0, F'x = F'y = F'z = 1, |grad F | = ^/3).
In this case for the vector a : P = y ,Q = z, R = x, therefore

| (y d x + z d y + x d z ) = —J (cos a + c o s fi+cos y) dS = — | dS9


r s s

where S is a circle of radius b lying in the plane x + y + z = 0. The


surface integral o f the first kind of a unit function is obviously
equal to the area o f she surface, therefore
J (y d x + z d y + x dz) 7tb2 = —y/3nb2.
V3
r

579. Compute, by Stokes’ formula and directly, the circulation

J ( ( y - z ) dx+ ( z - x ) d y + ( x —y) dz ),
r

where r is an ellipse: x2+ y 2 = 1, x +


+ z = 1 (Fig. 33) traced anticlockwise
when viewed from the direction of the
positive z-axis.
580. Using Stokes’ formula, compute
the integral

J (( y + z ) d x + ( x + z ) d y + {x + y ) dz ),
r
y
where r is a circle: x2+ y 2+ z 2 = b2,
x + y + z = 0 (see Problem 578). Fig. 33.
C hapter 9
FOURIER SERIES
AND FOURIER INTEGRAL
(See [3], Chapter 4)

Sec. 9.1. Trigonom etric Series


(See [3], Secs. 4.1 and 4.2)
581. Construct the graphs of the partial sums Si(x), S 2(x) of the
series
cos kx
E
k =l
582. Show that the function
// \ r-i sin kx
/ ( * ) = X -TT-
k= 1
is continuous and has a continuous derivative on ( — 00 5 00 )•
583. Find out at what points o f the period the series
Z cos kx
- nKr -
,
(a > °)
k =1
is convergent.
584. How many times is it possible to differentiate the series
°° sin kx
E 0
k=i
2k ‘

585. Find the domain of convergence of the series


°°
Z sin kx
. ~ k \~ ’

Sec. 9.2. Fourier Series


(See [3], Secs. 4.3, 4.4, and 4.6)
586. Expand the given function f (x ) into a Fourier series with
period 2 n if
(a) f(x ) = x on {—Tt, 7i)\
(b ) f ( x ) = ^ on (0 ,2 * );
( c ) /( x ) = |x | on {-7l,7t)\
Ch. 9. Fourier Series and Fourier Integral 101

— 71 < X < 0,
( d )/(x ) = I ° ’
I 0 < x < tz;
—71 < X ^ 0,
w /w = { J ;
0 < .V < 7l\
(f) / ( x ) = sin ax , — < x < Jt;
(g) / ( x ) = cosh ax, —itt < x < JT.
587. Investigate the convergence o f Fourier’s series for the perio­
dic function
_ | -7 1 X 0,
/ « (see Problem 586, (d)).
\ x, 0 X 71

Solution. The given function is bounded, piecewise continuous


and piecewise monotone on [0, 2ti]. At the point o f discontinuity
x = 7i it is not defined (see Fig. 34),
i.e. it does not satisfy Dirichlet’s
condition. But the value of Fourier’s
coefficients is independent o f what
values are attained by the function
/ ( x ) at a particular point. Therefore,
let us define the function / ( x ) at the
point x = t i, putting
\ /O + 0 ) + /( :7 i- 0 ) 0+JI 71
f(n ) = --------- j--------- = ~ tT ~ ~1 '
Then the Fourier series of this function (see Problem 586, (d))
ti 2 yi cos (2 k+ \)x ^
4" - h (2k +1)2 ~ ir-
converges at all the points x £ [0, 2tc ] to the extended function, in
particular, it converges to tt/2 at the point x = ti, i.e.
71 71 ( 2 1
2 = £ o(2k+iy'
whence
o 00 ,
71 ^r-i 1

T = ^ 0(2FT^*
588. Expand the indicated functions into Fourier’s series in terms
o f sines and cosines:
(a) / ( x ) = x, 0 < x < ti\

(b ) / ( x ) = , 0 < x < ti.


102 A Collection o f Problems

589. Expand into Fourier’s series the function / (x) with period
21 defined by the formula / ( x ) = |x | on ( —/, /).

Sec. 9.3. O rthogonal Systems of Functions


(See [3], Secs. 4.5, 4.8 and 4.9)
590. Find the scalar product of the functions:
(a) f(x ) = x, (p(x) = sin x, 0 < x n\
(b) / ( x ) = sin x, </>(x) = sin x, 0 ^ x ^ tz / 2.
591. Find the norm

o f the indicated functions/(x) (a ^ x ^ b) :

(a ) / ( x ) = x2, 0 < x * sl;


(b) /(x ) = cos x,0 < x ^ n\
(c) /(x ) = ex, 0 ^ x < 1;
(d) /(x ) = 1 - x , 0 ^ x ^ 1.

592. Let there be given a sequence of


functions (Fig. 35)

{ «a- « i +ax,
0 ^ x ^ 1In,
A 1/ 1
0, \/n < x < 1.
For what a 0 is this sequence convergent to zero in the mean?
593. Investigate the sequence
^ x ( s i n 7tnax , 0 x ^
/»(*) = { „
[ 0, n~a < x < n
for uniform and root-mean-square convergence.
594. Prove that the set of Legendre polynomials

*«(*) = 2 5 7 (x2- 1)n = 0, 1, . . .)


is a complete set o f orthogonal functions on the interval ( —1, 1),
i.e.
i
| Pm(x)Pn(x) dx = 0 (m ^ n \
Ch. 9. Fourier Series and Fourier Integral 103

and satisfy the condition

j P%x) dx = — .
-l
Solution. Note that from the definition of the Legendre
polynomial P„(x) it follows that its coefficient o f .v" is equal lo
(w-H) .M2/j-1)2/j
. Hence
2nn!
d"Pn(x) _ (M+ l)(/H 2) . . . In
dxn 2n
Further, it is obvious that if k < /i, then
d*
dxk
where A(x) is a certain polynomial. Therefore for k ^ n
dk
(v2- 1)- = 0.
dxk JC= ± 1
Let, for the sake o f definiteness, m > n. Integrating by parts n
times, we have
i i
J P„(x) P„,(x) dx = c J P > ( x ) ~ (x2- 1)"' dx
-1

= c P „ ( x ) ^ r l (x2- \ y - C J P n ( x ) - ^ i (x2- l)m dx

1
= ( - \)C IJ'P'n(x)
-1
(*S- l)m dx =

1
. . . = ( - 1)» c I /><">(*) £ 1 (x2- 1r dx

(w + 1) . . . 2n C dm’ n , o 1W ,
= (-1 )" ^ 2n J
d x ^ {x
-1
X) dX
A m —n —1
= 0,

where
104 A Collection o f Problems

In a similar way, we obtain

J P„(x)Pn(x)dx = (-1)» (/»+!) ... 2/i


n\2**
J (x2—1)" dx
(n + 1) . . . 2« f
= 2 »! 22" f (I-x*)"dx.

Let us evaluate the integral

In = J
o
(1 - x 2)" dx, (« = 0, 1, . . . ) .

Obviously, Io = 1, h = 2/3. Integrating by parts, we obtain the


following recurrence formula:
/ - 2n I
~ 2n + l Jn~u
Hence,
. _ 2-4-6 . . . 2n _ 2M-/i! 22n(/i!)2
3-5-7 . . . (2/i+'l) ~ 3-5 . . . (2/1+1) ~ (2zi)! (2/i +1)
Thus

(n+1) . . . (2/i-l)2 n 22”(/i!)2


’v = 2
22"/i! (2/i)! (2/1+1) 2/1+ 1 ‘

Sec. 9.4. Fourier Integral


(See [3], Secs. 4.12 and 4.14)
595. Find the cosine transform of the function

m - V [ 0: , x > a.
596. Find the function defined on (0, «>) whose cosine transform
is equal to
2 sin as
V s
597. Find the cosine transform
insform of the function
f cos x, 0 < x < a,
/(* )= {I 0, x > a.
598. Find the sine transform of the function
f (x ) = e~5X/x.
Ch. 9. Fourier Series and Fourier Integrat 105

599. Evaluate the following integrals :

s \ C sin xs dx , A

(b) J e~4x sin 3.v cos 2x dx;


o
oo

(c) J e~3x cos 3x cos 4x dx.


o
C hapter 10
EQUATIONS OF MATHEMATICAL PHYSICS
(See [3], C hapter 5)

600. Let a function f(0) with period 2n be defined on (—n, n)


by the equality /( 0 ) = |0|. Construct the harmonic in the unit
circle function generated by these boundary values, and find out at
what rate, in the mean square, the function w(p, 0) tends to its
boundary values / ( 0 ) for g 1-0.
601. Find the solution of the heat equation

for the initial condition

u(x, 0) = / ( . v) = T— A , 0 =£ -V n.
and the boundary condition
m(0, t) = u(n, t) = 0.
Estimate the integral

for t —+ 0, i.e. find out the character of convergence in the mean


of the solution u(x, t) to f (x ) as t + 0.
602. Find the solution of the string-oscillation equation
62u _ &u
0 /2 “ 0 a:2
for the initial conditions

and for the boundary conditions


w(0, /) = u(n, t) = 0.
Ch. 10. Equations o f Mathematical Physics 107

603. Solve the equation of oscillation of an endless string


02W __ 02W

for the initial conditions

u(x, 0) = u',(x, 0) = y * xs (-CO < X < °°).

604. Let u(x, j ) be a harmonic in the upper half-plane function


attaining the values of f(x ) for y — 0. Prove that if f(x ) V x
satisfies the condition
\ f ( x + t ) - f ( x ) \ ^ L \t\a, 0 < a < 1,
then
\u(x,y)-~f(x)\ < c r (y > 0),
where c is a constant independent of x and y.
605. Find the stationary distribution of temperature u(x, y) in the
upper half-plane and the isotherm (level line) u — 1/2 if on the
x-;axis the temperature

fix ) = \ .
[ o, |x | => /
is maintained.
Hint . The function u(x, y) is harmonic in the upper half-plane.
606. Find the temperature distribution in an endless rod if the
initial temperature distribution was
f ix ) - exp (—x2)
(put a = 1 in the heat equation).
607. Prove that the Legendre polynomials y = P„(x) (see Prob­
lem 594) satisfy the differential equation
Chapter 11
FUNCTIONS OF A COMPLEX VARIABLE
(See [3], Chapter 6; [1], C hapter 5)

Sec. 11.1. General Concepts


(See [1], Sec. 5.3)
608. Find the moduli of the following complex numbers:
(a) z == 4 + 3 /; (b) z = cos a —/ sin a; (c) z = —2 + 2 y/3i.
609. Write in the trigonometric form the following complex
numbers: (a) z = —1 —/ y /3 ; (b) z = —y / 2 + / y/2.
610. Represent in the exponential form the following complex
numbers:
(a) z = —2; (b) z = /; (c) z = —1 —/ \ / 3 ;
(d) z = sin a —/ cos a ( t t /2 < a < n).
611. Compute ( —1 + / \ / 3 ) co.
Solution. Represent the number z = —1 + / \ / 3 in the exponential
form
- 1 + / \ / 3 = 2 exp (i
Hence,
( - 1 + / \ / 3 ) 60 = 2f>0 exp (40 to) = 260.

612. Compute: (a) ( \ / 3 —3i)6; (b)

613. Find all the values o f the radicals:

(a )-v /l - i ; (b) \ / i -
614. Let Re w = x, Im w = y2 (w ^ 0). Find iv, 1/w.
615. Prove the indicated equalities:
(a) z + z = 2 R e z ; (b) z —z = 2i Im z; (c) |z| = \z\.
616. What curves are given by the following equations:
(a) \z—z0\ = r, r > 0; (b) | z + c | + | z —c\ = 2a, a > c—^real
numbers; (c) Re (1/w) = 1/2, Im(l/w) = 1/4, w = x+2yi\
(d) Im z2 — 2; (e) Im ( - ) = 1?
Ch. 11. Functions o f Complex Variable 109

617. Find the images of points z0 for the indicated mappings:


(a) w = z2, z0 = /; (b) — w = z/z, z0 = 2 + 3 /.
618. Into what curve is the circle |z| = \ / 2 mapped with the
aid o f the function w = z2?
Solution. We have Rew = jt2—j 2, Imw = 2xy. Eliminating x
and y form the system
u = x2—y 2,
v = 2xy,
x2-hy2 = 2,
we obtain
u2+ v 2 = (x2+ y 2)2 = 4,
i.e. this is a circle of radius 2 with centre at the origin in the uv-
plane (in the w-plane). Note that the circle \w\ = 2 is described
twice when the point z traverses the complete circle |z| = \ / 2 ,
since
Arg w = 2 Arg z+2fc7r.
The present problem can also be solved by applying another
technique. The equation of the circle | z| = \ / 2 may be written in
the form
z = \ / 2 e i(P,
where 0 < (p < 2 jc. Therefore
w = z2 = ( \ / 2 e i<p)2 = 2e/2^.
Hence it follows that the circle \z\ = V 2 goes into the circle
| w | = 2 in the mapping w = z2, the circle \w\ = 2 being described
twice when the point z traverses the circle \z\ = y / 2 once in the
positive directibn (anticlockwise).
619. Find out on what lines of the w-plane the following curves
in the z-plane are mapped with the aid of the function w = 1/z:
(a) |z| = 1/2; (b) arg z = n) 4, (c) Re z = 0.

Sec. 11.2. Limit of a Function. Derivatives


(See [3], Secs. 6.1 and 6.2)
620. Find the limit of the function w = z at the point z = /.
621. Find out whether the limit of the function w = z/z exists at
the point O = (0, 0).
622. Will the function w = Re z be continuous in the z-plane?
110 A Collection o f Problems

623. Which o f the following functions have a derivative:


(a) w = z2, (b) w = Re z, (c) w = z; (d) w = z-z?
624. Find the modulus of expansion and the angle of rotation at
the point z0 = 0 in the mapping w — sin z.
625. At what points is the mapping w = f (z ) conformal:
(a) w = z3; (b) w = cos z; (c) w = zezl

Sec. 11.3. Cauchy-Riemann Conditions. Harm onic Functions


(See [3], Secs. 6.3 and 6.4)
626. Find out which of the given functions are analytic:
(a) w = zez; (b) w = zz2; (c) w = sin 3z; (d) w = cosh 2z.
627. Restore the analytic function/(z) = u+iv by its known real
part u(x, y ) :
(a) u = x2—y 2+2x; (b) u = x2—y2+ x y \
(c) u = rep cos cp+ r In r sin cp (z = rei(p).
628. Find all the harmonic functions of the form u = q)(x2+ y 2).
Solution. We have
U& — 4x2(p"(x2+ y 2)+2(p'(x2+ y 2),
Uyi = 4y2<p"(x2+ y 2)+2cp/(x2+ y 2),
whence
zIm = 4( x2+ ^ 2) ^ ,/(a:2+ ^ 2)+499/(^2+J;2).
Thus, in order for the function u to be harmonic (Au = 0), the
following equality must be fulfilled:
(x2+ y2) (p"(x2+ y2) + y'(x24- v2) = 0.
Setting x2+ y 2 = we obtain
(p"(t) _ 1 cl In (p\t) _ 1
(p\t) ~ T ’ dt ~~~ ~ T y
<p'(t) = C/r, 9?(r) = C In t + Cx.
Hence, the harmonic functions have the form
u = C In (x2+ y 2) + C u
where C and Ci are arbitrary constants.
629. Find all the harmonic functions of the form u = <p(ylx).
Ch. 11. Functions o f Complex Variable Ill

Sec. 11.4. Simplest Conformal Mappings


(See [3], Secs. 6.2 and 6.15)
630. Find the conformal mapping transforming the upper half­
plane onto itself.
631. Map the unit circle | z | ^ 1 onto the upper half-plane so
that the points zi = —/, z2 = 1, z3 = i go into the respective
points wi = — 1, w2 = 0, w3 = 1.
Solution. The desired mapping is realized by the linear-fractional
function
w+ 1 1 + 1 _ z + z e i+ z _ i'( 1 - z)
w-0 ' 1-0 ” z-1 ' / - l °r W 1+ z
The inverse function
_ i —w
w+ z
maps the upper half-plane onto the unit circle so that wk go into
zk (k = 1, 2, 3).
632. Map conformally the angle 0 < q> < tt/4 onto the upper
half-plane (Fig. 36).
633. Map conformally the strip 0 ^ y < n\ (a) onto the upper
half-plane; (b) onto the entire plane.
634. Map the vertical strip 0 ^ ^ n/4 onto the unit circle
|w| ^ 1 (Fig. 37).
112 A Collection o f Problems

Solution. Let us turn the vertical strip into a horizontal one.


To this end, let us perform a rotation through jr/2 which is realized
by the function
z* = z exp (i y j = iz.

In the plane z* = x*+ iy* we obtain a horizontal strip 0 < y* < ti/4.
Let us expand this strip fourfold:
z' = 4z* = 4 iz.
In the plane z' we obtain a horizontal strip 0 ^ y' ^ n. We then
map this strip onto the upper half-plane with the aid o f the expo­
nential function
z" = e* = e*iz.
Finally, we map this half-plane onto the unit circle \w\ 1, for
instance, with the aid o f the function from Problem 631:
_ i ~ z" exp(4iz)
z" + / /-f-exp (4/z)
635. Find the entire linear function which maps the triangle with
vertices at the points 0 , 1, i in the z-plane onto a triangle with res­
pective vertices 1 + /, 0 , 2 in the w-plane.
636. Find the conformal mapping of the circle |z| < 5 onto the
circle \ w\ < 1 so that the points 5, 4 + 3 /, —5 go into the points
1, /, - 1 .

Sec. 11.5. Integration of Functions


of a Complex Variable
(See [3], Sec. 6.6)
637. Compute the integral
J (1+ /—2z) dz
c
along the lines connecting the points zA= 0 , z^ = 1 + /: (a) along
a straight line; (b) along the parabola^ = a*2 ; ( c ) along a polygonal
line Z1Z3Z2 where z 3 = 1 (Fig. 38).
711
638. Evaluate J z cosh z dz.
0
*(i+0
639. Evaluate J z cosh z dz.
0
Ch. 11. Functions o f Complex Variable 113

If a function / ( z ) is analytic in a simply connected domain D9


then the function

m = / m
Zo
is also analytic in D, and
F'(z) = f(z).
Indeed,
z+h
F(z + h )-F (z)
F \ z ) = lim = lim 4- f m
h^O h-+ 0 n J

z+h z+h

= lim 4 f [ f ( z ) + m ] # = / 0 0 + lim \ J r?(l) dt = f(z),


h-+ 0 " J h-+0 n J

where 7?(£) 0 as £ — z, since / is continuous at the point z. For


small h, 177(!)| < e, therefore for such h
z+h
l_
h
e
W\ I
z
*
e.

It is understood here that integration is performed along the


straight line connecting the points z and z+h. We are allowed to
proceed so, since /( z ) is analytic and, consequently, the integral is
independent o f the path of integration.
Thus, we have proved that F'{z) = / ( z ) . The function F(z) is
called the antiderivative (or primitive) for /(z ).
The same as in the case of a real variable, we can establish that
two arbitrary antiderivatives for the function / ( z ) differ by a cons­
tant term.
Hence it follows that if 0 (z ) is an antiderivative for /( z ) , then

f f(Z )d Z = 0 ( z ) - 0 ( z O)
z0

which is known as the Newton-Leibniz formula.


640. Using the Newton-Leibniz formula, compute the indicated
integrals:
2+ i i

(a) J (3z2+2z) dz; (b) J z sin zc/z„


l —i 0
114 A Collection o f Problems

Sec. 11.6. Cauchy’s Integral Formula


(See [3], Secs. 6.7 and 6.8)
We know that if f(z ) is analytic in a domain D bounded by
a piecewise smooth contour C, then there takes place Cauchy’s
integral formula

c
641. Evaluate the integral
T— f eXP (**) dz
1“ J ~~z^ 6 T ’
c
if: (a)C : | z - 2 | = 1; (b) C: |z| = 1; (c) C: | z - 6 | = 1 (Fig. 39).
642. Compute the integrals:
/ 0\ f exp (z2) dz m ... f exp (z) </z
(a) J ~z(z+T r ’ (b) J
| z —1 1=3 /2 | z + 1 1= 1/2

© © ■CD-
Fig. 39. Fig. 40.

643. Evaluate the integral


sin n:z
= j
|z - l | = l
dz.

Solution. The function f (z ) = is analytic on the circle


|z — 1 1 1 (Fig. 40). Therefore, using the formula

■' y "J 2 n i J (z-z„)"' 1 ( 2)


c
for n — 1, where C is the circle \z—1 1 = 1, we obtain
f sin nz dz ./s in 7 r z \'|
J (7 W ~ ln iW + r?) U i
|z-l| =l
.n (z + 1) cos nz —2 sin nz I ~ .n n£i
= 2ni = - 2» t = - t
(Z+1)3 |z=l
Ch. 11. Functions o f Complex Variable 115

644. Evaluate the integral I round the circle \z\ = 2


v j _ j cosh zdz
(z+1)3 (z 1)
M= 2

Hint. Construct the contours C i and C 2 incorporating, respec­


tively, the points z = —1 and z = 1 and lying inside the circle
|z| = 2. Then J .. J . = ... + J ..., and then apply
I21=2 C, C2
formulas (1) and (2).

Sec. 11.7. Series in a Complex Domain


(See [3], Secs. 6.9 and 6.10)
In Problems 645 to 651 find the radius R of convergence of the
indicated series:

645. E FT • 646- £ k\ zk. 647. £ ( - 1)* A:!


k=0 k =0 /c = 0

648. £ ( - l)fc . 649. £ ( - 1 )* 2*. 650. £ «*•*".


A:=0 V k= 0 «=0
OO

651. Y, +
«=0
In Problems 652 and 653 expand the given function in a Taylor’s,
series in the neighbourhood of z = 0:

652. 1 653. 1
(1 +z)2 ■ (1 +z) (z—2) *
654. Expand the function / ( z ) = -5 ^2z *n Powers of (z—3).
Solution. Transform the given function in the following way:
1 _ 1 _ 1 _ 1 1
5 -1 2z ” 5 i 2 ( z - 3 4 3) “ 'll -f2(z-3) IT “ 2 r . *
i I j j yz j )

Substituting in the expansion

1. 1i Z = 1 - z + z 1— . . . + ( - l ) « z"+ . . .
2
jj (z—3) for z, we get

= A - jfi <2~ 3>+n* ( * - V - ■■• •


116 A Collection o f Problems

The last series converges for

A | Z- 3 | < 1 , or | z —3| < y .

Hence, the radius of its convergence R = 11/2.


655. Expand the function /( z ) = 1/(3—2z) in powers of (z—3).
656. Determine the domain o f convergence o f the given series:
OO
1
(a) I (v ^ + ; v ^ ; (b) I 4*0 + 1)” *
n=l 71=1

657. Expand the function / ( z ) = ^ (z_ 2^ in Laurent’s series


in powers of z in the following domains: (a) 0 < |z| < 1;
(b) 1 < |z| < 2; (c) |z| > 2.
658. Expand the function

^ = (z2- l ) 2
into a Laurent’s series in the annulus 0 < |z —1 1 < 2 .
In Problems 659 and 660 expand the given functions in a Laurent’s
series in the neighbourhood of z = 0:

659.— . 660. z3 exp(l/z).

Sec. 11.8. Isolated Singularities. Residues


(See [3], Secs. 6.11-6.13)
661. Determine the character of the singular point z = 0 for the
indicated functions:
(a) /( z ) = (e2- I)/z; (b) ./(z) = l/z 4, (c) /( z ) = exp(1/z-).
662. Find all the singular points and determine their character
in the given functions:

(a) ; (b) cos — ; (c) z sin — ; (d) tanh z.


v / sinz v/ z z v/
A
663. Find the residue of the function — - at the point z = <x».
Solution. We have
A_
z —a

consequently,
Res = -A .
Ch. 11. functions o f Complex Variable 117

664. Find the residues of the function

at its singular points.


Solution. It is obvious that z — 0 and z = n/4 are the end singular
points o f the given function.
Let us find the limits of the function f(z ) at these points:
1 4
lim f(z) = lim ljm
z —> 0 z-+ 0 Z“ z —►0 z —(n/4) 71 ’

lim / (z) = oo .
Z —►7l/4

Thus, z = 0 is a removable singular point and R e s/(z ) = 0.


z= 0
Further, z = n/4 is a pole and
16 . 7l2
Res / ( z ) = lim /( z ) ( z — = lim s = sm fT
z=n!\ z-+ -n /4 ' ^ ' z-*nl& 7 I2 16

The point z = oo is also singular, it is an essential singular point.


To find the residue o f /( z ) at this point, one has to expand the func­
tion in a Laurent’s series in powers of z:
z 2(2Jfc+l)

£ (£ )'• 2 <-•>*
n=0 X / k= 0 (2* + l)! *

Multiplying the series and grouping the terms with like powers of z,
we obtain

^ (Z> = t ( 1“ 4*3T+ 4*5T_ 4i* 7 r t‘ "' ) + k^ L 1 CkZk'

R « /(z )= +

_ 4* /7I2 71® 7l10 \ _ 16 JI2

The same result will be obtained if we take advantage o f the basic


theorem on residues according to which the residue o f the function
/ ( z ) with respect to z = oo is equal to the sum o f the residues with
respect to the end singular points taken with the minus sign (see [3],
Sec. 6.13, Theorem 1).
Let us consider one more method for finding the residue of the
fu n ction /(z) at the point z = oo. The function — sin z2 may be
regarded as analytic on the z-plane, if it is supposed that it is equal
118 A Collection o f Problems

to 1 at z = 0. Therefore it is expanded into a power series in powers


o f {z — which converges in the entire z-plane:

sinz2 sin(7r/4)2 , ^ / 71 \ k
~r - t)
rtulthen
sin (ti/4)2 sin (ti/4)2
m = - + £ ckzk,
z
71 W4)2 + V(z) = —
z-
71 (ji/4)2 k=0
“T 4

where the power series on the right is convergent for all z. In fact,
ip(z) is a function analytic in the z-plane and it is expanded in a
power series convergent in this plane. Now, taking into considera­
tion Problem 663, we obtain
sin (71/4)2 sin (nr/4)8
R e s/(z ) = Res
W 4)2 (77/4)-

665. Find the residues of the indicated functions at their singular


points:
(a) /( z ) = z- sin (1 /z2) ; (b) / ( z ) = z2 exp (1 /z );

^ = ( 7 W ^ ; (d) f (z ) = ( z - 2 ) e x p ( ^ 2).

Sec. 11.9. Computing Integrals with


the Aid of Residues
(See [3], Sec. 6.14)
The basic theorem on residues can also be formulated as follows:
the integral o f / ( z ) about a contour 71, traced anticlockwise, is
equal to the sum o f residues with respect to all singular points
zi, . . . , z„, found inside r , multiplied by 2 tci:

J /( z ) dz = 2ni £ R es/(z*). (1)


r
666. Evaluate the integral

J SSi)*-
|r| = 2

e* — \
Solution. In the domain |z| < 2 the function / ( z ) = is
analytic everywhere except for the points z = 0, z = —1. Let us
Ch. It. Functions o f Complex Variable X19

find the residues of f(z ) at these points. Since z = 0 is a removable


singular point, R e s/(z ) = 0. At the point z = —1 the function
z=0
/ ( z ) has a simple pole, therefore

Res f(z) = lim ( z + l ) / ( z ) = lim —- - = \~-e~1.


Z — —1 z-K-1 z -> -1 2

r ez —1
According to (1), we obtain J dz = 2jt/(1 —
| ri =2

667. Evaluate the given integrals:

(a) / ( z - i f z - 3 ) » where r : T + T = 1;
r

<b> J ; (c) / , r : - v H / = 2x;


|z! = 5 r

(d) / 7T+2Z-3 dz' wherC T + > ’2 = K


r

668. Compute the integral

2%
Solution. We introduce the function /( z ) = ° f the com­
plex variable z. It satisfies the conditions of Theorem 1 from Sec. 6.14
[3] for m = 4 and has in the upper half-plane a pole o f the second
order at the point z = ai:
Z2 _ 1
Res f ( z ) = lim ~ ( z - a i ) 2f (z ) = lim —
z=ai z -+ a iaz z - + a laz (z + ai)2 ~~ Aai ’

Then

/ = 2txi Res / ( z ) = — .
r=o/
669. Evaluate the integrals:

x 2+ \ dx
dx; (a > 0, b > 0);
x* + l (x2+ a2) (x2+ b2)

oo
dx
( « = 1,2, . . . )•
<!+**> Fi
120 A Collection o f Problems

670. Evaluate the following integrals:

W /o t *
0
0» /& ! ? * ■
-o o

671. Compute the integral

(lx (a > 0).


xix^d1)

eiz —
Hint. Consider the function f(z) = - 7^— and the contour
J 7 z{z1+ a2)
r e, r (Fig. 41). For a small e and large R the function / ( z ) has one
pole inside the contour R. Note that / ( z ) has a singularity at the
point z = 0 on the real axis. Then pass to the limit:

lim | f(z)dz.
e —►0
R-+OC re,R

672. Compute the integral


oo

/=J exp ( —ax2) cos bx dx (a > 0, b > 0).


o
Solution. As we know,

J exp ( —x2) dx = y jr ,

therefore

| exp ( - ax2) dx = - y / --.


Ch. 11. Functions o f Complex Variable 121

Consider the function f ( z ) =


= exp(—az2) and the contour 71
having the shape of a rectangle
with sides 2 R and b/(2a) (Fig. 42).
Inside the contour/1 the function
e x p ( —tfz2) is analytic, therefore

J exp (—az2) dz = 0,
]'
i.e.
R b!2a

J exp ( —ax2) dx+ J / exp [—a(R+iy)2] dy


- r o
- r o

+ J exp a(x+ i ^-)2j dx+ Ji exp [ - a ( - /? + iy)2] dy = 0.


R b/2a

As R -►oo, the second and fourth integrals tend to zero at the ex­
pense o f the factor exp ( —aR2). Therefore, in the limit, as R -* oo,
we obtain

r r v
I exp ( —ax2) dx+ I exp ( —ax2) exp ( —ibx) exp — dx = 0.
—oo oo

Hence, separating the real part in the second term, we have


oo oo

exp | | exp (—ax2) cos bx dx = J exp ( —ax2) dx = ' y / —

or

J
o
exp ( - a x 2) cos bx d x = - -y / - exp
C hapter 12
OPERATIONAL CALCULUS
(See [3], C hapter 7)

Sec. 12.1. Transforms of Simplest Functions


(See [3], Secs. 7.1 and 7.2)
673. Using the definition, find the Laplace transforms o f the
indicated functions:
( a ) / ( 0 = e2t\ ( b ) / ( / ) = sin 3/.
674. Is it possible for the function cp(p) = 1/sin p to be the trans­
form o f some object (or original) function?
675. Using the property o f linearity and the property o f similarity
find the transforms of the following functions:
(a) / ( / ) - /+ 2; (b) / ( / ) = 2 sin 3 t+ e ~ 2‘ (/ ^ 0).
676. Is

an object function?
677. Find the transform of the function / ( / ) = cos mt cos nt.
678. Using the theorem on differentiation o f object function
L[f';p]=pL[f;p]-f(0)y
find the transforms of the following functions:
(a) / ( 0 = cos2 3 1; (b) f(t) = cos4 1.
679. Using the theorem on differentiation o f result (or image)
function (F'(p) = —tf(t))9 find the transforms o f the below func­
tions :
(a) f(t) = t2 cos t ; (b) f{t) = t sinh 3*;.
680. Using the theorem on integration of object function

o
find the transforms o f the following functions:
vt t
(a ) / ( / ) = J r sin h 3r dr\ (b ) / ( / ) = J r2 c o s r dr.
Ch. 12. Operational Calculus 123

681. Using Ihe theorem on integration of result function

At) .
— ^ j HO) dq.

find the transforms of the following functions:


/ \ sin2 / , >, sinh /
(a) — ; (b) — - ; (c) — .

682. Using the theorem on translation (shift) of result function,


find the transforms of the indicated functions:
(a) e3' sin /, (b) eH2 cos t.
683. Using the theorem on delay of object function
f ( t - to ) = e ~ P ‘o F ( p ),

find the transforms of the below functions:


(a) sin ( t-b)<To(t-b );

(b) *•'-% «(/-3), where a0(t) = { *’ *"


1 ( 0, t < 0.
684. Find the transforms of the following functions:

(a)/(0 = | 2J / ] / =s /0 + 2 )= m
f t, 0 ^ t ^ 1,
2 o; ,

(b ) / ( 0 = |sin t \ .
685. Find the transform of the given convolutions:

(a) J sin r dx\ (b) J (t—r)2 cosh r dt.


Sec. 12.2. Finding O bject Function
by Ics Transform
(See M], Sec. 7.2
6 8 6 . Find the obj^cHfinction/ ( / ) if F(/?) = 1 —cos (Up).
687. Find the object function for the result function

= p(p-D(P2+i) •
6 8 8 . Find the object function for the indicated result function:
1 _ N r., X 1
(a) F(p) = - 5—-— _ , (b) F(p)
P -(P 2+ 1) ■
124 A Collection o f Problems

689. Find the object function for the function F(p) — I/ \ / 1 +/?2.
Solution. Let us expand the result function F(p) into a Laurent’s
series in the neighbourhood o f a point at infinity:
1 3
2 * 2
F( P ) = 2 \p 4
_ ~ ( —1)* (2k)!
£ 0(k\)*2*p*+' *
By Theorem 11 (see [3], Sec. 7.2),

m = S 0(T !rft -,2* s


where J o(0 is a Bessel’s function of the zero order (see [3], Secs.
1.25 and 5.9).
690. Find the object function for the rational functions F(p),
using the equality
m
fit ) = £ Res [F(p)e”'],
k=l p=pk

where pu . •., pm are the poles of the function F(p):


1 . 2p3+ p2+ 2p + 2
(a) F(p) = (b) F(p) = p \p * + 2p + 2) *
p2+ 4p + 5 9

Sec. 12.3. Applications of O perational Calculus


(See [3], Sec. 7.3)
691. Solve the given differential equations for the indicated initial
conditions:
(a) x ' + x = x(0) = 1;
(b) x " + x = 2 cos t, x(0) = 0, Jt'(0) = —1;
(c) x " + 2 x '+ 5 x : = 3, x(0) = 1, x'(0) = 0.
692. With the aid of Duhamel’s formula, solve the following
equations:

(a) x" = — , x(0) = x'(0) = 0;

(b) y " + y = sin x, >>(0) = /(O ) = 0;


(c) y '" + y ' = 10c2*. y(0) = / ( 0 ) = ?'(Q) = 0.
Ch. 12. Operational Calculus 125

693. Solve the system


f x ' + y = 0,
x (0 )= l, # ) = -!.
I y ' + x = 0,
Solution. Let X(p) = x(t), Y(p) = y(t). Write the operator equa-
tions
( pX(p)-x(0)+Y(p)=Q,
\ X ( p ) + p Y ( p ) - y ( 0) = 0,
or
j pX( p) +Y( p) = 1,
I X(p)+pY(p) = - 1.
Solving this system with respect to X(p) and Y(p), we have

Hence,
x(t) = er, y(t) = - <?'.
694. Solve the following systems:

w {[ Ty + y,'' == Tx + e;‘, m - m - u
x" = 3 ( y —x+z),
(b) y" = x - y ,
z" ——z,
,v(0) = x'(0) = 0; y(0) = ( ) ,/ / ( ( ) ) = - 1; z(0) = 1, z'(0) = 0;

(0 I '' =
\ z' = y + z + e * , y(Ch ■ z(0) = 0.
695. Compute the below integrals:

sin x t cos t
(a) /(* ) = J ~ ~ d t ; (b )/( * ) = / dt.
Chapter 1.

8. a = 0.(1) = 1/9. 9. a + b = 0.(25) = 25/99. 10. A +B = [2, 6), AB =


= (3, 5], A \B = [2, 3]. 11. (a) -3 .1 < jc < - 2 . 9 ; (b) * « s - 7 , j c ^ 13;
(c) a: < - 3 / 2 ; (d) 0 < * < 1/2; (e) * < - 3 / 2 , * > 1 / 4 . 12. a if
0 > 0, —a > a, if a < 0. 13. (a) 6 => 0; (b) 6 0; (c) 6 < 0, (d) b > 0.
14. (a) 1; (b) 1; (c) a/2/2. 16 0. 17. 0. 18. 1/2 (see Problem 1). 19. 1/3
(see Problem 2). 22. No. It is simply unbounded. 27. 9/8; 1/6. 28. 2; —30.
29. 0 ,1 ,1 ,1 ; —1/6,4/3, 1/3,1. 30. —1, 0,1. 32. Hint. Take advantage of
the inequality ~ (A: s= 2). 34. £ = (-0 0 , 00); £, = (0, 1],

35. E =
'3 - V I 7 3 + \/l7 l
2 J’
V2 4- Ax 4- 3
3 6 , 1 , - - ; T ^ -T -c > - / W , 2/(1+**), (1 +a-2)/(1 —a*2).
* 2+4* + 5 *

42. { sin ;c, 0« =7 1 / 2 ,


(Fig. 43);
1, x ■tt/2
0, 0 7T,

<P(x) = sin 71 < a: 3:rc/2, (Fig.44).


— 1, jc > 3 ^ /2

44. (a) 1; (b) 2/3; (c) 1/2; (d) 3; (e) 4/3; (f) 0; (g) -1 /1 6 , (h) 1/144,
(i) l l x f i a i (j) 0. 45. (a) 3; (b) 2; (c)4. 46. » . 47. (a) (b) e3;
(c) 1. 48. (a) 4; (b) d In a. 49. Continuous everywhere (Fig. 45). 50. If
A — 2, then / ( jc) is continuous everywhere. If A ^ 2, then x = 1 is a
removable discontinuity (Fig. 46). 51. * = —1, x = 3 are points of dis­
continuity of the first kind (Fig. 47). 52. * = —1 is a removable discon­
tinuity (Fig. 48). 53. jc = - 1 is a point of discontinuity of the second kind.
Answers 127
128 A Collection o f Problems

2k + 1
54. x = — — n (k = 0, ± 1, . . . ) are points of discontinuity of the first kind
(Fig. 49). 55. x = 1 is a point of discontinuity of the first kind (Fig. 50).
56. x = 0 is a point of discontinuity of the first kind if a 1 (Fig. 51).
57. d ^ e / S . 58. <5<e/12. 59. 0b - d y ) /( c y - a ). 60. 3*. 61. jc. 6 2 . / ' =
= 3x2—2, / ' (0) = - 2 , / ' (2) = 10.

= - + -^ = . 64. ( |* |* 1 ) .
6 2 a/3
1
65. (a) 1 + - j L = (x * 0); (b) - — - 1 (* > 0);
3#? x2 2x V * 3x \ / x
1+ 2x2
(C) (1 -X)3(1 + *)4 ('X1 ^ 1): (d) (1 +X2)V2 ’

(e) (a2- * 2)3'2 (1*1 •« |a|);

(0 l + 2V x + 4 V x V x + V x (x ^ 0).
8 v * v 'x + V * /v /* + v * + V *
(g) - j - ^ + ---- — ( x > 0 ) ; (h) — — (* * kn , A = 0, ± 1 , . . . ) ;
3 y jc x y x smzjc
n sm x / 2&—1 . . \
(0 (cos x + x sin x)2 ; (J) &—integerj;

1 \ ~ 2 In 2
(k) - 2
=(- IY ; (I) e V '( l | e O ;

(m) "1-|-a*',"*aI" In a; (n)—p— (* * 0);


1+*2
(o) JMILL (0< |.v| < I).
y/\ —>
66. — ( x * ^ ! * , k = 0, ± 1, . 6 7 . - 2 * exp ( - *2).
cos4a: \ 2 /
68. (a) 2jc cos x 2 ; (b) sin 2* ; (c) 21a:6 cos x 7 sin2 x 7; (d) —cos x «sin (sin x ) ;
(e) —2jcsin;c2; (0 —4a:3 sin 2x4. 69. (a) l / \ / a 2—x 2 ( a > 0 ) ; (b) aj{a2+x*)m 9
(c) 1ly /(P + x 2; (d)sgncosjc, x ^ ^ -in teg e r; (e) —l/yA*2” * 2

(a > 0); (f) (3*2+ l)c?*,+*. 70. (0 < x -2 k n < n, k —integer).


1
71. (a) - - 2 H arclan *; (b) In2a 2 (a 5*0); (c) :( v>e);
x In A' In (In x)

(d) (|jc| 1); (e) -___ _ (x > —1); (f) 1/cos aX


jc4- 1 2(l + V * + l)
In3a: 2x
X ( \ x - 2 k n \ < tt/2, &—integer); (g) - ( x > 0 ); (h)
l+ \/I + ;c 2 ’
Answers 129

(i) (* > 0 ); (j) — (jc ^ 1); 00 —


2(1 -hjc) l+ x 2 *»+l
12jc5 , x sin 2x / 2 k —1 « . \
(1) (m) (* * —j — *—integerj ;
(1 + *12)2 ’ sin4 a - f cos4
k*2
(n) f T ^ arctan^ <°> 2(TT^)-

72. / '( a) =

73. / ' ( a ) = (Fig. 53).

76. (a) 2x cosh (a 2 + 1);


(b) 18 a 5 s i n h 2 a 8 c o s h a 6. 77. ( 2 A + l ) s i n h 2 ( A 2+
- fA
A + 1l )) . 78. (a) 2tanhjc/cc
2 ta n h A /c o s h 2A;

1
(b) Zxr/cosh2jc2. 79. (a) - ; (b) 1 /v T w * ; (c) V ^ , + 'V/I,t.'!2;
a cosh2 (1 + ln a) V2(l + jc2)8/1
(d) cothjc (x > 0); (e) — sinhln x (a — v,, 0); w —
(f) cosh2
a AT
(g) (sinh jc)c08h* [sinh x In sinh A+cosh x coth a]; (h) - ( 3 sinh A)/cosh4 x;
(i) 2 cosh A/sinh2 a (at*0). 80. d f(x ) = (2a+ 1) Ax; d f( \) = 3Ax; A f( l) =
= 3Ax + (Ax)2; for Ax = 0.1, d f( 1) = 0.3, A f{ \) = 0.31. 81. **(1+a)</a.
82. cosh Acdx. 83. —a sinh x dx, 84. — ( |a | < 1).

85. (a) ( - 2 + 4 ^ ^ * ; ( b ) ^ — ^ . V .U O x d x K 8 8 . e * [ l n * + J ^ 4 ^ ] dx*.


3 —3t2 33
89* ^ = 2=27 = T (1 + /)’ = 4(1^0 (* * 1}* ^ = ” Cot *’ ?" =
= - l/£ 2 sin3 /) (f k n 9 k —integer). 91. / = f, y ” = 1//"(/)• 92. / =
sm t -1
(r s* 2kn, k - integer). 93. (a) y +1 1a - 1 8 = 0;
1 - cos t 4 sin4(//2)
11^-a-f-46 = 0; (b) have no common tangent. 94. <p = arctan 2 y/2 .
130 A Collection o f Problems

95. <p = ji/4. 96. n/6. 98. (a) c = l / \ / 3 ; (b) c = A ± v ' A^ + 3 A + 3 >
V B, C; (c) c = l/V 3 . \M , B, C. 100. (a) The function is increasing on
( —oo,l/2) and decreasing on (1/2, oo); (b) Increasingon ( —<=°, 1), and
decreasing on (1, °°). 103. a/b. 104. 2. 105. 1. 106. 4. 107. 1/4.
108.1. 109. (a) * -«•; (b) e,,s; (c) a/b; (d) 3; ( e ) - 1 /3 ; (f) 2; (g) 1/3;
(h)0; (i) e~1’, (j) 1; (k) 1; (I) e~l ; (m) a° ( - 1 + ln a)- (n) 1/n;
(o) (p)n/m ; (q) 1/2; (r) 2/n; (s) 1/2. 111. tan * = * + - - + — *5+
m n J ID
2
-ftfC*6). 112.f { x ) = 1 + 2 x + x 2- ^ jc 3 + <?(jc 4). Hint. Use the expansion by
JC2 JC4
Taylor’s formula for the function ex. 113. In cos* = - ——- + o(x5).

114. sin(sinjc) = j c - y f<?(jc4). 115. -1/1 2 . 116. 0. 117. -1 /6 .

118. -1 /2 . 119. (a) 2.718281; (b) 0.017453; J c ) 2.236. Hint. Represent


the number y/5 in the form \ / 5 = 2 l + ~ and take advantage of
Taylor’s formula for the function (1 +jc)m for x = 1/4, m = 1/2; (d) In 2 =
= 0.69315, In 3 = 1.09861. 120. (a) For jc = - 1 /2 a maximum, y { - 1/2) =
= 9/4; (b) for jc = 0 a minimum, ^(0) = 0, (c) for jc = 0 a minimum,
y(0) = 0; for jc = ± 1 a maximum, ^(+ 1) = 1; (d) for x = —1 a maximum,
y (—1) = - 2 ; for jc = 1 a minimum, y( 1) = 2; (e) for jc = 37t/4 a maxi-
/3tt\ y /l . 7n . . (l7i\ y /2 . ..
mum, - 1 =*= - j e ’ ^or v = 4 a minimum> .vl y I = — 2 ~e ‘ 9
(f) for jc = 0 a maximum, ,y(0) = 1/4; (g) for jc = 1/2 a maximum, y{ 1/2) =
= 1/4; for jc = - 1 /2 a minimum, v ( - 1/2) = - 1/4; (h) for .v = 1 a maxi­
mum, v’(l) = 0; for v = 3 a minimum, >’(3) = —4; (i) for a = k n (A —
1 2tt
= 0, ±1, .. .) a maximum, v(Ajt) = ( —1)* fy ; for v —:t ^ \ 2k jj (A

= 0, ±1, . . . ) a minimum, y + 2A:rj = —- ; (j) for jc = ^ |- 2A*jt

71 \ 3 a /3 JT
(-- + 2A7ij = ; for * = - - r 2 b

(A = 0, ± 1, . . . ) a minimum, y ^ -™ -\-2k?ij 121. (a) 32; 1;


(b) 10, I; 2. 122. c/- 7 \/2 /8 . //iw/. Investigate the function v=
= - _ |a '2 - a 1 2 1 for extremum. 123. (a) 2; 0; (b) (l \-\/2 )/2 -- 1; 0.
v2
124. (a) -V = 1 is a point of inflection; the graph is convex downward on
( —0 0 , 1); on (1, 00) the graph is convex upward; (b) jc = ± l / \ / 2 are inflec­
tion points; on ( - 1 / \/ 2 , l / \ / 2 ) the graph is convex upward; on the
intervals | jc | >» 1/2 the graph is convex downward (Fig. 54). 125. (a) Inclined
asymptote: y = jc ; vertical asymptote: jc = 0 (Fig. 55); (b) horizontal
asymptote: y = 1, jc = 0 —a vertical asymptote as j c -»-+0 (Fig. 56);
(c) y = ‘0—a horizontal asymptote (see Fig. 54), (d) y = jc—an inclined
asymptote as jc —00; horizontal asymptote: >> = 0 as jc — - o o (Fig. 57).
Answers 131

Fig. 55.

126. (a) x — - —1/3—a point of minimum, >»(—1/3) = —\ / 10; * = —1,


a = 1/2 are points of inflection; >> = - 1 is a horizontal asympotte as
x —oo an d ^ = 1 is the same for x -*■+ °° (Fig. 58); (b) x = 0 is a point of
minimum; x = —1 is a vertical asymptote; y = 0 is a horizontal asymptote
as a —oo; the graph is convex upward on ( - o o , - 1 ) and downward on
( - 1 , oo) (Fig. 59). 127. (a) See Fig. 60, A (t > 1), A ( - 1 < * < 1), A
{t < - 1 ) ; (b) See Fig. 61, A (' > 0), A (' < 0). 128. (a) The sides of the
rectangle: a y / 2 and b \/2 ; (b) x = a/6; (c) PB = 12 km; (d) a = <//V3,
h = d y /2 l'\/3 . 129. The rectangle is a square with side 2.
130. — ! s*n - - . By the cosine law, the distance between the
Vw2+ v 2—luveos 8
ships (see Fig. 7) at an arbitrary instant t is r2(t) ----- (a f ut)2 f (6 f v/)2-
—2{a -\-ut) (b+ vt) cos 0. The instant of time when the ships were at a distance
a and b from the meeting place is supposed to be equal to zero. Then
/ 2 r \3/2
investigate the function r \t ) for extremum. 131. (a) p \ \ + — J ;

(t>) (g4~ a2jCl t ^ 2)3,i! ; (c) 2 V ?- 132. I = a(<—3 sin t), t? = 3 a (l-c o s *)•
132 A Collection o f Problems

Fig. 58.

Fig. 60.

Fig. 61.
A n sw ers Hi
Chapter 2.

(For the sake of brevity, the constant C is omitted).


133. ~ x 3- 1 25x4+^0x5- y x6+ - -- x7. 134.x - y x 3+ y x 3.
l-.Y
135. y —arc tan y. 136. x —cos x I sin x. 137. v -f In . 138. —x H-tan x.
I KY
139. y —tanhY. 140. x —coth.v. 141. arcsin.v | In \x I- y /\+ x * \.
142. — (2x —3),0#. 143. (a) - - arc (an (b) x —3y2+ —■x 3——x i ;
200 A/6 3 2
, v , , . a2 cP 4 4 / - 24 12 , 4 4 /—
(c) a\n U I - — ~ 2 x 2 > (d)
W) y- - x* {V/ *x “- j - x V\ /*x 5+
+-3y 'V * 3'. (e) - x +

+ 31n | * | + ~ - ( 0 j V ^ ( x + 6 ) ; (g) - - In(1 + x 4); (h) - x +


1 . 1+x
+ 2 n 1 - x ; (i) Y -f21n S ll’ P £ +2E * + E ^ (h> "COsh* +
j-Z>sinh.Y; (I) - (2x - 9)*'; (m) \ arc sin ^x j [ - j ;

(n) 4 = l n ( v /3-x+\/2H-3Ar2); (o) — ln V \/3 + V '3 x 2- 2 |; (p) X


V3 __ a/3 nL X 6
X(3e~2x+ 2e~3x). 144. —\ / \ —x2. 145. In (2^he^__146^tan x —cot x.
147. —[ (x + l ) 3l2- ( x — 1)3/2]. 148. —ln |c o sjt|. 149. 2 a rc ta n \/jc .
3* —2*
In H in t. Divide the
W . - ? 3 * r *»-«*>». «“ - s s r r n n > 3*+ 2*
numerator and denominator by 2*3* and set t = (3/2)*. 152. (a) y V * 2“ 2 +
-1 »—\/2
+ ln |x + V * 2- 2 | ; (b) — (1 + x 3)4'3; (c) ; (d) : In
4 2(1 + xs) 8 \/2 JC4+ \/2
(e) arctane*; (0 * - l n ( l + \ / e 2x + l ) ; ( g ) y l n 3*; (h) In |sinjc 1

(i) In | tanh (y /2) | ; (j) 2 arc tan e*; (k) y (arc tan x )2; (1) - y + y sinh 2y

(m) y + y sinh 2y ; (n) - 2 c o th 2 ^ ; (o) (1-3ac)2/3; (p)

X (1 + x 2)4/3 (4y2- 3); (q) x - In' (1 + e*). 153. x arc tan x - y In (1 + y2).
154. —(x -f l)e~x. 155. ac: cosh y - sinh y . 156. —y cosy + sin y .
157. V l “ * 2+ *arcsinY . 158. at sinx + cosx. 159. —V * + U + * )arctan y / x .
160. (a) — sin 2x —^—
^ ~ cos 2y; (b) - y + * " y - arc tan a:; (c) — i-X

X e - to( * * + * + y ) ; (d) - - ( 1 H - x V * 1; (e) x (—1 + ln x); (f) — X

x ( ^ — + ln x j ; (g) ( y + ~ j sinh 3 x - + y ) cosh 3x; (h )x 2c o s h x -

—2jc sinh y I 2 cosh x ; (i) In tan * —cos y In tan y . 161. (a) In | y —2 1+


134 A Collection o f Problems

-fin |* + 5 |; ( b ) ~ ln x —2 49(x —2) i4(Ar_2)2 ' 162,(a)arctan(jc 1);


(b) 1 /(1 -* ); (c) ln |x * - 2 x + 2 |. 163. (a) y In |a + 1 \ ~ In |**+1 I +

+ — a re ta n ^ ; (b) — [2 In |.v 1 2 |- l n (** + 1) +14 arc t a n A + ^ ~ ^ j .


gx 1 1 . 1 1 +.Y ( v I 2)4
164. (a) 2(| I 4 In I j - v (b) 2 In (C)
Xs 3a7 5ac 11a5 21a:4 43a3 85a" 1 , *-1
- 6 +s — 4 - +i — r + i m + 3 ln (jc-f 2)1024
(d) i - In (J: + 1)2 + 4 , arc tan ^ ;
, , 1 . ( * - l) 2
(c) — ln --------— +
6 x 2—x + 1 \ / 3 *2+.y + 1
1 2x +1 m 1 | Jt-l
+ —^ arc t a n ------- ; (f) — ln ----- arc tan jc;
\/3 3 4 *+1 2
1 2+ at\/2 + 1 1 c\/ 2
(g) ln arc tan —— . 165. 2 \ / x - 2 In (l + V * )-
4 \/2 x 2—x \ / 2 + 1 2 ^ /2 1 —x2
166. (a) -—f - ^ + 4 1 n ( l + \ /V); (b) 4 - In
l+ { ^ 4 ( l + V a ) 2 ( l —- \Z a + 2 \/a ) 3
3 . 4 V x - l
4 ; (d) * . * y * , - * +
arc tan ~ " ' =— ; (c)
2 a/7 V7 (l+ V * > * 1 + |^

+ -i- l n \x + -\/x i —l |; (e) f [ A + 2 v ^ - V * ( l + * ) _ l n ( V * + V l+ * ) ] -

Hint. Multiply the numerator and denominator by the expression \ / l + a —

( 0 b—- a aV/ — •
167. ln 11 + 2 x + 2 V l +*+•**!•

168. ^ - V x 2- 2 x + 2 + y In |* - l + V * * - 2 * + 2 |.

, v 3* sin2jt sin 4* ... 1 . .


169. (a) y ----- 4— + - 3 2 “ » <b) y ln Ito**!-
sin**
170. (a) s in * — 3 ' (fe) y ln tan
K ) |-
171. i ln 2 tan y - 1 1 . 172. (a) — arc tan ^ tan a ^ ; (b) tan (a /2 ) ;

(c) -co t(A /2 ); (d) —tan —~ ); (e) — ln [sinh (2 a - 1 ) cosh (2 a + 1 )]

^a =- - i - j . 174. (a) Since 2 a / t i •< sin a < a [0 , ji/2 ] , we have


7l/2 7l/2 7l/2
J y d x < J sin x d x J* a: */*;
Answers 135

1 i
(b) since c*' - I I .v [0, 1J, we have J erd x ^ J (1 \ x )d x . 175. (a) t/6 ;
o o
(b) 1; (c) 1; (d )In V 2 ; (e) tt/ 3; (f) 1; (g) 1; (h) - - + ln 4.

176. n/(2ab) (see Problem 172). 177. n (see Problem 156). 178.Iny
(see Problem 154). 179. / ' ( I ) - / ( ! ) -1/(0). 181. (a) sin a 2; (b) - \ / l l rr;
(c) 0. 183. 2. 184. 4.5. 185. 2Wi/3. 186. nab 187. 3ju/2/2.
188. (e2 + l)/4. 189. 6a. 190. 8a. 191. Taking 6 for the parameter, we have
x = q cos 0 = f(0 ) cos 0t y = 9 sin 6 = f(6 ) sin 0,
P
\n = J V f 2(ff)+(fm?d6.
192. 3ira/2. See Problem 191; one half of the curve is described when y changes
from 0 to 3ti/2. 193. 8a. The upper half of the cardioid is described when
(p varies from 0 to n. 194. (a) (b) y [/-f H-r | + ri/-2]; (c) tt2/2;

(d) n p t f S m . (a) 8 (b) (c) — [103' 2- l ] ; (d) - n a \


196. (a) By th^njethod ofjeefangles the required integral
/ Rj I . y _ L _
8 ,^ + 4 ’
where
Sk = (* = 0, 1,. .. ,8).

Substituting the values of £*, we obtain


15 1
/ % 2 y —-— 0.6927.

The remainder
2
^8<g2- 0.0027.
12 384
By the trapezoid method

/ «= - { / ( a 0)+ 2 /(.v 1) + . . . + 2 / ( a ,) + / ( a 8)} « 0.6941

with the same error as for the method of rectangles.


By Simpson’s method,

/ = ^ {/(*„) ■i~4f(xl) l 2 /( x 2) + 4 f( x a) + . . . 1-4f ( x 7) h f( x s)} «. 0.69315

witl\ the remainder of the quadrature formula

R - Mt 5-10 5 10 *.
4 " 4 * - 2880
m A Collection o f Problems

Thus, Simpson’s rule gives the first three correct digits for the value of the
integral. The exact value of the integral
i
/ = f = In 2 = 0.693147 . . . .
J i+x
0
(b) By the method of rectangles, / 0.8358 and by the trapezoid rule,
7 rs 0.8352 with the remainder 7?12 «= 0.004. By Simpson’s rule, the remainder
Re 10~6. Therefore, evaluating 7 by Simpson’s formula, we obtain
2oo0*6
four correct digits: 7 rs 0.83565.
197. L 3( x ) = - — (* -3 );t2. 198. (a)ji/4; (b) r — , 0 «= <x < 1; oo, « s * l;
2 1—a
(c) 7i/2\ (d) oo. 199. All integrals converge. 200. (a) 1; (b) » _ ;

(c) — (en + 1). 201. (a) Converges for p > 0, a ^ 0; for a = 0 it is convergent
if p 1; (b) convergent if either p ox q exceeds zero; (c) convergent for
m > —1, n —m > 1.

Chapter 3.

202.2. 2 0 3 .- 2 . 204.1. 205. 1. 206. cos (a +0). 20 7 .4ab. 208.1.


209.4. 210.0. 211. (a) Odd; (b) even; (c) even; (d) odd. 212. A n = b c - x \
A 12 = x 2—cxy A 13 = x 2—bx, A 21 = x 2—cxy A 22 = ac—x 2, A 23 = x 2—ax,
A 31 = x 2- b x 9 A 32 = x 2- a x t A 33 = a b - x 2; A = On/411 + 012^12 +**13^ is =
= 2x 3—x 2(a+b+c)+abc.
213. (a) 0; ( b ) - 4 . 214. (a) 72; (b) (c -o ) ( b -a ) (c -b ).
215. A = A r A2 =
rows by columns:
7 10 9
11 14 10 = 35;
9 13 9
rows by rows:
7 10 9
11 14 10 = 35;
9 13 9
columns by rows:
7 10 10
12 14 11 = 35;
8 11 9
colunins by columns:
7 10 10
12 14 11 = 35.
8 11 9
Answers 137

2 1 2\
217. rank A = 2, /I* - 2 3 2 .
1 1 1/
218. AT! = - 7 , * 2 = 5. 219. = 3/2, * 2 = -1 /2 . 220.* = 3/2, v = -1 /2 .
221. The system has no solution. 222. * = —84, y = -9 3 /2 , z -■■■■ 31/2.
223. The system has an infinitude of solutions:
7 3 1 i5 \ /° 1 -7 5 -i
B =.
3
5 -9
5 -2
8
:
i
»L 11

3
2 -1 4
5 -2
10 -2
3

18 4 5 12/ \0 3 -2 1 15 -3
'0 1 --7 5 - n

°l3I
1 3 5 -2 1 -7 5 -]
0 0 0 0 " ll 3 5 - ■2 :
^0 0 0 0 0/
0 1 -7 5 —1\
r\1 01 26 - 1 7
Hence,
= 6 -2 6 z + 17f, y = —1 + 7z —5/,
t^ere z and t arp^rbitrary numbers.
224i-fank^C= 3, rank 5 = 3. 225. (a) 5 /\/2 ; (b) jc = \ / 2 , y = 1, z = - 1 ;
(c) 1/2. 226. 3 ,- v /6 ; V l l; 2. 227. (a) 5/21; (b) l / \ / 2 ; (c) l / \ / 2 .
228. It cannot, since in this case cos2 a + c o s2 £ + co s2y cos2 a+ c o s2/f =
= 5/4 - 1, which is impossible. 229. x — y = z = \^3 . 230. \a —b\ = 2 2 .
/f/rtf. a±b| = V |a |2+ |b|2±2(a, b). 231. |a+ b| = 20. 232. co = n /4.
(a, a) / |a|
233. A = " ( ^ = " " ( w ) * 234‘ * = 2/5, ;y = * = 4/5. 235. ( 1 ,- 2 ) .
///rtf. This problem is equivalent to dividing the line segment AB into two equal
parts. 236. * = 1/2, y = —5/4. 237. Let M x = (zl9 Zg) and M 2 = (flf z2) be
the division points (Fig. 62). We determine the
numbers p and A(see [2], Sec. 7) for the point M x:
. . . , IAMI 2
=
Therefore

, = ( , 4 ) .
Analogously,
*i = 3, - 1/3 (/« = 2/3, A = 1/3),
M s = (3, 1/3). Fig. 62.
m A Collection o f Problems

238. Of the three given points, only A/., lies on the indicated line. 239. v =
2 13
= —y * + y • The P0^11 ^ 2 - (2, 3) lies on the given line, therefore y —3 —
2
= —y (x —2). If for the point lying on the given line we take another point, for
13
instance M ti ^ (0, 13/3), then the equation takes the form: y-~ -■
~ 3 V-
240. (a) ~ x - 2 y I 4 0; (b) -2.v I ,i< 1-3 - 0. 241. 2a— 3 v 0, 3.v I 2i' -- 0.
4 A" V
242. (a) —y —-— -p ---- t-- 0; (b) x I y 1 ' 0; (c) - I
-\/29 \ 2 9 \ / 29 V 2 V 2 "\/2 \ / 5 a /5
1 2 -1+ 4-2-51 _ V 5 . , U W _ I - 2 - 1 - 8 - 2 - 1 )
- ^ = 0.243. (a)rf = z-------------------' — ■ >(b) “ ",—
\ / 2 2+ 42 2\/6 8
J3_
244. ( * - l ) - 2 ( > - 2 ) + 3(z+3) = 0.
\/68 V2 V 2'
245. —4x—y + 2z+3 = 0. 246. (a) A ( x - \ ) + B ( y - \ ) - 2 ( A + 2 B ) ( z - \ ) = 0,
where A, B are arbitrary numbers, not all zero; (b) 2 ( jc - l) + 4 ( y - l) +
+{z —1) = 0. 247. Cases (a), (b), and (c) define parallel planes. In Case (c) we
have even two merged planes. In Case (d) the planes are not parallel.
248. = 0; ( b ) - A ~ - > > + | z - - = 0. 249.(a)rf = - X

X12-1 —3-2+6-1 —7| (b) rf = i- 1 2 - l + 1 . 2 - 2 - l - l | = -


250. cos (p — 59/63. 251. x 2-hy2+z2 = 144/29. Hint. The radius of the sphere is
the distance from the origin to the given plane (d = 1 2 /^2 9 ). 252.
3 6

+ = 1. 253. —l(x —2) + (y +1) + 5(z - 1 ) = 0. Hint. From the conditions


- 6 /5
of the orthogonality of the planes kfind the ratios A /C and B /C , where A, B, C
are the coefficients of the required plane. 254. (a) a = 7t/3, (3 = n/4, y — n /3 ;
(b) a = n/6, = ji/3, y = 0. 255. (a) 5/3; (b) 3/14. Hint. Take a point on one
of the planes and find its distance to the other. 256. (2, —1,0); ( 4 /3 ,0 ,- 1 /3);
(0, 2, - 1 ) . 257. (a) A XD2 = A 2DX; (b) A 1 = Dx = 0, A 2 = D2 = 0.
258 . £ z l = ^ = i + i . 259. x = \ +2t, y = —\+ t, z = —3 + 5t. 260. (a)

— y f — = -y ; (b) . //inf. In Case (a) solve the system


with respect to x and y, and in Case (b) with respect to z and y. 261. cos <p =
1 -1 -1 -1 + V 2 V 2 1 n ... , , „. _ ,
= ---------- y — — — = y , 99 = y . 262. / = 3. Hint. Pass to parametric
representation of the straight lines. Supposing that the lines intersect at some
point, we obtain the system
2t^—ltx — 5,
3/0+4/i = - l ,
4*0-2/! = 6,

from which we find /, /0, tj. 263. y ! = . /fm/. The vector


(2, —4, 1) is collinear with the vector a = (au a2, tf3) lying on the straight
line. 264. (a) The vectors a and b are indented in an opposite way as compared
Answers 139

with the coordinate system; (b) the vectors a and b arc oriented in the
same way as the coordinate system (the determinant formed from the coordi-

nates of the vectors 1 2I 1


3 71 °>
265. |a x b | = 21. 266. Yes
((aXb) == 0). 267. The vectors a and b must be collinear. 269. sin q> ~
|aX b| 5 \/M 270. S 2. 271. (a) Yes (coplanar);
la Mb I 21 ‘ || 3 ^
(b) no. 273. (a) Linearly dependent (see Problem 224, matrix li); (b) linearly
independent, the rank of the matrix made up from the coordinate of the vec­
tors is equal to three. 275. A = 15.

(C), the product BA has no sense.

H i 3= (b>(i i)= <c)(i 0


' 1 -1
279. (a) A 1 -3 8 41 - 3 4
t 27 - 2 9 24;
_x _ I cos a sin a\ _
(c) A — \-s in a cos a / * -G ::
1 1 1 \
2 2 2 '
1 1 1
282. (1) A
2 "2 2
7/
1 1 1
'2 2 2 /

283. (a) Operator A is a linear one. Its matrix has the form A =
(0 1 1\
= I2 0 1). Hint. The column of A represent the coordinates of the
\3 - 1 1/
images of the basis vectors. For example, e1 = (1, 0, 0). Ae 1 = (0, 2, 3);
(b) operator A is not a linear one:
/4(x+y) = (*1+.>>!, x 2+ y2 + l, *3 +^3 + 1) 5* A x + A y =
= Cx'i+.Vi* x 2-\-y2+ 2f JT3-l-J'3 + 2).
(2 -1 1 6\ j / - 6 11 5\
284. BA~l = 1 -7 4 L 285. BA~l = y I —12 13 101-
\2 - 1 0 / \ 6 - 5 -5/
140 A Cotteciiori o f Problems

286. (a)

Hint. In the old basis Ai1 = i1+ 3i2+ 2i3+ i4. This can be written as Ai1 =
— il + 2i3+ I4, therefore the first column of the new matrix will consist of the
following elements: 1, 2, 3, 1. Then we also transform A i3, Ax1, Ai4.
-2 0 1 0\
1 -4 -8 -7 1
(b)
1 4 6 4 1*
1 3 4 7/
Hint. The transformation of A i1 to the necessary form leads us to the solu­
tion of the system:
A i1 = i1+ 3i2+ 2i3+ i4 =
= ai1+0O1+ i2) +y(i1+ i2+ i3) + <5(P +12+13+14).
Then i4(i1+ l 2), v4(il + i 2+ I 3), ^4(il + I 2 + i3+ i 4) are transformed in a similar way.
287. j - 1 ^ . Hint. We have a1 = il + 2 i2, a2 = - i 1+ i2, b1 = i1- 2 i 2,
b2 = 3 ^ —i2, where (il, i2) is the original basis of the space. Express a1, and a2 in
terms of b \ b2: a1 = —~ b1+ b2, a2 = —- b1- b2. Then find Abl, Ab2
expressed in terms of b1, b2.
This problem can also be solved in the matrix form. Let a = (a1, a2), b —
= (b1, b2). T is the transformation matrix from basis b to basis a, the columns
of T consisting of the coordinates of the vectors a1, a2 in the basis b1, b2. Then
we can write in the matrix form
* = bT, (1)
where in the present case

Now let the linear transformation (p in basis a be given by matrix A and in basis
b by matrix B:
cp(a) = aA, q)(b) = bB, (2)
where the columns of matrices contain the coordinates of the images of the
basis vectors in the corresponding basis, <p(a) = (<p(a1), <p{a2)), (p{b) =
= (<p(bx), <p(b2)). It is obvious that
(p(a) = cp(b)T. (3)
Therefore, from (2) and (1) we have
(p{b) T = bBT and q>{a) - bTA,
whence, by virtue of (3), we have bBT = bTA.
Answers 141

Thus, B T = TA,
B = T A T -1. (4)
Let us now find matrix B by formula (4). It is easy to compute that
16
/ 7
3
r -i
T~ i =
13 19 1
B = TAT~X

\ 3 3,
71 98 \

288. B = T A T -1 =
15
B1
32
15
41I
15/
289. (a) Orthogonal, (b) and (c) not orthogonal.
290. The vectors e1, e2, e3 form an orthogonal basis in R 3l since the rank of
the matrix constructed from the coordinates of the vectors is equal to three
(i.e. e1, e2, e3 are linearly independent) and the vectors are pairwise orthogonal;
1 1 1 3
X = i4e - I 4 e -
291. z = 1 __L M t /_L _ ± _1
\ 2 ’ 2 ’ 2 ’ 2}’ \2 ’ 2 ’ 2 ’ 2 )'
292. (a) The basis is oriented in an opposite way: (b) the basis is oriented in
the same way as i1,12, i3 (A = 1).
1 , V3,
293. X i : . V3 ,
2 2- x 2, x 2 2 x1+ -^-x2, i.e. the passage (x [,x 2) to
(jci, x 2) is realized with the aid of the rows of matrix A*.
294. The passage from the coordinates (jcj, x 2) to the coordinates (x[, x 2) in
the new basis is realized with the aid of the rows of the matrix (A*)-1, and the
passage from the coordinates (x[, x 2) to (x lt x 2) with the aid of the rows of
matrix A*. We have

''•-(! ?)• « * > - - ( " ! -? )•


therefore
{ * i = *i + 2x2, i x[ = —x x+ 2x2i
x 2 - x [ + x 2, [ x 2 = x L- x 2.
295. (a) No; (b) no; (c) yes; (d) no; (e) no; (f) no, if the given line does not
pass through the origin; (g) yes; (h) yes.
296. The entire plane; the vectors lying on any straight line passing through
the origin; the origin.
297. The set of vectors lying on the straight line x 2 = — (k ^ 0); x x ■ 0
for k = 0.
298. (a) The dimension is equal to 3 (the rank of the matrix formed from the
coordinates of the vectors is equal to 3). A basis is formed, for instance, by the
vectors a1, a2, a4; (b) the dimension is equal to 2. A basis is formed by any two
vectors of the system.
142 A Collection o f Problems

299. Subspace V consists of the vectors v = (* !,* 2, * 3, *4) for which (v, a1) =
= (v, a 2) = 0, i.e. the coordinates of the vectors v satisfy the condition *4 = x lf
2x ! + x 2+ x 3 = 0. The vector a = (yly y 2, y3, y j is orthogonal to all the vectors
v £ V therefore its coordinates satisfy the condition
( y i+ y i- 2 y 3)x 1+ (y2- y 3)x 2 = 0, V*!, x 2.
Hence, y 2 = y 3) y i+ y ^ —2y3 = 0. For the numbers a and ft we obtain the
system a + 2£ = y l9 ft = y 2, (i = y 3, —a = y4. This system is solvable for the
indicated y l9 y 2, y 3i y4, namely a = - y 4, = y 2 = y 3 (the equality a + 2 ft = y x
is fulfilled automatically).
/ —36 - 3 7 —15\
301 302. A * {f) = I 30 30 14 .
\ 26 27 9/
303. (a) Aj = 2. /?//!/. Investigate for extremum the quadratic form u = * 2+
+ y2+ 2xy in the unit circle * 2+ y 2 = 1; (b) Ax = 3.
304. (a) The form is indeterminate by sign, since A 2 — —3 < 0; (b) the
form is indeterminate by sign (z12 = —1 < 0); (c) the form is strictly positive
(A t = 2 > 0, A2 = 1 > 0, 4 , = 2 ^ 0).

305. (a) Ax = — [a n +a22 + \/4 tf22 + (tfn —tf22)2]

= ^ [ l - l + V M + 4 ] = Vs,

A2 — [° n +^22 —\ / 4 a 22+ ( ^ n -^22)“] —

27+A/725 _ . _ 2 7 -V 7 2 5 n
the form is of hyperbolic type; (b) Ax - — > u, a2—------ - ------ > u
the form is of elliptic type; (c) Aj = 4 > 0, A2 = 0; the form is of the parabolic
type.
306. (a) The characteristic equation has the form
-A 2 2
2 3 —A - 1 - 0.
2 - 1 3 —A
The roots of the equation: Ax = A2 = 4, A3 = —2. The canonical form of the
form: 4 £ ?+ 4 £ l-2 |§ ; (b) 8£f + 8££ + 5£§ is the canonical form of the form.
307. (a) The characteristic equation is
6—A - 2 2
- 2 5—A 0 (6 —A) (5 - A) (7 —A) —4(5 —A) —4(7 —A) - 0,
2 0 7 —A
or
(6 —A) (5 —A) (7 —A) —8(6 —A) - 0,
has the roots Ax = 9, A2 = 6, A3 = 3. The eigenvector x1 is found from the sys­
tem
- 3 * ! - 2 * 2+ 2 * 3 = 0, j
'*’2x1 4*2 = 0, 1
3*i —2*8 = 0. I
Answers 143

Hence, x z = x u - 2x2 = x v The vector y1 = —— , x^j is a solution of the


system. Normalizing this vector, we obtain
xi =JL = ( l - 1 1)
Iy11 V3 3 ’ 3T
Analogously, we obtain
A 1\ X3 = ( l A _L\
I 3 ’ 3 ’ 3 l’ \3 ’ 3 ’ 3 /'
The canonical form of the form: 9 |f + 6 ll + 3£f. Orthogonal transformation:
__ 2 1 2 1 2 2
X\ 3 3 *2 + ^ 3» *2 = ~ y ^1 + y £2 + y ^3»

■*3 “ l i + x- £ 2 “ ^ ^

(b) 18{f + 18II + 9fJ; x ^ - ^ y {2 + y ^3»


2 1 2 1 2 2
■*2 — y *1 ~2~^2+ y *3 = y £i + y £2 + y £3 -
308. (a) A C —B2 = 9 > 0 —a curve of the elliptic type; 3(jc-1)2 +
+ 3 (y -2 )2 = 12; I = x —1, 7) = y - 2 ; 3l2+ 3i f = 12—a circle of radius 2.
(b) A C —B2 = 6 > 0—a curve of the elliptic type; 3£2 +2?72 = 6—an ellipse
with the semiaxes a = \ / 2 , b = \/3 .
(c) A C —B'1 = —2 < 0—a curve of the hyperbolic type; I 2- 2 rj2 = 2—a
hyperbola with the semiaxes a — b = 1.
(d) A C - B 1 = 9 > 0—a curve of the elliptic type; 3 |2 \-3rf — 0—the point
(0, 0).
(e) A C —B'1 - 2 ■< 0—a curve of the hyperbolic type; I 2- 2 rf1 = 0—a pair
of intersecting lines £ —\/2rj = 0, £+V2?7 = 0.
(f) A C —B'1 = 0—a curve of the parabolic type; 4 |- 3 ^ 2 = 0—a parabola
with the axis of symmetry f .
(g) A C —B2 = 6 > 0—a curve of the elliptic type; 3£2+ 2?72 = —1—an imag­
inary ellipse.
309. (a) A C - B 2 = - 1 6 < 0—a curve of the hyperbolic type; = 8,
A2 = —2;

X-
V2
1 A & >■ - - 7-(£+»/);
V 2

8£*-2jj*-A -(*->»)—^ ( f+i»)-13 = 0


V2 V2
is the equation of the curve in the system (1,77). This equation can be written as
follows:
1 \* / 3 \2
/
144 A Collection o f Problems

The translation

u V

reduces the equation to the form

This is a hyperbola (Fig. 63) with


the transverse (real) axis u. The ge­
neral transformation of the coordi­
nates has the form

(b) A C - B 2 = 576 > 0—a curve


of the elliptic type;
Fig. 63. Xx = 32, A2 = 1 8 , B < 0;

V2

jp = — 7=(f+v);
V2
64 64
32£2+ 18??2+ —^ (£-??)+ - —(!+ ??)-224 = 0,
V2 v2
3 2 (|+ \/2 )* + 187?2 = 288;

<
-£±5a ’ + T ? = 1; — «++i .-w
<y2
— + — = 1—an ellipse with the semiaxes a = 3, b = 4 (Fig. 64);

at = — ( w - v - \ / 2 ) , y = — G1+ U - V 2 ).
y2 v2
(c) A C - B 2 = 0—a curve of the parabolic type; At = 25, A2 = 0, 1* < 0;

- t )=
* = y (3£-4*j),

y = - y (4 | + 3>?);

25i2- y ( 3 i '- 4 7 ? ) - ~ ( 4 f + 3>?)-50 = 0, ( f - 2 ) 2 - 2(»y { 3); « ={ -2 ,


Answers 145

Fig. 65.

v = 77+ 3; u2 = 2v—a parabola with the axis of symmetry v (Fig. 65); x =


= y (3w-4v + 18), y = - y (4w 4-3v-l).
310. A C —IP = 0—a curve of the parabolic type; solving simultaneously the
equations of the straight line and the curve, we obtain the equation
x \ 2 - k Y + 6 x + \ = 0.
The discriminant of this equation has the form
9 —(2 —k)2 = (1 + k) (5 —k) 0k * 2).
(a) Therefore for k = —1 and k = 5 the straight line has one common point
with the curve. For k = 2 the straight line y = 2x and our curve also have one
common point; (b) - 1 < k < 5, k 9* 2; (c) k < - 1 , k > 5.
3 1 1 . k = - 3 , k = -1 /3 .
3 1 2 . x 2+ 2 xy+ y2—x —3y = 0 (parabola, A C —B2 = 0).
3 1 3 . (a)(Ar + l)2+ (>;+2)2+ 22 = 9 ;* + l = l y + 2 = 77, z = C ;l24-*?24-f2 =
= 9—the surface of a ball of radius 3.
| 2 ^2 £2 _
(b) -^-4--^- + — = 1—an ellipsoid with the semiaxes, a = 2, £ = \/2 ,
c = 2.
£2 t?2 £2
(c) 1—a hyperboloid of one sheet with the semiaxes a = 2,

6 = \/2 , c = 2.
(d) £24-2?y2 = 2{—an elliptic paraboloid (p = 1, q = 1/2).
£2 £2
(e) — -ry2— = 1—a hyperboloid of two sheets with the semiaxes
<7=2, 6 = 1 , c = 2.
|2 „2
(f) 4 " + 2 = *—an cylinder (the equation contains no Q.
314. (a) The characteristic equation has the form
11 - A 8 2
8 5 —A - 1 0 —A34- 18A24- 81A—1458
2 -10 2 - A
= A2( 18 —A) 4- 81 (A—18) = (A2—81) (18 —A) = 0.
146 A Collection o f Problems

Its roots: Aj = 18, A2 = 9, A3 = - 9 . Find the eigenvector from the system

(11 —A1)jf1+ 8jt2+ 2*3 = 0,


8*i + (5 Aj)*2— 10*3 = 0,
2*x- 10*2 + (2-A1)*3 = 0,
7*i+ 8* 2+ 2*3 = 0 ,
8*1 —13*2—10*3=0,
2*i -10*2 -16*3 = 0.
The rank of the matrix formed from the coefficients of the system is equal to two
(all the three eigenvalues are different). Therefore we solve the system of two
equations (in this case of any two):
- 7 * i + 8*. = -2 * 3 ,
8*!-13*2 = 10*3, *1 = “ 2*3, *2 = -2*g.
The vector v = ( - 2 * 3, - 2 * 3, * 3) is a solution of the system; normalizing it, we
get the eigenvector
X1 = (2/3, 2/3, -1 /3 ).
We find in a similar way:
x2 = (2/3, -1 /3 , 2/3), x3 = (-1 /3 , 2/3, 2/3).
The linear orthogonal transformation

1 2 2
*3 =
reduces the quadratic form to the form
*i£? + A2l i + A3| 2 = 18!2 + 9 ! § -9 !2.
The equation of the surface relative to £3 takes the form
18|f + 9£| —9 || + 2 li + 2 |2+ 213+1 = 0.

—18m2 —9u\ + 9m| = 17/18


is a hyperboloid of two sheets.
(b) The characteristic equation is A(A—4) (A—2) —8(4 —A) = 0, or
(4—A)2 (A+2) = 0 (we expand the determinant in terms of the elements of
the first column). The eigenvalues: Ax = A2 = 4, A2 = —2; the eigenvectors

X2 =
\ V 30

X'
Answers 147

Orthogonal transformation:
- 1 t 2 t 2 t
Xl V 5 l a/30 * V 6 h ’

1 1
X3 — —£iH--------~zzz^2 ~\—

-
5 V 30 \/6
Canonical equation of the surface:
—2wf —2w| + w§ = 1
is a hyperboloid of revolution of two sheets.
315. Ellipse = 1 with the semiaxes a = 3, b = \ / 3 in the plane
* = 2. Its vertices have the following coordinates in space:
(2,3,0), (2, - 3 ,0 ) , ( 2 .0 .V 3 ) , (2 ,0 , - y / i ) .

316, = 1.
16 16
y l + zt = 2y—z,
317. 7. (a) |
x = 0,
—the equation of the projection on the >z-plane. This is the equation of a circle.
' x * -2 * z + 5z*-4x = 0,
(b)
y= 0,
—the equation of the projection on the xz-plane. This is the equation of an
ellipse ( A C - B 2 = 4 > 0).
xt + 4xy+ 5y2—x = 0,
| x*
(c)
z = 0,
—the equation of the projection on the xy-plane. This is also an ellipse.

318. Parabola: 1-)* = a ( z + - - ) .

319. z = c. Hint. Regard the equation of the surface as an implicit one:

F ( x ,y ,z ) = - J + - J - £ T -1 = 0-

Then the equation of the tangent plane at the point (jc0, y 0, ^o) has the form

= °-
321. (a) x 2+ y2 = 2z—a paraboloid of revolution or an elliptic paraboloid,
X2-f y2 22
(b) — —— + = 1 - an ellipsoid of revolution.

322. (a) (3, 4, - 2), (6, - 2, 2). Hint. Pass to the parametric equations of the
straight line.
(b) The straight line and the surface have no points in common.
148 A Collection o f Problems

324. 9X2- 1 6 Y 2- 1 6 Z 2-90X + 225 = 0. Bint. By virtue of symmetry, it is


clear that the directrix is a circle obtained in the section of the sphere by the
plane x = a. The value of a is found as the abscissa of the point of tangency of
the straight line passing through the point S and touching the great circle
x 2+ y2 = 9 in the *z-plane.

Chapter 4.

325. (a) x z -\-4y2 1—the interior of the ellipse with the semiaxes a = 1,
b — 1/2 including its boundary (Fig. 66).
x 2 y2
(b) — 1—the region between the branches of the hyperbola with
the semiaxes a = 3, b = 2 including the branches of the hyperbola (Fig. 67).
(c) y 2 => 4x—the exterior of the parabola, including the curve itself (Fig. 68).
(d) The entire plane except the origin (0, 0).
(e) x + y > 0—the half-plane above the straight line y = —x (Fig. 69).

Fig. 68.

(0 \y!x \ ' ^ 0 . The part of the plane adjoining to the a-axis between
the straight lines y = ± x , excepting the origin (Fig. 70).
x 2 y 2 z2
326. (a) ^2+ ^2 +^2 < 1—the part of space found inside an ellipsoid with
the semiaxes a, b, c including the surface of the ellipsoid (Fig. 71).
X 2 V“ Z"
(b) 2 + To—cz, < 1—the part of space found inside a hyperboloid of
one sheet, including its surface (Fig. 72).
(c) x 2+ y2 < 2z—the part of space found inside a paraboloid of revolution
(Fig. 73).
Answers 149

z,i

Fig. 75.

X z V“
(d) —z-----9 —-T9 ^ 1—the part of space found outside a hyperboloid of
cr c r b
two sheets, including its surface (Fig. 74).
-(e) | jc 1=ss l,\ y \ ^ 1, \z\ 1—the interior of the cube centred at the origin
with edge equal to 2 including its faces. This cube is bounded by the planes
x = ± \ , y = ± l , z = ± l (Fig. 75).
150 A Collection o f Problems

327. / ( I , 0) = 1; / ( l , 1) = 2; /(2 , 1) = 9/2.


328. /(a*, jO = (x2—y 2)/%. Hint. Introduce new variables: u = x+ 2 y, v =
= x —2y.
x2 y2
3 2 9 . (a) = 1 —c—ellipses (c < 1); for c — 1—the origin; for
or b
c > 1—imaginary ellipses which means that the plane u — c does not intersect
the graph of the function;
( b) y = cx2—parabolas with the jy-axis as the axis of symmetry. For c > 0
the parabolas are found in the upper half-plane, and for c < 0—in the lower
half-plane. For c = 0 we obtain the jc-axis.
330. e = \/(1 - 2)2+(0 -1)'- + (I-0 )2 = V3.

331. The point M n = (0,1); o (M \ M») = \ / ° >


k -*■ oo.
332. All the points of the set E 1 = {|*| < 1, \y\ < 1} c E are interior.
333. (a) Yes, E is the interior of the square bounded by the straight lines
± y = ± x -f 1; (b) no, E is the interior of a hyperboloid of two sheets. Therefore
it is impossible to connect two points found in the upper and lower sheets of the
hyperboloid by a continuous curve belonging to E\ (c) no.
334. (a) 2; (b) does not exist; consider the approaches to the point (0,0)
x = y; x 0, y = 0.
335. c = 0. The limit of the function \ f 1—x2—4y2 is equal to zero as the
point (*, y) tends to the boundary of the ellipse x 2+ 4y2 = 1.
336. (a) No; (b) the limit of the function in the direction of the vector
= (wi» ^2) is equal to ; therefore the function will be continuous
W j -f- CO2
at (0,0) only in the direction of the vectors co = (1, 0) and co(0,1), i.e. in the
directions of the coordinate axes. Hence, this function is continuous at (0, 0)
with respect to the variables x and y separately, and is not continuous with
respect to both of them simultaneously.
337. Hint. The function u = 1 - x 2—y 2 is continuous on the entire plane.
338. u'x = 3x2, i/y = 2y—2x, du = 3x2d x + 2 (y —x) dy.
339. ux = 2 x y \ u'y = 3x 2y \ du = 2xy3dx + 3x Y dy.

340. ux = - , ufy = . ,
V x 2+ y2 \ / x 2+ y2( x + \ / x 2+ y2)

du =
v * 2+ y2 V x 2+ y2( x + y /x 2+ y2)
-y —y d x + x dy
341. (a) u'x = du =
x 24-y2 9 x 2+ y2

{ b )u ’x = y + — , u', = x - — , du = ( y + ^ d x + x { l - y ^ d y \

(c) u'x = yx v~1i x u'y = x* In x, du — yxv~x dx +x* In x d y;


(d) ux cosh (x +y), m' = cosh (x +^), du = («dx+ d y) cosh (x + y) ;
(e) m' = sinh (x2y + sinh y) «2xy, m' = (x2+cosh y) sinh (x2y + sinh y),
du = [2xy dx -I- (x2+ cosh y) dy\ sinh (x2y + sinh y).
342. (a )z l = r; (b) A = 4rcp-\.
Answers 151

_ __ , s 0w 0 jc du dy et +r 1
343. (a) — = --------- 4- ------- = '— T.------ 4-— ~ -------=
0/ 0a: dt dy dt 2 yx+ y 2 y j<+yt
^ r+t~' . 8“ 1___ C,+T_ **+T ■ (b )^ ,
2 \ / e t+T + \ n t ’ 0 r 2 \/x + y 2 \ / e t+r + \n t dt
= -^ sin (/+ r)+ A -co s ( t - i ) = -sin (f-r )sin (/+t)-hcos (r-t-r)X
du
Xcos (/ —r) = cos 2/, -z.— = - y sin ( t \ r ) - x cos ( / - t ) = -c o s 2r.
OT
344. (a) grad // = {2, 1} (Fig. 76); (b) grad // - {4, - 6 ) (Fig. 77).

V3 + 1 ,,, 0m , , x . /-z
= — 4- ; (b) gn = (grad u, n) = 2 0 -
346. gradw = {4, -6 } . The unit vector of this direction n0
= 8«_ = 4 - L - 6 . " 3 26 2 \ / l 3 . We may write
\V n V l3J 0n, V'13 V l3 \/l3
at once that — - = Igrad m| = 2 \/l 3 - This is the maximum directional
on0
derivative.
347. (a) Let a and (3 be angles formed by the gradient of the function with
the x- and .y-axes, respectively, cos a = ~ rz~ ^T E \t = > cos P =
|grad«(P)| ~ 2 ’
u'y{P)
71/6, 0 = 71/3; (b) a = n /i, 0 = rt/6.
Igrad u(P) |
348. (a) «" = 2 ,
-1 „ - 2 jc
(x2+ y)2 ’ (x * + y f ’ ** (x2+ y)2
<Pu = [2/ y —x 2) d x -—4x dx d y—dy2\/(x2+ y)2.
-r xy
(b) u" =
(2x y + y 2)3>- ’ x" (2x y + y 2)3'- ’ 1 (2x y + y - frz ’
d2« = —( y d x —x d y f/( 2x y + y 2)312.
, . 02« 02/ . 02« d2f &u = d2/
= ab
55,1 (a) 0JT2 - 0 ? “ ’ 8x dy d£ drj ’ dy2 drf *
d2u = a2f p dxr 4- 2abf"]dx dy 4- b2f''z dy2;
152 A Collection o f Problems

02W 02/ „ 02/ 02/ 02w 02/ 02/


W 0a:2 0 !2 ^ d^drj + 0*?2 ’ 0jc 0y ~ 0£2 d rf ’
02m 02/ 02/ 0y
0y2 ~ 0£2 0f 0^ + 0^2 *

rf2“ = I «WS+ 2 g f ^ ( d x ^ d y - ) + ^ O d x - -d y )- .

x ac—1 v —2 //—5
352. (a) // —5 — 2(a —1) I 4(y -2), - - = - - - = - - ;

„x « .v ac—2 y-!-l m—1


(b) / / - l — 2(ac —2)-f-2(yH i), -2—= 2 = ~~y •

353. (a) by Taylor’s formula,

Au = u(l +h, 2 + k ) —u(\, 2) = </« + -- rf2« = 4h ~ 3k + h - - k 2+ kh

(the derivatives of order higher than 2 are equal to zero);


(b) An = 2h + k -\~/t2-|- 2/i/c i h2k.

354. y + x y + ~ (3Ac2y - y 3). Remark. It is possible to take advantage of


one-dimensional Taylor’s formulas for the functions
X 2 X '2 V2
e* = l + * + y !--+ yj-+ siny = y - y j - + . . . .

2 , [ ( * - l) + ( y + l)] , [(ac~ l)2+ 2(ac—1)(y +1) + (y + 1)2]


J3 3 . i + - + 2]

[(a c —1)3+ 3(ac —l)2 (y + 1) + 3(ac —1) (y + l)2+ (y + 1)3


+ 3!"
- i (*+?) (x + y f (a c + y)3
“ + 1! + 2! + 3! ‘
356. (a) 0 = 1/2; (b) 302+ 20 = 2, 0 = ( V ? - l ) / 3 .
357. (2, 0)—a stationary point; z^ = 2, z'£ = 4, z" = 0; on = z"2 (2, 0),
#22 = (2, 0), ~ ^ (2» 0)» #h#22 ^12 = 8 > 0, fljj = 2 > 0, hence,
at the point (2, 0) the function has a minimum, zmin = 0.
358. Stationary point (2, 0); #n#22“ #i2 = —8, no extremum.
359. Stationary point (0, 0); an = 0, a22 = —4, o12 = 4, #n # 22_ #i2 =
= —16 < 0, no extremum.
360. Stationary points: (0,0), ( ± y /2 , op\/2); z"M= 1 2 ac2 - 4 , z '2 = 12y2- 4 ,
K: = 4; < .( ± V 2 , + V 2 ) = 20, r ”( ± V 2 , t V 2 ) = 20,
= 396 >■ 0. At the points ( \/2 , —\/2 ) , ( - \ / 2 , \ / 2 )—a local minimum. At
the point (0, 0) aix = —4, a22 = —4, a 12 = 4, #n#22- #i2 = 0. The question
on extremum remains undecided. Investigating the increment of the function
on the straight lines y = 0 and y = a% we make sure that there is no extremum
at the point (0, 0).
361. umiIX = —4/3 for ac = - 2 /3 , y = -1 /3 . z = 1.
362. (a) The function has no greatest value; sup z = 2. The function is
discontinuous.
Answers 153

(b) Has. The domain of definition |* |= s s l,|y l* s s lis closed and the func­
tion z is continuous on that domain, therefore it has the greatest value; (0, ± 1)—
a stationary point; z(0, ±1) = ± 1 /2 ; at the boundary of the square at the
points x = ± 1 , y = \ / 3 - l the function reaches its greatest value equal to
(1 + V3)/4.

«3|*
y2

©
363. F {x,y) = ^ ~ -

!l
n =

1!
dy n b2x d2y b*
dx n a2y 9 d x2 " a2y l
0z _ z s in x - c o s y 0z * sin y —cos z
364.
0* c o s * —y s i n z ’ dy c o s * —y sin z*
du dipjDjfp, ip) dv dip I D((pt ip)
0* dv/ D(u, v) 9 dx du I D(u, v) ’
du _ 0<p/£>(<p, ip) 0V _ 0<p //>(<?, VO
0y dv I D(u, v) 9 0y 0// / D(//, v) ’

where = j <Pu <Pv


D(u, v) Iy>n V>v
0Z C . dz c dz _ dv
367. —•— sin v. — cos V. Hint.
0* u Vy u 0* C dx

The derivative is found from the first two equations by regarding w, v


as implicit functions of * and y.

368. (a) * + 6 a/ 3 z - 3 7 a/ 3 = 0; (b) ^ +^ + = 1.

369. *+4y + 6z= ± 21.


/x * y —a z +a
370. * + z + a = 0, y = - q- = - j — •

371. A square (S = xy, 2x + 2y = /, Lagrange’s function Z, =


= *y + A(2* + 2y—/)).
372. Method 1. Reduce the equation of the ellipse to the canonical form.
The eigenvalues = 9, A2 = 1; 9f2+?72 = 9, £2+ — = 1. Hence, the semiaxes
of the ellipse: a = 1, b = 3; 2a — 2, 2b = 6.
Method 2. The given ellipse is situated symmetrically about the origin,
therefore the square of the distance from the origin to the point of the ellipse
(*, y) equal to *2+ y2 attains the greatest (least) value when the point (*, y) falls
on the major (minor) axis of the ellipse. Therefore it is necessary to investigate
the function u = x 2+ y2 for a conditional extremum with the restraint
5*2+ 8*y + 5y2 = 9. _______________
373. An equilateral triangle. Hint. S = V /(/—*) (l—y) (/-z ) , * + y + z = 2/,
*, yA, z being the sides of the triangle. We substitute the value / —z = * +y —I
into 5, and investigate the obtained function for an ordinary extremum. We
may also investigate the problem by writing Lagrange’s function:
L(S, A) = S + A(* +y + z —21).
154 A Collection o f Problems

Chapter 5.

1
374. (a) S = 1. Hint.
n(n+ 1) n+ 1
13 1
(b) 5
36 * mL (n + l) ( /i+4) 3 (n +1 r t f i) ’
1 1
. 1 r 1 1---- 1— 1 1
a (/i+ 1) (n-\~A) 3 1 22 3 ' 4 AH-2 AM-3 A'' M
(c) 5 - 1/4. Hint.
1____ 1 r 1 __2_ 1 I
n( «+l )( « + 2 ) " 2 l « ' /i + 1 + ai + 2J‘
375. Hint. When proving the divergence of the harmonic series by Cauchy’s
test, consider

,S“ “ ,SJ * ; r r f + , . i 2 + ••• + a -

377. Diverges ( i t i - { H »).


1
378. Diverges —----------^ •
V w2+ 2n *%/ai2+ 2/i2 n

379. (a) Converges, - J f ; (b) converges, — ^ — .

380. (a) Diverges; (b) and (c) converge. Hint. Apply theorem 1 of Sec. 9.4
from [1],
381. Converges. 382. Converges. 383. Converges. 384. Converges.
385. Diverges.
386. Converges for e > 1; diverges for 0 < e 1. 387. Converges.
1In
2 C 1
388. (a) Converges, un ^ ^ - 2 ; (b) converges, un I .v3 dx = 389. Con-
0
verges conditionally. 390. Converges absolutely. 391. Hint. \akbk\
392. (a) jf =- 1; (b) * > 0; (c) - o o < * < 0 0 ; (d) - 1 <
< * < 0, 0 < * < 1.
393. (a) Converges uniformly to zero; (b) converges uniformly to zero;
(c) converges uniformly to zero; (d) converges nonuniformly to zero
( max /„(*) = — -m 0).
395. (a) The given series is convergent for - 1 =ss x < 1. The differentiated
oo
series £ x n~x converges uniformly on the set [-<5, A] for any S < 1. There-
i
oo
fore termwise differentiation is valid on the indicated set. Let S(.x) = £ x n/n, I

then S'C*) = £ x n~ l = 1/(1 —*)• Integrating, we obtain


i
Answers 155

S(x) = J =-In(1 —t) = - I n (1 —x) (-1 1).

(b) The given series is uniformly convergent for | * | S < 1, which can be
checked by D ’Alembert’s test. Therefore it can be integrated termwise:
X

j s « ) d t = [ h r- + ... i■/«"+ ...J1 - !•'» = - 4 - (i.vi^ <5<i).

Differentiating the last equality with respect to x, we get


1 I-* 2
S(*) = ( U |< 1 ) ; (c) S(x) = (I* I < 0-
(I -* )2 (1 +X2)2
396. (a) R = 1 ,( —1,1), for r = ± 1 the series converges; (b) R = 0 ,x = 0;
c) Ji = 1/3, ( —1/3, 1/3), for x = ±1/3 the series diverges.
m . («> * + 1 ?+ (b )» -y + ...; (c) « ( l - y + ...)■

r 3 v 5 r 2 n 11
(b) x ——— —— + ( - l ) n l1 - _+ (“ I 1)
K ) X 32 + 52 V (2/1 + 1)2 +
Hint. Expand the function arc tan x into Taylor’s series in powers of *.
399. 32.831. 400. eT2[l+ £ (1*1 < °°). Hint, e* = e T2ex±2.

Chapter 6.

401. (a) y - 2 x y ' = 0; (b) / ' = 0; (c) / = +; (d) x + y y ' = 0;


(e) y ' —y ' —ly = 0.
402. (a) Isoclines are straight lines x = k (straight lines parallel to they-axis)
JC2
(Fig. 78). The exact solution of the equation is y = — + C.
(b) 1 +>>2 = k—isoclines, 1. These are straight lines parallel to the
Af-axis (Fig. 79).

Fig. 78. Fig. 79.


156 A Collection o f Problems

(c) isoclines: x = - k (Fig. 80).


403. y = C(x + l)e~ x, x = - l .
404. ln |x | = C+y/^+i-
4 0 5 .1 - e ~ ‘ = Ce‘.
406. y = -2 /(1 + C exp ( - * 2)).
407. y = {x —2)3. 408. y = 2 —cos a*.
409. .y = C jc3, 3^ = xy ' is the differen-
lial equation of the family of curves.

410. 0.5 kg. Hint. Write the differential equation of our problem. Let y(t) be
the amount of salt in the tank at time t. Let us find out the change in salt con­
tent during the time At (from the instant t to t + At). Since, by hypothesis, 5
litres of unsalted water is supplied per minute, during the time At, 5 At litres
of water will be supplied. This quantity of water will contain 5 At»0 = 0 kg of
y ( t)
salt. One litre of the solution contains —— kg of salt, hence, the salt content in
the flowing out mixture will amount approximately (with an accuracy up to an
infinitesimal of higher order than At) to

5/1< -— kg = 005 M 0 kg.

Thus, the solution, inflowing during the time At, contains 0 kg of salt, while the
solution, flowing out of the tank during the same time, contains 0.05 At y(t) kg
of salt. The increment of the amount of salt during this time
y(t + A t)~ y(t) « 0 -0 .0 5 At y(t).

Dividing by At and passing to the limit as At -*■ 0, we obtain the differential


equation
/« = - 0 .0 5 * 0 .
The general solution of this equation has the form y — C exp ( —0.05/).
By hypothesis, * 0 ) = 10 kg, hence, C = 10. In / = 1 hour = 60 min we obtain
(when deriving the equation, we supposed the change in time to be in minutes)
*60) = 10 exp ( - 3 ) * 1/2 kg. _

411. t0 = 40 min. Hint. If 0(t) is the temperature of the body at time t, then
the differential equation of the problem will be written as

^ = - m o -20].

The general solution of this equation 0(f) = 20+ C exp ( —kt). The constant C
and the proportionality factor k are found from the conditions: 0(0) = 100,
0(10) = 60, C = 80, k = 0.1 In 2.
Further, 0(/o) = 25, i.e. 25 = 20 + 80 exp ( —0.1/„ In 2).
412. x 2+ C(y + *) = 0; x = 0, 413. y = 0; x ( y —x) = Cy.
414. y = C exp (y/x).
415. The general solution: x 2(y + C) = Cy.
Answers 157

4 16. x ^ C e x p (-g ^ ).

417. The equation is reduced to the form


2y
y' = a^cos2 -+
)•
i.e. a = 2, f ( t ) = cos2/ +2/. The solution is carried out by the substitution
y = **2. We may also take advantage of the formula derived in [3], Sec. 1.3, (7):

* = c exp [/ m = a ] = Cexp [/^ sb ]= c exp(tan0


= C exp (tan

418. Hint. Set up the differential equation of the problem (see Fig. 81).
Let M = (x 9 y) be the point of tangency; M N tangent line; ON _L M N ; by
hypothesis, ON = OP = |* |. If
Y = y = y '( X - x )
is the equation of the tangent, then

ON =
V i +y2
is the distance of the point (0, 0) from the
straight line MN. Thus,
\ x y '- y I
\x\
Vi + /2 ’
whence
(x2—y 2) dx -I- 2xy dy = 0.
This is a homogeneous equation. Its general solution: Cx = y 2+ x 2.
419. The equation is reduced to the form

/ = - ^ ( 4 - x y - y V ) = —- /(x y ),

i.e. a = - 1 , / ( / ) = 4 The general solution:

“ x ' c x 5- x •
420. (a) The equation is reduced to the form

y' = ^ ( 2 + x Y ) ,

i.e. a ——1, / ( / ) = ------(2 + /2). The equation can be solved by the substitu­
tion xy = t or by the formula (7) of Sec. 1.3, [3]:
158 A Collection o f Problems

(put C3C = 1)
2C xli * - l 1 C x llz 1 1 / 1\
y ~ x i C x W - l ) = * + x ( C x 1l * - l ) “ a: + C*x*z + x \ “c )'
(b) Substitution: yx~ a = t.
421. y = C e - ^ + l x - 1. 422. ^ = Ca:2+a:4. 423. ^ = e*(\n \x\+ C ).

424. . 425. = * (C + sin *).

426. x = sin y (C -co s j). The equation is linear with respect to the function
x = x(y):
dx . „
—r~ — sm2.y+;c cot y.
dy
427. y — 0; y 3 = - l / [ 3 cos3 a :(C -Han *)]. 428. j>2 - Ca:2- 2 a . 429. y = 0;
^ = a:4 In2 |Cjc|. 430. ^ = 0; y = l/(x 2+ Cx).
431. (a) Yes. All distance axioms are readily checked.
(b) Yes. The first and second axioms are obvious. Check the triangle in­
equality: q(x9y) gix, z) + q(z, ^).
If x = y = zt then 0 < 0 + 0 ; if a: = .y, z & x, then 0 1 +1 = 2; if a:
x = z, then 1 =s= 0 +1 = 1; if x ** y, x z, y ^ z, then 1 1 4-1 = 2.
432. Yes. The first axiom: if f i x ) = g(*), then g i f g) = 0. Conversely, let
gif, g) = 0. Then
b
j [ f i x ) - g i x ) f d x = 0.

Since [ fix ) -g ix ) ] 2 is a nonnegative continuous function on [at b]9 we have


[ f i x ) —gix)]2 = 0, i.e. f i x ) = gix) (see Sec. 6.2, theorem 8, [1]). The second
axiom: g i f g) = gig, f ) is obvious. The third axiom: we have

f [fix ) + tg(x)]2d x » 0

for V K i e. a quadratic trinomial with respect to A


b b b
J J
f \ x ) dx + 2A f i x ) gix) dx + A2 g2ix) d x J 0.

This is possible if the discriminant of the equation is nonpositivc:


i b \ 2 b b

J
I A x ) gix) d x ) -j P ix ) dx J g \x ) d x 0.

whence
1/2 1/2

i J
a
f i x ) gix) dx P ix ) dx g \x ) dx
Answers 159

(BuniakowskVs inequality for integrals, see also [3], Sec. 4.8). Further, applying
Buniakowski’s inequalty, we have
b

a
J lf(.x)+g{x)fdx

b b
J|
a
f ( x ) +g(x) 11 I |
f(x) d x + j
a
f ( x ) +g(x) |•| I g( x ) d x

, b \ 1/2 r- / b x 1/2 , * v 1/2-j


^Ij Uix) +g(xW d x \ IJ f\x ) dx\ +M g\x) dx)

or
i b \ 1/2 / b \ 1/2 i b \ 1/2

IJ[/(*)+£'to]2 ^IJ/2M +M ^j dx\ dx\

{Minkowski's inequality for integrals). Now, by Minkowski’s inequality, we


obtain

e(/> g) = [ [ [/(*)-s(*)]2^
ft \ 1.'*
{ [ / W - y W ] + [ fix ) - £ « 1 } 2 dx I

ft , 1/2, 4 ,1/2
lf(x )-< p (x )fd x I + ( ( l<p(x)-g(x)]2 d x \ = e(f,<p)+ +ei<P,g)

for any function fix ) continuous on [a, b],


/ >/ » \ V* a- 1
433. a < 1/2. H in t. o(/„, 0) = I J z/2®dx I —n 2.

434. No. The fundamental sequence | 3 —- j converges to the number 3


which does not belong to M.
4 3 5 . (a) Yes. q{F{x), F(y)) = | F(*) - /-’O') I = \x*-y'-\ = \ x - y \ - \ x + y \ «

=s \ x —y \ (|j(l + lj’l) ^ I* —.H = ag(x, y), where a = 2/3 -= 1.


(b) No. I f * = l , y = 0, then |F(Ar)-F0>)| = 1 = 1 \ x - y \ (a = 1).
4 3 6 . AT0 = 1/2,*4 = F (x0) = 1/2*,* 2 = 1/2*, = 1/2*"...........
437. (a) x = 2/(l +\/5). Yes, since
,. 1 4
max | F'(x) | = max —-----^ = — < 1.
l/2 ^ x ^ l( l + xY 9
(b) The axis x 2 = 0; (c) the straight lines x 2 = 0 and x2 = 1.
160 A Collection o f Problems

438. (a) <5 < 1/36. Hint. According to the existence theorem,
<5 < min {a, 1/N , b/M},
where

N = sup 6/ A/ = max |/( x , >>) |.


2> ’ D
In this case M = 36, N = 24. The solution of this problem has the form y =
= 2/(3—lx*). Thus, as x -►\/3 /2 the solution >>(*) o o . Hence, in fact the
solution exists in the interval (0, \ / 3/2), which exceeds the interval (1 —8,
1+8) ( 8 ^ 1/36).
Note that within the entire interval (0, 2) = (1 —a, \+ a ) the solution of this
problem (with the indicated initial conditions!) does not exist.
(b) 8 < a = \/2 /4 . In the present case a = = 4? = ~ . Thus, the
M N S a
solution exists in a minimum possible interval [ - 8, 6] (5 < a), i.e. the existence
theorem yields a nonimprovable result in terms of the size of an interval, where
the solution for the given right-hand side of f ( x , y).
439. y(l) Rs 1.248. Hint. When equations are solved approximately, it is
always recommended to determine the interval (x0—8, * 0+ <$), where the solu­
tion y(x) exists. The number 8 is found from the existence theorem. If the point
at which we are interested in the value of the solution belongs to the indicated
interval, then we may apply Euler’s method. This problem illustrates the appli­
cation of this method. The equation can be solved. Its solution satisfying the
initial condition, has the form y = exp (jc2/4), i.e. the solution exists throughout
the real axis and, therefore, we may apply Euler’s method without any limita­
tions. By Euler’s method,

/ ( i) *= y<>+h Y, /(•**>
*=o
where x 0 = 0, x t = 0.1, . . . , .v» = 0.9, * 10 = 1; = 1, ^1 = ^0 +
+ VC*0, ^0). • • •, = y» + h f( x 8, j 8). y 10 = y» + */(*9. y»)- Hence, v(l) >>!(,.
All these computations can be tabulated:

k xk yk h /( x k, yt)

0 0 1 0
1 0.1 1 0.005
2 0.2 1.005 0.010
3 0.3 *1.015 0.015
4 0.4 1.030 0.021
5 0.5 1.051 0.026
6 0.6 1.077 0.032
7 0.7 1.109 0.039
8 0.8 1.148 0.046
9 0.9 1.194 0.054
10 1 1.248 —
Answers 161

The true value of the solution X I) = exp (1/4) % 1.284, i.e. we have obtained
an approximate value of the solution with the correct first decimal digit.
4 4 0 . y(2) & 4.781 (the exact value of y(2) = 3(e—1)). Hint. The solution of
the equation exists throughout the number axis. The general solution of the
equation: y = Cex—x —1.
4 4 1 . (a) y = C exp (± * ) (Fig. 82). No singular solutions.
(b ) y(x + C)2 = 1, y = 0 (Fig. 83). No singular solutions.
(c) (x+ C )2+ y2 = 1, y = ± 1 are singular solutions (Fig. 84). At each of
their points the integral curves y = ± 1 are touched by one more integral curve
(by a circle). *

Fig. 82.

(d) y [1 -f (* —C)2] = 1; y = 0; y = l i s a singular solution (Fig. 85). Note


that y = 0 is not a singular solution. This integral curve is not touched by other
integral curves.
(e) x 2+<y = 2Cy—parabolas; y = ± x —singular solutions (Fig. 86).
(f) (Cx + 1)2 = 1 - y 2—ellipses; y = ± 1—singular solutions (Fig. 87).
Hint. Singular solutions can be sought for in different ways. For instance, in
(e), solving the equation with respect to y , we obtain the homogeneous equation
11
162 A Collection o f Problems

y ± \ / y 2- x 2
y = (x 7* 0). If the partial
derivative with respect to y of the
right-hand member of the last equation
becomes infinite along a smooth curve,
then this curve can be a singular solu­
tion. In the present case
6 I"y ± \ / y 2- x 2l
*y[ x J

Fig. 86. *L vV-*2J


yi
P T l ~
hi
H

CM
- rT

} A z

Fig. 87.

for y = ± x . By checking, we make sure that y = ± x are solutions of our


equation. It can be readily ascertained that these straight lines are touched at
every point by one more integral curve of the family x 2+ C 2 = 2Cy. Hence,
y = ± x are singular solutions.
The same solutions can be found from the system
x 2-hC2- 2 C y = 0, | (x2 C2—2Cy) = 0,
>y = ± a .
- (x2+ Cs —2Cy) = 0, | C —y = 0.

The further investigation is carried out as above.


442. (a) The given equation does not contain the variable y explicitly. Intro-
dy
ducing the parameter: —- = p, x = p3+p, dy = p d x = p(3pz + l)d p ; we
have
.V - fV1i p,
3
,v = -4 r .U -p2
^+C^

which is the parametric representation of the solution.


(b) The given equation does not contain the variable atexplicitly. Introducing
dy dy
the parameter = p, y = p2+ 2p3, dx = — = (2 + 6p) dp, we obtain the
ax p
solution
x — 2p + 3p^ + (7,
y = p2+ 2/?3,
represented parametrically; y = 0 is also a solution of the equation.
Answers 163

(c) The given equation does not contain y either. The parameter p can be
introduced by the formula — = p. Then x = p V 1 + p 2, dy = p d x =

= p / y i + p 2+ — \ dp, 3y = (2p2- l ) \ / l + p 2+ C. Here the parameter


\ V i+ W
can also be introduced by the formula — = sinh p, x = sinh p ^ /l+ s in b 2p =

= y sinh 2p, dy = sinh p = sinh p *cosh 2p dp = (2 cosh2p —1)</cosh p.

^ = y cosh3p -c o s h p + C.

at = - sinh 2p,
* = pV h -/,
____ or
3y = (2p2- l ) V l + P 2 f-C 2
y = y cosh3 p —cosh p + C

—parametric representation of the solution.


444. y = A :C - y C2 is the general solution; a singular solution is found
from the system

y —xC -h— C 2 = 0, y - x C + — C2 = 0,
4 4
6
+ = 0, -AT+y = 0,
0C
C = 2.v, y = .v2.
We make sure by a check that y = x- is the solution of Clairaut’s equatioa,
hence, this is a singular solution (Fig. 88). As is seen from the figure, the para­
bola y = x 2 is an envelope for the family of straight lines y = x C —y C2.
445. y = C x + V 1 + C 2 is the general solution. For x = 0, y = \ / l + C 2 ^ 1•
Singular solution: x 2+ y 2 = 1. Bearing in mind that the straight lines y = Cx +
+ \ / l + C 2 intersect the y-axis at points with ordinate 1, we conclude that the
upper half of the circle is a singular solution (Fig. 89). This is kn envelope of a
family of straight lines.
164 A Collection o f Probletns

446. (a) y = -c o s x + Cxx + C2\ (b) y = ^ + Cxx 2+ C^x + C3.


447. The given equation does not contain the desired function y in the explic­
it form. Reduction of the order is obtained by introducing a new function:
z(x) = y'. We have z'(x) = y"; x V = z2. Thus, we have obtained an equation
of the first order with variables separable. It is obvious that z(x) = 0 is the
solution of the equation, then y'(x) = 0, y(x) = C is the solution of the original
equation.
Let z ^ 0; then, separating the variables, we obtain
dz dx
~ (x ^ = -+ C i. dy
= —+
x
c i,
x dx
y = ^■— InlC^ + ll+ C , (C ^ O ).
C^cTT

If C x = 0, then z = x, y' = x, y = - - + C .
448. 2y = (1 + 2C 1)jc2+ C 1 cos 2x + C^x +C8. Hint. z{x) = y"(*).
449. y — + Q ) In | C2(x + 01 )| -\-x + C3J y — Q jc + C’2.
4 5 0 . The given equation can be solved by the substitution y'(x) = z(jc). But
it is easily seen that 2yy' = O'2)', therefore the equation can be written in the
form
(/)' = O'2)',
whence
[ _dy_
/ = y*+p. X =

Considering the cases p = 0, p < 0, p > 0, we obtain:


J y 2+ P *
x= 2c1* + C2 = In y~c1
y+ cx
(p —-C J );

y — Cj tan —C2) (p — C2).


y - c 1 17(2(7!)
451. y In | Cxx | = 1; y = C x tan (Cx In | C2x |); C2x =
y + c 1
Hint. Reduce the equation to the form (xy'Y = (y2)'.
4 5 2 . y2 = (Ci+A:)2+ C2. Hint. y y " + y '2 = ( y y j .
4 5 3 . The given equation does not contain the argument * in the explicit form.
The order is reduced by introducing a new function: z(y) = y'(x). Hence,
/ ' ( * ) = z '( y ) - y ' = z '- z .
The equation takes the form
* dz „
2y - j - z — z--f-y2.
dy
This is a homogeneous equation. Solving it, we obtain (y A 0):
z = ± v V + C ir.
Further,
dy
^ =±vy+c^ , = ±(/jc.
dx V W C lV
± * + C2 — In j + j*

The function y(*) = 0 is also a solution.


Answers 165

454. y In \y\ + x + C1y + C2 = 0; y = C. Hint. y'(x) = z(x).


455. x In
y+ c i + C 2;y = C. On introducing a new function
z(j>) = y ', we obtain a linear equation with respect to z(y).
4 5 6 . The given equation contains x and y. But it is a homogeneous second-
degree equation with respect to y, y \ y " . The order is reduced by introducing
a new function z(x) by the formula y' — y z ( y ^ 0). In this case / ' —y(z2+ z’)
and the equation takes the form x z ' = z. Solving this equation, we obtain z —
C\x. Substituting y'/y for z, we obtain a first-order differential equation:
y' - C\xy. Integrating, we get v — Co exp (C,x 2/2). This solution incorporates
the solution y = 0.
457. y = C2x exp ( —C Jx). Hint, y ' = yz(x). 458. y = C 1e~x + Coe3x.
459, y = C 1+ C2e2x + C3e3x.
460. The characteristic equation has the form: k 4—1 = 0. Its roots can be
found by taking the fourth root of unity. But we can also factorize the left-hand
member: (k2- l ) (k2+ 1) = 0, whence kx = - 1 , k2 —1, k3 = - i , kA= i.
Therefore the general solution of the original equation will be
y = C xe~x -f C2e* + C3 cos x + CAsin x.
461. The roots of the characteristic equation: kx = 1, k2 = kz = 2;
y = + e2x(C 2+ C3*).
462. (a) The characteristic equation k 4+ 2k2-f 1 = 0 is readily reduced to the
form (k2+ 1)2 = 0. Hence, it has the roots k x — k 2 = /, k 3 = kA = —i. The
general solution can be written as
y = eix(Cx+ C2x ) + e~ix(C3-f CAx).
Taking advantage of Euler’s formulas, we have
y = (Cxx + C2) cos * + (C3x + CA) sin x.
(b) y = C i f + C # - * .
464. The general solution of a homogeneous equation is already known to us
(see Problem 461). It remains to find the particular solution of the nonhomo-
geneous equation.
(a) The right-hand side of f ( x ) = 2e3x has a special form (see [3], Sec. 1.16),
where k 0 = 3 is not a root of the characteristic equation, therefore the partial
solution
y = Ae3x,
where
A R 3(k) = k* + 5k2+ S k - 4 .

Hence, the general solution of the nonhomogeneous equation has the form
y = C 1ex + e2x(C2^-C3x) + e2x.
(b) k 0 = 1 is a simple root of the characteristic equation, therefore the partial
solution should be sought for in the form y = A xev. Determining the derivatives
of y and substituting them into the equation, we find that A = 4. In general,
we can prove that
A = a /K (k 0),
if/Cx*X= a exp (At0jc) and k 0 is a simple root of the characteristic equation. The
general solution of the nonhomogeneous equation will be written as
y = C1ex+ e2*(CJ + C 3A:)+4xex;
166 A Collection o f Problems

(c) k 0 = 2 is the root of multiplicity 2 of the characteristic equation. There­


fore we seek for a partial solution in the form y — A x2e2x. Determining the
required derivatives of y and substituting them into the original equation, we
find A = 3/2. In general, it is possible to prove that if f( x ) = aek°x and k 0 is the
root of multiplicity 2 of the characteristic equation, then

K(k») *
In this case R2(k) — 6k - 10, a — 3. Thus,

y = + e2'(C , h C3x) + - n :'.

465. See Problem 462. (a) y = -i- e*. (b) Pass to the functions e ±,x according
to Euler’s formulas; k 0 = ± i are the roots of multiplicity 2 of the characteristic
_ 1 _ x2 _ 1
equation, y = - ~ x 2 sin x; (c) y = - — (cos * +sin *); (d) y = sin 2*.
0 . 0 9
466. y = f V ’+ Q e - * '- — . 467. (a) ^ (b) .y =
= (^4jc2+ (c) y = A cos * + 2 ?sin x; (d) y = (A 1x + A 2x 2+ A 3x 3+
+ A ^ + A sxs)ertx; (e) y = A 1x + A 2x* + A 9x 3. Here the right-hand side
P(x)et,z, k 0 = 0 is the root of the characteristic equation, (f) y =
= e*[(A i~t~A 2x) sin x + (A 3+ A ax ) cos x].
468. (a) Linearly dependent: a«l + fism 2x + y cos 2jc = 0 for a = —1,
p = 2, y — 1; (b) linearly dependent; (c) linearly independent, since their
Wronskian is not equal to zero; (b) linearly independent ( W[x2, x 3, x 4] =
= 2x* ^ 0, a: t* 0).
469. (a) y = C xx 2+ C2x 3. Hint. Seek for the partial solutions in the form
y = x k. The functions x 2 and x 3 are partial linearly independent solutions of the
equation, therefore their linear combination yields the general solution of the
equation; (b) y = Cxx + C2x 3; (c) we seek for the solutions in the form y = x k;
the characteristic equation A:2—4A:+4 = ( k —2)2 = 0 has a repeated root
k \ = k 2 = 2; y = x 2 is a particular solution of the equation. We seek for
another solution in the form j = A x2 In x. It is easy to make sure that for any A
this is a solution of Euler’s equation. Hence, the general solution is y =
= (C 1+ C 2In x)x2; (d) the equation turns into Euler’s equation after multiplica­
tion of both sides of the equation by x; y = C 1+ C2x 3+ C 3 \n x .
471. y = extf2(C1 cos x + C2 sin x).
c v2 r r10
4 7 2 . (a )^ = C , x + ^ - + y ; (b) y = Cxx + ^ + ^ .

473. The right-hand sides of the equations have no special form


e**(PJx) cos (ix+ Q Jx ) sin fix).
Therefore the partial solution of the nonhomogeneous equation should be
found by the method of variation of arbitrary constants.
(a) The general solution of the homogeneous equation has the form
y = Cj cos x + C2 sin x.
Regarding C^jc), C2(;t) as functions of *, let us find them so that the function
y(x) = QCy) cos x T C2(.v) sin * be a partial solution of the nonhomogeneous
Answer# w

equation. For this purpose, we have to solve the system (see [3], Sec. 1.17)
I C7(*) cos * + C2(*) sin * = 0.
- C'dx) sin x -f C2(*) cos x = —.
sm *
The determinant of this system is the Wronskian JF[cos *, sin .t] = 1 ^ 0.
Solving the system, we find
r;(.v) - - 1 , C'(.v) - cot x.
Integrating, we obtain

Ci(jc) - —x 9 C2(x) = f
J
cot x dx = f —f-n X = In | sin jc |.
J sin x
Hence,
y = —x c o s * + s in * In Isin jc |,
and the general solution of the nonhomogeneous equation will be written as
y — C x cos* + CZ s in * —* cos* + sin* In |s in * |.

We find now the constants C x and C2 from the initial conditions:


y i n ! 2) = C2 = 1, y ' ( n / 2) = - C x+ nl 2 = 0, Cx = n /2 .
Thus, the solution of Cauchy’s problem will be:

cos* + s in * —* cos* + sin* In Isin * |.

(b) y = Cj cos 2* + C2sin 2* - * cos 2* - y sin 2* + sin 2* In | cos * I.


/ * 71 \
(c) y — C 1+ C2cos* + C3sin*-f-ln tan
(t + t )
—* c o s * + s in * ln |c o s * |.
(d) y = (C1+ C 2x)e~'z + ( —* + * In \x\)e~x = (a + bx)e~x + xe~x In |* |.

474. (a) ^ = C 1^ + C 2e - ^ + y ^ + - x - ^ ,

— ^ +T ‘- * - T + h ~ T r
Hint. Differentiating the first equation, we obtain
y + 2 y + 4 z = 4. (1)

From the first equation we find the function z = * - - i- .v - y .y . The derivative


i is found from the second equation

z = z - y - h, 2 J 2 —
x 2 = x-\-x1- 1 y• - —
3 y.

Substituting this value pf i into (1), we obtain a linear non-homogeneous differ­


ential' equation of the second order with constant coefficients with respect to
the function y(x ):
y + y - 6 y = 4(1 - * - * 2).
168 A Collection o f Problems

(b) The given system can be solved in the same manner as system (a). But we
may also apply the theory set forth in Sec. 1.22 of [3]. Let us reduce the system to
the form

This is a homogeneous system, therefore it is reduced to one and the same equa­
tion with respect to either v or z. We then write the determinant of the system:

Z)(A)
o+j 0-..........
+A
A2+ l
(‘ -a-
The required equation with respect to the function y has the form: jy ~ 0
d*y
or = 0. The general solution of this equation
y — C x cos x 4- C2 sin x.
The function z is found from the first equation:
dy
z = -±- = - C, sin x + C2 cos x .
dx
Since the function z satisfies the same differential equation as the function y,
we could write at once that
z = a cos x + b sin*
and then would choose the numbers a and b so that the functions y and z satisfy
our system (i.e. we substitute the functions y and z into the system and express
the constants a and b in terms of C 1 and C2, or vice versa).
A -1 -1
(c) Z>(A) = -1 A -1 (A—2) (A+1)2.
-1 -1 A
The differential equation with respect, say, to the function x(t) has the form

( A - 2) ( ' + , ) ■ * , „ 0,
and its characteristic equation will be:
( k - 2)(* + l)2 = 0, kx = k2 = — k z = 2.
The general solution of the differential equation:
x(t) = (Cj + C ^ + V .
The functions y(t) and z(t) are expressed in a similar way:
y(t) = (a-1-bt)e~*+ de2t, z(t) = (a + fit)e~l+ ye2t.
Substituting these functions into the system, we find that b = 0 = C2 = 0,
d = y = C 3, a + a = —C l9 where a may be regarded as arbitrary, then a =
= —C1—a. Thus,
j x(t) = C xe~l + Cze \ y{t) - ae~' + C&2',
zit) = +
Answers 169

(d) The given system is nonhomogencous, therefore either of the functions y


and z will have its own equation:

where
°(zr)' -
*,<» - , « „ ( £ ) « , > - « . , ( £ ) ■1* « . , ( s K

'- ( i) i!
MslI , - ) is the cofactor of the element bgi of the determinant

A -1 d

i.e.
D(A) =
1 A
- A2 -|-1
(> -&
A /n (A ) = A, M 21(A) = 1, M 12(A) = - 1 , M 22(A) = A.
Thus,
= ax
-r- • 1 +1

Finally, we obtain the following differential equation with respect to the func­
tion X *)' y+y = x (the function z satisfies the equation z + z = 0). Solving
these equations, we get
y = C\ cos x + C2 sin x + * , z = C2 cos x —Cx sin x.
475. (a) Solution. Method 1. Let us write the characteristic equation
I2 —A 1
A2—6A 4- 5 = 0.
I 3 4 —A
Its roots are: Ax = 1, A2 = 5. Let us find the eigenvectors a 1 = (aj1*, a |1}) and
a 2 = (a52), <42)), corresponding to the eigenvalues Ax = 1 and A2 = 5:
( 2 - 1 ya tp + o# >= 0, a + a^> + 0, = - o# >,
where a^> is an arbitrary number. To make the notation simpler, let us set
a^> = - l,th e n a £ 1) = 1. And so a1 = (1, -1 ). Analogously, form the equation
(2-5M 2>4-oc<
22>= 0
we find a 2 = (1, 3). The solutions of the system will be written in the following
way:
Y\ t ) = {e‘, -e % Y2(0 = {e5t, 3e5t}.
The general solution:
Y(f) = CjYHO + C2\ \ t ) = {CV + - Cxel + 3C2e5'}.
In the expanded form:
yi(t) = C ie<+ C 2e5t, y 2(t) = - Cxe' + 3C&".
Method 2. On having found the roots of the characteristic equation Ax = 1,
A2 = 5, we can write at once the form of the general solution:
, JFiW = C Ye' + C2e>l\ y 2(t) = ael + be5t.
Substituting these function^ into our system, we find a and b expressed in terms
of Cx and C2. Here we escape the process of finding the eigenvectors a 1 and a 2.
170 A Collection o f Problems ^_____

(b) x(t) = C xe - l +C&»9 v(/> =


(c) y = e-x(C\ cos x + C %sin a t ) ,

z = y e-x[(C2- 2C\) cos * - (C j+ 2 C2) sin * J.

(d) £>(A) = —A*+ 1 = 0; A, = 1, A2 = (l + »V 3), A2 = - ? ( l - » \ / 3 ) -


—the roots of the characteristic equation:

■V - r ,^ - l « - « " ( c ,c o > ( V ? < ) + C '1 >ii> ( ^ ? / ) ) ,

, - « ( ^ ,) .in ( £ , ) ) .

. - c „ . + ,-.= (--< ;.VL-- £ . co, ( V i , ) + ci v | z £ ! „„ ( V i , ) ) .

476. The solution of the homogeneous system is already known to us (see


Problem 475, (b)):
‘ x = C j e - '+ C y ', y = 2 C je -'-2 C se3‘.
(a) Regarding C x{t), C2{t) as functions of t, let us choose them so that the
functions
* = y = 2C1(f)e“<—2C2(t)e2t
would be the solutions of the nonhomogeneous system (Lagrange's method o f
variation o f constants). Differentiating these functions and substituting them
into the system, we obtain
j Ci(/)e-* + C ' ( 0 ^ = 1,

| 2C ftfy -‘-2CS(0e* = t. C[(t) = e1, C't(f) = ~ e~*

Integrating, we get

c m = - ( 1 +t)e‘, c m = i ( - 5 + 3 / ) e - 3<;

The general solution of the nonhomogeneous system has the form:

x = V + C Z +i +l , y = 2Cy-‘-2< V >« + - + y .
Answers 171

477. y =-- 2 (-a - - ^ a j -|- . Hint. Seek for the solution in


the form of the series
y = a0+a1x + a 2x 2+ . . . ,
where, by virtue of the initial conditions, a0 — 2, ax — 1.

479. v - 2 -| a
2 12 +
A 2 A 3 A 4*
4««. v - 6 + 6- , . . . •

481. (a) The Lyapunov function can be taken in the form v = a 2 + > ’2 ;

dv dv dx dv dy . . . .
— — -------------------- 1-------------— - = — 2 a 4 — 2 v 4 < ; 0 .
dt dx dt dy dt y
The stationary point is stable.
(b) The Lyapunov function in the form of a quadratic positive definite form
does not suit this problem. Let us find it in the form
v =
with even a and We find the total derivative of v (along the solution a , y)

~ = ax«-'(2y3-x * ) + P y P - \- x -y * +y>).

In order that this function be <= 0 in the neighbourhood of the origin, it is


necessary that the terms of the form Aa" V and yP~*x be absent. Thus, it must
be a = 2, ft = 4. In this case
rl'ij v
— = -2 a 6- 4 / + 4 / = -2 [ A 6 + 2 / ( l - y 2)] <= 0
dt
in a sufficiently small neighbourhood of the origin. Besides, v = a 2 + j > 4 > 0
in the neighbourhood of the origin and v = 0 only for a = y = 0. Therefore, by
Lyapunov’s theorem, the solution a ( / ) = y { t ) = 0 is stable.
(c) For the function v = a 2 + y2
dv
— = 2 x ( x 3- y ) + 2 y ( x + y * ) = 2 x 4 + 2y* > 0

outside the origin. Hence, by Cetaev’s theorem, the zero solution is unstable.
482. (a) A = ^ j j j , axla22—a\2 = 1 > 0; the system is elliptic;
alx < 0, a22 < 0; the stationary point is a stable node.
(b) A = ^ ^ 4); ^11^22—fli2 = 0, the system is parabolic; Ax = an +
+a22 = 5 > 0; the stationary point is unstable.
(c) A = Q g j; axla22—c$2 = —1 < 0, the system is hyperbolic; the
stationary point is unstable.
483. (a) The characteristic equation
3 —A 0
(3 - A) (1 —A) = 0
2 1 —A
has two positive roots A4 = 1, A- = 3, therefore the stationary point is an un­
stable node.
172 A Collection o f Problems

(b) The characteristic equation


1—A 3
= A2+4A + 13 = 0
-6 5 —A
has the roots Ax = —2 —3i, A2 = —2 + 3i. The real part of these roots is nega­
tive, therefore the zero solution is a stable focus.
(c) The characteristic equation
1- A 0
- A2--1 - 0
2 -I - A
has two roots At = - 1 , A2 — I of opposite signs, hcncc, the stationary point is
a saddle.
(d) The characteristic equation
I —2 —A - 5
A2+ 6 = 0
| 2 2 —A
has the complex roots Ax = - i \ / 6, A2 = / V 6 with the real part p = 0,
therefore the stationary point is the centre.
(e) The characteristic equation
1 —A 1
= (1 —A)2 = 0
0 1 —A y
has a positive multiple root Ai = A2 = 1, hence, the stationary point is an un­
stable node.
Chapter 7.

484. —oo < x < oo; F(x) is continuous and differentiable

F'(x) = (sin27rjc —sinTTAc).

485. , f ToT-f~ arc tan ——. Hint. Differentiate the equality with
2cP(a*+b*) 2( P a
respect to the parameter a.
*a
486. (a) F'(x) = 2x exp ( - * 5) - e x p ( - x 2) - J* y 2 exp ( - x y 2) d y ;
X
X

(b) F'(x) = y In (1 + x2)\ (c) F'(x) = - f i x , - x ) +2 j /.'(«, v) dy,


0
where u — y + x , v = y —x. Hint. First make the substitution z = y —x in the
integral.
487. (a) -1 /1 0 ; (b) 1/2. 488. In (25/24). 489. 50.4. 490.12/5.
0 \/* x + A 8 2 —x

491. / = f dx j f( x ,y ) d y + j dx J fix ,y ) d y (Fig. 90).


- 1 _ V it+ 4 0 - \Z ix + 4

1 3p 6 3

1492. / = J dy j f{ x , y ) d x + j dy J f i x ,y ) dx (Fig. 91).


u r/2 l »/2
Answers 173

1 2 2 2

493. / -
1/2
J J dy
M„
J \ x 9y ) d x + ^ dy
l
J
y
f ( x i y)dx (Fig. 92).

1 Vv
494. / = J J dy /( * , y) dx (Fig. 93).
° v;
1 e \/toc Va «y/z-x*
495. j dx J f i x , y )d y + j dx j f i x , y) dy (Fig. 94).
174 A Collection o f Problems

1 2-t/ i Vv
496. I = J dy (x + y2) dx = ~ . 497.I = J J
dy Jty2*/;t =
o y o y
498. ^ -a b c (a + b + c ). 499. 1/48. 500. 16.
a y / 2 \ / 2 a * —** •%/ 3o* —x2 —y*

501. (a) J dx J dy J f ( x , y, z) dz;


- a 's fi - \/la * - x % x 2 + Vi
2a

l* v 5 V ! * * - 1* v«*
(b) J rfjc J dy J f { x , y, z) </z.
—j* V* _-y/»*«_*» J-V/s, -*,- i ‘
502. — toj3.
4
503. / = — nabc. Hint. Since the integrand is even with respect to all the
variables, the given integral is equal to eight integrals over the part of ellipsoid
found in the first octant. We make the substitution:
x = ar cos t cos r,

y = br cos t sin r, ^0 < r 1, 0 < r =s=

z = cr sin *
The Jacobian of this transformation
ID(x>y' I abcr2 cos t>
| D{r> t, r) I

therefore I = 8abc cos t dt.

504. 710*16.
505. Inabfi. Hint. Take into consideration the evenness of the integrand and
introduce the generalized polar coordinates
x — ar cos /, y = br sin t (0 < r ^ 1, 0 < /* :r/2).

The Jacobian — al>r-


D(r, /)
506.
507. 8a2/9. //w*. The domain of integration is half of the cylinder of altitude
a whose base is a semicircle (x —l)2+ y2 1 O > 0 ) . The equation of the semi­
circle (* —l)2+ y 2 = 1 (y > 0) in polar coordinates has the form q = 2 cos <p
(0 <= <p 7i/2) (Fig. 95). Therefore
rr/2 2 cos (p a

* I— J J
0
dtp
0
q2dg J
0
z dz.
Answers 175

508. 4jiRb/\5. Hint. Pass to polar coordinates.


509. if -V lS (Fig. 96). 510. V = \ (Fig. 97).
3 o

511. V = — (Fig. 98).


3
512. V = — jig3. Hint. To compute the corresponding integral, it is suitable
to introduce polar coordinates (Fig. 99). The equation of the semicircle
{x—a f + y 2 — a2 (y > 0) in polar coordinates will be:
Q = 2a cos (p (0 < (p «= n/l).
176 A Collection o f Problems

513. |5 |
li ■\/l+(z'z)2+ (.z',Ydx dy
= JJ 1+ j dx dy = - ^ V a W + a V + tft? ,
D

where D is a triangle
0 as x a |
1
0 ^ y =«£ ( a - x ) — J

514. | S | = 8a2 arc sin (b/a). Hint. By virtue of symmetry, the sought-for
area is equal to the sum of eight areas cut on the spherical surface and found in
the first octant.
a \ \ / a* —**

IS, = 8* f , * .
JJ Va2—x 2—y 2 J J \ / a 2- x 2—y 2

= 8tf j* arc sin — dx = 8a2 arc sin (b/a)

where D is part of an ellipse

*»«•
515. Let (xe, ye) be the centre of gravity. By virtue of symmetry, it is clear
that x e = 0. The area D of half of the ellipse equal to nab/2 is numerically equal
to the mass of the figure, therefore

y‘ = ^ b S S y d x d y = ^ b i dx
D -a
i
0
y d y =%i-
4
516. (a) — 7zab2. Hint. Take advantage of the second Guldin’s

theorem and Problem 515. (b) — nabc. Hint. V —


ill
(v)
dx dy dz ■

ff / x 2 v2 x2 y2
= 2c I I 1 —- 2 —'^2 dx dy, where 5 is an ellipse —2 + ^ ^ 1; then see

Problem 505.
2
517. x e = y „ = 0, Zg = -- a. Hint. See Problem 506. Introducing the
spherical coordinates
x = r cos ip cos tp,
y = r cos ip sin (p, ^0 ^ r ^ a, O ^ ip ^ ^ , 0 < <p =*£ 2^j,
z = r sin ip
Answers , 177

we obtain
2n n /2 a nit
zt = J JJ
0 0 0
r4 cos sin dip d<p dr = — a J
0
sin ip d sin ip.

518. x e = 0, y c — 8/5. Hint. The area of the figure |5 | = 32/3;


2 4-**

y^ i S S y d x d y ^ j 2 S dx /
512
519. — tt. Hint. See Problem 518 and the second Guidin’s theorem.
521. (a) jt/4; (b) o o ; (c) 1/4.
522. F(y) = In (1 +y). Hint. Integral F(y) converges for any y > —1.

7'0) = J e-z(if+l) Jx _
J+ l
converges uniformly for any y ^ y 0 =>• —1, therefore differentiation with respect
to the parameter y under the integral sign is legitimate in the indicated interval.
Taking into account that F(0) = 0, we obtain F(y) = In (y +1).
523. F(y) = \J n y (y > 0). Hint. As we know,

J exp ( —/*)<* =

F'(y) = J exp ( ~ y x 2) dx = - F j* exp ( - t - ) d t = y


Vy
0 0
The last integral is uniformly convergent for y y 0 > 0.

524. Integrating j x v dx = y — with respect to the parameter y between

/? and a, we obtain
r 1 i i r ; 1 r .
jx 'J x * ~ j
* - J Is
.0 o Ip

Hence,
Cx*-xP a+1
I —j------- a* = In ^—r .
J lnjc B+ 1

Note that in the domain 0 < * < 1 , - 1 < / 5 < (y < a t h e original integral con­
verges uniformly.
178 A Collection o f Problems

525. Converges. Hint. Consider the circle with the e-neighbourhood of the
origin Ue(0) removed and pass to polar coordniates:
Zn 1 1
JJ In V x 2+.V2dx dy = JJ r In r dr dtp = 2n J r In r dr
s \u e{0)
. / e 2- 1 ez\ 7i
= 2 n (~ Y ln e - T + - ) ~ - - , e -* 0 .

526. (a) Converges uniformly by virtue of Weierstrass* test

f cos*y r dx 71
| cos xy | 1, = arc tan x
J T + x * ~ dx * J Tl+x 1 Y ’

(b) Converges nonuniformly. The substitution x y / y = u shows that

J
o
y f y exp ( - y x 2) d x = j exp ( - u 2) du,
o
i.e. it is independent of the parameter y. For y = 0 the integral is equal to zero.
Hence, the integral F(y) is a discontinuous function for 0 ^ y <«>. Conse­
quently, the integral converges nonuniformly. It will be uniformly convergent
if 0 < y 0 y < oo.
Chapter 8.

527. y/5 In 2; the equation of AB: y — ~ ( v -4 ). 529. 24.

530. Hint. The equation of the arc of an ellipse:


3(a + b)
y = — V a2—x 2 (0 < x a). Taking x for the parameter, we obtain

J* a* y /a x f (6 2 —a2)a 2 </a*.

531. J r
y ds = 2 1 V i y j
o
1+— dx = y (2 \/2- 0-
532. Pass to polar coordinates: x = g cos <pf y = q sin (p. The equation of the
circle x 2+ y2 = la x or ( x - a ) 2+ y2 = a2 in polar coordinates has the form
Qi= 2a cos (p ( - n / 2 (p tt/ 2). The differential of an arc of this circle will be
ds = V f'(<p)2+f(<pY d(p = V 4«8 sin2 <p+4<72 cos2 <jptfrp = 2«
A n sw ers 179

Therefore
nh
J (x—y) ds —n/2J = (2 a cos2 <p - 2a cos (p sin <p) 2 a dtp
r
71!2

= 4a 2 J cos2 (p dq> = 2 a 2n .
-nl2

b2 a 2b . ' s / a 2—b 2
533. m = ---- b (a => b).
2 2 y / a 2- b 2

535. 2n 2b \ / a 2 + b 2>
4 4
536. xr ee =
= - - o, y, = y a. /fi'/if. The length of the arc of the half-arch

of a cycloid

\r\= J V x ' ( t Y + y ' ( t y d t = 4a;


0
71
x , = — j- J" * '( 0 * + /( 0 ! dt = y a.

y. = jj^ y V x '( .t) 2+ y V ) 2dt = 3 a.

537. I n a - y ' t f + b - . H in t.

=J (** + ^ 2) d t.

539. 2. Take advantage of the property


=J - M r=r1+r2.
r A a
540. (a) 512/15; (b> 64/3; (c) 0. 541. (a) 1; (b) 1; (c) 1.
542. grad u = {2 x + y , 4 y + x , 6z-6}. 543. (a) z2 = x y ; (b) x = y = r.
544. curl a = {0, 0, 0}. 545. curl a = {1, 1, 0}.
546. (a) Has, since curl a = o and R 3 is a simply connected domain;
(b) has, curl a = o; (c) does not have, curl a = {0,1 —y 9 z } o in R z.
548. (a) and (b) are exact differential equations; (c) is not an exact differential
equation ^curl {2—y 9 ^ o or — (2 —y ) s*

549. jc2 + x * y - y * = C. 550. 3 x 2y - y 3 = C . 551. x e ~ '-y 2 = C.


x? 5
552. x + - j p + — = C . H in t. The potential function U (x 9y ) for the vector

*t 3 x 2 -f-y2 2 x 3 ~b5y 1
a= |— , ------------ 1 should be found as the integral of the second kind
of the vector a. Any curve not intersecting the *-axis may be taken for the
180 A Collection o f Problems

integration path. For instance, we may take


the polygonal line connecting the points (1,1),
(x, 1), (jc, y) (x > 0, y > 0) (see Fig. 100). If
the point (x, y) lies in the lower half-plane,
then for the initial point we take any point lying
below the *-axis, for instance, the point (0,
- 1 ).
553. curl {yz, xz, xy) = o; U(x, y, z) = xyz.
The general solution: xyz = C. Any of the
variables can be regarded as a function of the
two other independent variables.
554. x y + x z + y z = C.

555. y(x2+ z2) = C. 556. JJ (x2-hy2) dx dy.


L>
557. JJ e* » (y -x )d x d y. 558. 0. 559. -1 /3 .
Q
560. nR*!2. Hint. Apply Green’s formula and pass to polar coordinates.

561. mQ = y | ( —v d x + x dy)
r
2n
I f 3
= — J {a sin3 t*3a cos2 1 sin f + gcos3 /*3a sin2/cos t)d t =
o
562. curl a = o. This means that in the plane, vector a has a potential
function. The work done by vector a is a line integral of the second kind which
is independent of the path of integration; hence, the work done is also indepen­
dent of the shape of the path of displacement:

A = J ds) J
(a = (2xy dx + x2 dy),
r r
where J 7is an arbitrary curve connecting the points (1,1) and (2, 5). Computing
the integral along a concrete curve, we just obtain the numerical value of the
work done. Here it is better to find the potential function by solving the exact
differential equation: 2 x y d x-hx2dy = 0 which is an equation with the
variables separable. Solving, we obtain U(x, y) — x 2y. Now the work A =
= 1/(2, 5) —1/(1, 1) - 2 0 -1 = 19.

564. — noA.

565. m = J\ / x 2+ y2dS = j j 's/x'^+y2 y / \ -\~z2+ z2d xd y , where is the


s a ________
part of a circle x 2+ yz a \ found in the first quadrant: z = \ / a 2—x 2—y \

” = ___ ___
J 566. 7t2[ a \ / l + a 2+ ln { a - y \/\ + ^2)]. H int. The element of the area of the
helicoid: dS = Iru| • | rJ du dv = \ l l + « 2 du dv.
Answers m

568. Tia*/!. Hint. Reduce each term to a double integral along the correspond­
ing projection on the coordinate planes.
569.0. Hint. Carry out the computation for each of the four faces separately.
For instance, on the lower face S* (an oriented triangle) the outward normal
n(/4) = —k, z = 0. Therefore

J (yz dy dz \-xz dx dz | xy dx dy) — J xy dx dy — — J xy dx dv — —~ ,


,S'f
where “V.
1
At
|0 jr a —x f
Analogously, we proceed for the other faces lying in the coordinate planes.
For the face S f lying in the plane x-h y-h z = a the cosine of the angle made by
the outward normal with the z-axis is determined by the equality

cos (n, z) = l l V l + zl + z2,


therefore

J (yz dy dz I xz dx dz -f xy dx dy)
s*

= j j y z d y d z + j j xz dx dz + JJ xy dx dy = — a*.
^3 a 2 zh
Finally,
*| ^
1

j . . . - / . .
II

J J - ♦ f - +
s* sf St s*

570. diva = 3(x2+ yz + z2). 571. div a = - f ( r ) + f ’(r).

572. div (grad u) = 6.


573. (a) curl a = o;

i j k
0 0 0 f'(r )
(b) curl ( /(r ) *c) = = — ■ -cx r.
dx dy dz r
m Ci f(r )c 2 f(r)c 3
574. (a) 0. Vector a = {yz. zx, xy), therefore div a = 0.

(b) 2 JJJ (x -\-y -f z) dx dy dz; here a = {x2, y2, z2}.

W 2 fff h.re. - ( i . i , ‘ 1, r = \ / x 2+ v2+ z2.


JJJ V x “ + y * + z * \ r rr \
182 A Collection o f Problems'

(d) JJJ Au dx dy d z; here a = grad u ; div (grad u) = Au, where Au =

d2u 6 2u 8 »n
~~ 0JC2 + 0J>2 + 0Z2 *
1—i 1—x-y
575. 1/2; div a - 3; J’j J 3 dx dy dz - 3 J d x J dy j dz.

576. 0 (see Problem 567). 577. 0.


579. —47i. Hint, a = (>>-z, z —*, x - y } , n = { l/\/2 , 0, l/\/2 } ,

| ( O — d x + ( z - x ) dy + ( x - y ) dz) = j*
r a
where S is an ellipse lying in the plane x + z = 1. As is seen from Fig. 33, its
semimajor axis is equal to \ / 2 , and the semiminor axis to 1. The area of this
ellipse is n y / l . Prior to performing necessary computations, write the equation
of the ellipse in the parametric form taking z (0 z < 2) for the parameter:
x = 1 —z ,y = ± y /2 z —z2. We then divide the integral over r into two parts:
for y > 0 andj> < 0. Note that in the first case z varies from 0 to 2 and in the
second from 2 to 0.
580. 0.

Chapter 9.

583. V*» i f a > l ; 0 < j : < 2n, if 0 < : aa .^ 1. Hint. For 0 < a ^ 1 apply
Dirichlet’s test (a* = k~*9fik(x) = cos k x ).
584. As many as desired. 585. —00 < x < 00.
sin nx sin/i*
586. (a) 2 £ (-1 ) * +1 (b) £
n n—1 n
71 4 ^ cos (2k +1) x
U fa (2k + 1)2
^ cos (2k +1)
2 S + \ ) x* S ^ , .sin k x
nr-
71
(d)

71 1 2 ^ cos(2n+l)* , ^ / i\« j sin nx


(e) ' I * ; £ - & ,+ ,? + £ < -■ > • n
(0 If a is an integer, then sin ax; if a is not an integer, then
2 sin an n sin nx
if
, x 2 . , fl cos /ijc 1

4 yi cos (2* + 1) X
588. (a) 2 M
V M n * V ~ (2 * + 1)2
Answers 183

Hint. When expanding the function into a series in terms of sines, we extend the
function f( x ) to ( —n, 0) in an odd way, and then periodically over the entire
number axis (Fig. 101). Further, see Problem 586, (a). When expanding into a
series in terms of cosines we extend the function f( x ) onto ( —;r, 0) in an even
manner (Fig. 102). Then see Problem 586, (c). Note that in this case the extend­
ed periodical function is continuous on the entire axis. The Fourier series
converges uniformly throughout the axis.
oo
sin 2nx 2_ cos (2/1 + 1) x
(b) n—£ 1 In (Fig. 103); (Fig. 104).
71 (2« +1)2 “

589. f ( 2 n + l) - 2cos (2' ~ - jr*).

590. (a) ti; (b) n/4.


591. (a) ||/ ||_ = t / V h (b) ll/ll = V ^ /2 l (c) ll/ll = v V - l ) / 2 ;
(d) l l / l l = l / \ / 3 .
592. 0 < a < 1/2.
593. For any a > 0 the sequence converges to zero nonuniformly
I max f n(x) — 11. For V* ^ 0 converges to zero in the mean.
\0 « * « n J
184 A Collection o f Problems

•*’(■»)=‘\ J \ 0j costsdl =\ / | 0j costsdt =-\/I -~ •


596. f ( x ) { 1, 0 < x < a. Hint. Take advantage of the repeated
0, ,v > a.
Fourier integral

f i x ) = - i cos xs ds J* f i t ) cos ts dt.

valid for the function f i x ) which is piecewise continuous and absolutely inte­
g r a te on (0, oo) (see Problem 595).
597. F{s) = i p in (1 +5) a + sin (5—1) o~|
's /ln l 1+ s s-1 J

598. Q(s, r) = \ Je _ tx-------


sin rx
dx
- V I arc tan Hint.

Differentiating with respect to the parameter r, we obtain ([3], Sec. 4.14, (4)):
oo

— = \ / — f e~,x cos rx dx = \ / — .
dr y J X n s2+ r2
o
Hence,

- V I arc t a n ---- f-C, Q(s, 0) = 0, C = 0.

5 " . (a) (1 - e ~ as) (see Problem 598 and [3], Sec. 4.14, (6)); (b) 63/697;
(c) 51/290.
C hapter 10.
\ 1/2

600. A = I I I«(e, 0 ) - /( 0 ) l2 dd I < ;-^(l-^ )-^ 0 for g+1-0.


( ]'
Hint. The Fourier series of the function fid ) has the form (see Problem
586, (c)):

m -\-n 4 y cos(2/c+1)0
J () 2 (2k + 1)2 •
The harmonic function (in the unit circle) generated by the function f(S ) can be
written in the form ([3], Sec. 5.3):
, a\ 71 4 J...COS (2k + l)f)
u(Q,0) - i - ~ E e » " --------------
(2k -|-1)- •
Answers 185

Hence, on the basis of Parseval’s equality, we obtain

( oo 1 \ 1/2 / 7l2\ 1
4 ?«< ” „=) " '"I4' J
(see Problem 587).

Remark. The harmonic function u(g96),


tends to f(0 ) along each radius (Fig. 105)
as g -►1 —0, also in the ordinary sense, but
the rate of convergence will be somewhat
worse than (1 - g).
Indeed, for a fixed g( 1/2 < g < 1) let us
choose a natural number N so that 1/(N+
+1) < 1 —g 1/Nf then (see Problem 376)
\u ( Q ,0 ) - m \

V 6 ’ (2A: + 1)2
* 1-Q&+1
i - e * +1. +c ? - 1 -
o (2* + l)! *+1 (2k ■+-1)2

■* c(1_e) ? 2F h + ^ ■* c(1_e) ln
=s c(i - g ) in + c ( i - g ) =s c(i - e ) in — — o, e ^ i-o,
i-e i-e
where the constants c are, generally speaking, distinct in different inequalities.
601. Let us expand the function f( x ) into a series in terms of sines (see Prob­
lem 588, (b))
r, v ^ sin 2nx

The solution of the problem under consideration has the form (see [3], Sec. 4.7,
(ID ): o° |
u{x, t) = — exp ( - 4 /i2/) sin 2nx.

Using Parseval’s equality, we have

A = [2 0 - e x p ( ~4m2/))2] ' .

Let us fix / (0 < t -= 1) and choose a natural number N such that 1j ( N I I)2
£.< l/N 2. Then

- Li, i c - <-«»=' I , i f
186 A Collection o f Problems

Applying Lagrange’s theorem and using the estimates from Problem 376, we
obtain
A *=( t ^ (4n2‘
/ ^
4
TT
exP (-4 « 20 )2+ - ^ J
f* \V 11 /2

/# 1 \ 1/2 / 1 \1/J

602.
c(? ,2w!+]vtt) H ,w w ) : ct11* — 0, t - 0.

cos (2n f 1) t sin (2n f 1) /I . _ f,


u ( .> . 0 = - - f ( - ! ) ” ['
«=o L
(see [3], Sec. 5.5, (8), (11)).
603. Using D ’Alembert’s formula, we have

U(x, t) = y [ 1 + (j _ tys + 1 + ^ + /) i] + y [arc tan ( * + / ) - arc tan ( * - /) ]

« « u<r ^ = —
Ji J
f yf® dt
(x -ty t+ y 2
- Z f _
7i J (x -t)’’ I y-
*

-- OO —I

l —X
c d{ y 1 r . l-x , t / + Jfl
= — = — arc t a n ------ arc t a n ------ .
J s£+y" n n L y y J
- l- x
If il — 1/2, then arc tan ((/—*)/>>)+arc tan ((/+ x)/y) = ti/2, i.e. (l2—x 2)/y2 —
= 1—a semicircle (y > 0) of radius / with centre at the origin.

606. u(x91) =
V l+ 4 r
exp
(-£ )•
Chapter 11.

608. ( a) | z | = 5; ( b) | z | = l ; ( c)|z| = 4.
609. (a) z = 2[cos ( —2n/3)-f1 sin ( —2jt/3)] ; (b) 2 (cos (3ji/4) + / sin (3tt/4)).

610. (a) 2em\ (b) 1 -einl2; (c) 2e~ 2ni,2\ (d) 1-exp^a—^ i.
612. (a) 1728; (b) 1.

613. (a) K / i exp j) (k = 0 ,1 , 2, 3); (b) ± 1, ± i.

1^ w _ x —iy2
614. w = x —iy2.
w ww ~~ x*+ y4 *
^2 y2
616. (a) A circle of radius r centred at the point z0; (b) ellipse
a2+b2 = 1,
b2 = a2—c2. Hint. \z —c\ is the distance from the point (c, 0) to z = (*, y).
By definition, ellipse is the locus of a point which moves in a plane so that the
sum of its distances from two fixed points in that plane is constant.
(c) Re (— ) = = 4-» Im ( —) = -0-7-0 = -A • These are
\w / * 2+ 4 jr 2 \ H' / x m+ 4ym 4

ellipses: (a - 1 ) 2 j-4v2 — 1, * + (v I I)2 = 1;


Answers 187

(d) xy = 1—a hyperbola; (e) v = x 2—a parabola.


617. (a) - 1 ; (b) - ( 5 + 120/13.
619. (a) A circle: | w\ = 2 or w2+ v 2 = 4. For z = w = — e - *?, and
e
consequently, the circle | w | = 2 is traced clockwise if z traverses the circle
\z\ — 1/2 anticlockwise.
(b) The ray coming along the bisector of the fourth quadrant from » into 0.
(c) The axis Ov except the point O = (0, 0). If the point z moves along the
.y-axis from —» to +oof then the point w moves first from 0 to +oo, and then
from —oo to 0.
620. lim z = —/.
z —►i
jf2—
621. Does not exist, since Re w = -=—^ has no limit as x -►0, y -►0.
jc2+y2
622. Yes.
623. (a) wr = 2z\ (b) the derivative does not exist at the points belonging to
the z-plane; (c) has no derivative; (d) has the derivative equal to zero only at the
point 0.
624. f \ z ) = cosz, |/'( z ) |,- o = IcosOI = 1, |a r g / '0 0 U o =
= | arg cos 0 1 = 0.
625. (a) z 0; w' = 3z2; (b) at the points z which are not zeros of the func­
tion sinz,i.e. z ^ (k n ,0) (k = 0, ± 1, ± 2, . . . ) ; ( c ) z ^ - l .
626. All the given functions are analytic except (b).
627. (a) /( z ) = z2+2z+C 7, where Im C = 0; (b)]/(z) = z2( 2 -/)/2 +
+ C i, where Im C = 0; (c) /(z ) = u + iv = (C —z In z) /, where v —
= r<p sin <p—r In r cos (p+ C, I m C = 0 and In z = In r+i<p.

629. u = C arc tan — + Cv


x
w - w 1 w3- w 1
630. — , where z l9 z2, z3 and wu w2, w3
w - w t w3- w 2 z - z 2 z3- z 2
are arbitrary triples of real numbers coming in an increasing order.
632. w = z4. 633. (a) w = e*; (b) w = e2*. 635. w = (1 + i ) (1 - z ) .
^ 2 z-5
636. w = —---.
1 0 -z
637. The integrand can be written in the form:
1 + i—2z = (1 —2 * )+ i(l +2y). (a) 2 0 - 1 ) ; (b) - 2 + | i ; (c) - 2 .
638. 2.
639. 1 + cosh 7i—7i(1 + /) sinh7r. Hint. Since the function w = zco sh z is
analytic throughout the plane, we may integrate along path connecting the
points 0 and 7i(l + i). Take advantage of the formulas: coshiz = cosz,
sinh iz = i sin z.
640. (a) 7 -h 19/; (b) - i/e. 641. (a) 7 = 0 , since the integrand is analytic in the
circle |z - 2 | ^ 1 ; (b) I = - y / ; (c) / = — e36.

642. (a) 2 n i; (b) 2 jr j( - e " 1). H in t, j ... = J... + where C ,


C C7, Ot
and C2 are the contours described about the singular points z = - 1 and
z — 0; C, and C2 lie entirely inside the contour C and do not intersect each
other.
188 A Collection o f Problems

644. 645. °o. 646. 0. 647. oo. 648. oo. 649. I. 650. 1.
2e
651. 1.
OO
652. £ ( —l)w(n + 1) zn (| z| < 1). Hint. Differentiate the expansion

1
653. Y, [(“ 1)n 1-1 ~ 2 ~ n ’] z” (|z| -< 1). Hint. First expand the func-
3 0
tion in partial fractions.

655. - - - - + - ( 2 - 3 ) - J ( z - 3)2+ . . . ( |z - 3 | - = - J ) .

656. (a) |z | > 2; (b) |r + l | > 1/4.

657. (a) - j l ( y ) " + g 2 » = g ( l - 2 - » - ‘)z«;


oo _n °° 1 00
(0 E O - l ) r - > .
0z 1 z 0

658' 4 ( 2 - l ) 2 - 4(2^I) + 16“ T ( z - , ) + 6 4 (z'"1)2“


00 MJ-l
= I ( - 1) " ^ T T ( z - 0 n (0 < | z - l | «= 2).
n = —2 Z

It is possible to take advantage of the formula for the coefficients of the


Laurent series

C%= 2M / /(z) ( z - l) » + l ’
y
where y is an arbitrary circle centred at the point z0 = 1 and lying in the annu­
lus 0 ■< | z —1 1 < 2. The function /( z ) = l/(z2—l)2 is analytic in this annulus.
It is also possible to use the method of expanding /(z ) in partial fractions:

i i i i
— 4(2—l)2 4 ( z - l ) + 4 ( z + l) + 4 (z + l)2-

When expanding the functions —— and -— in powers of (z—1), use


z+ 1 (z + l)
the technique applied in Problem 654.
1 z z3 z5
659- T - 3 ! + 5 T - 7 ! + - - (| z1>0)-
i «60. * * + * * + - + L + 4- - + . . . (Iz| =- 0).

661. (a) Removable singularity; (b) pole of the fourth order; (c) essential
singularity.
Answers 189

662„ (a) z = 0—a removable singularity, z = oo—an essential singularity;


(b) z = 0—an essential singularity, z = oo—a removable singularity; (c) z = 0
—an essential singularity, z = oo—a removable singularity; (d) zn = ^#i+ -i-j ni
—simple poles (n = 0, ±1, ± 2, .. .)• Hint. The numbers zn are zeros of the
function w = cosh z. In the neighbourhood of zn the function tanh z can be
represented in the form
cosh ( z - z n)
tanh z =
sinh (z—zn)
(z-zj

z = oo is an essential singularity, since the limit of tanh z, as z ->oo, does not


. . r „ , , r . , sinh iy i sin y
exist: for z = x tanh x -►1, x ->oo; for z = iy tanh z = — =—f- = =

-
cosh iy cos y
= i tan y and for y -►oo the limit does not exist.
665. (a) Res /(z ) = 0, Res /(z ) = 0.
z — oo

_1_
(b) Res /( z ) = 57 , Res /(z )
z=o 3! /=. oo '3!

(c) Res /(* ) = — , Res /( z ) = —


z=>-i 54e z-2 27
(d) R e s/(z) = 1/2, Res /(z ) = —1/2. Here we take advantage of the
z<='2 t = oo
basic theorem on residues. Expansion of the function (z - 2 ) exp ( l/( z —2)) in
powers of z is a rather cumbersome problem.
3 _
667. (a) —— ni; (b) ni; (c) —nil's/!; (d) ni.

669. (a) n /\/2 ; (b) n/ab (,a + b); (c) 2n/3;


^ 1*3-5 . . . (2«—1) *(2ji)!
(d) ^ ^ s-----a =
2 .4 .6 2n 22n(n!)2 *
670. (a) ne~a/2; (b) 0. Hint. Consider the function
*2
(z2+ l)2‘

671* Iz=i t f {x~ e %

Chapter 12.

673. (a) (b) 674. No, since is a periodic function.


p -2 ’ /?2+ 9 ‘
1 1
675. (a) — + (b) 2-
P* ’ P ' ' ' p + 2* Pl+ 9
676. No, since the growth of the function exp (t2) is higher than that of any
exponential function exp (s0f) (for any constant a0) as t -►oo.
p(p2+ m 2+ n*)
677.
(p2+ m 2 + n2) 2 —4m2«2 ’
190 A Collection o f Problems

= COS1f, m
<•> «'> - j & m
= 1, F ( p ) ^ f ( t ) ;
; <b> F(p> - • * “•/(,) -
f ' i f ) = —4 cos3 / sin f = —2 cos21 sin I t

= —(1 +cos 21) sin 2/ = —sin 21— ~ sin 4f;

/'(0 - p2+ 4 2 p2+ 16

680. (a) * ; (b) (see Problem 679).


(P2+ l ) 3
681. (a) In ; (b) 1 In ; (c) -A- In
/>—l z p z p —1
1 1
682. (a) sin t e* sin t —.
p2+ r (p —3)2+ l 1
2 ( p - l ) 2- 6 Q > -l) _ 2Q>—1) (p —4)
(b) (Problem 679).
[ ( p - l ) 2+ l ] 3 (p2-2p+2)3
683. (a) F or the function/(f) = sin t-a0(t) we have/(f) '=? , therefore
p2+ l

by the shift theorem, sin (t—b) a„ ( t—b) = e~^ , (b)


P2+ 1 P -1 *
1 ~ e~ p ... 1 1 +e~nr
6M* <a> p (H ^ ) : <b) p r i C T ^ - Witf.
<*+!)*
L[| sin 1 1; p] = £ f e~pt | sin / 1dt
o J
kn
r (2 * + l)7 i (2Jfc+2)ji

= 1
(2*+l)7t
^ _____ J^-(2A -I- l) p 7 i g — 2kp7i | Q — 2(* + l)j>7l _J_ Q —p(2*+ l)7lj

-?TT
1
fi85‘ ( } P - 1 P2+ 1 ’ ^ P3 ’ P2- 1 “ P2(P2- 1) '
... I f I f 3 1 f6
***• / ( 0 - j; jj 4t 3 J + 6! 5!
687. / ( f ) = —1 + -A. e*+ y cos f —y sin t. Hint. Expand the rational
function F(p) in partial fractions and make use of the table of transforms.
688. (a) /( f ) = y ( e - '- e - * ) ; (b) /( f ) = f - s i n f.
Answers 191

690. (a) e~2t sin /; (b) y + 2e~'sin /.

691. (a) x(t) = (/-f l)e“ ‘; (b) x(t) = ( / - l ) sin t;


3 2 1
(c) x(t) = — + — e~l cos 2 f+ - e~l sin 2t.

692. (a) x(t) = f arc tan f - — In (1 + r2). Hint. First consider the problem
* " = 1, *(0) = jc'(0) = 0. Its solution: xi(t) = t2/2. By Duhamel’s formula,
t
x( t ) = J
0
y ^ f / —t)</t,

(b) y — (sin x - x cos x)/2 ; (c) y — e2x + 4 cos x - 2 sin x —5.


694. (a) x(t) = e‘, y(t) = e*;

(b) x(t) = ^ (1 - t ) - ^ cos 2r + ~ sin 2r,

3 1 1
y(0 = (1 —f) + y cos 2r—— sin 2 /—cos f,

z(r) = cos /;

(c) y(x) = ~ e 2x+ -^-e~2x- e x9 z(x) = — e2* - - - e " 2x- y ex.

695. (a) ~ e ~ ax. Hint.

W (x ); p] - J e -» ' cos </*j - ~ 2

~~ J p 2+ / 2 <z2+ * 2 ™ a2 - p 2 f [p2+ / 2 a2+ r 2] ^


P r1 , f 1 A 11 ^ P a —p 7t 1
= ^ — arct a n ----------arc tan — = ^- — « • ---------= = - --------- .
aM—P LP P a <* J0 2 a£—p £ ap 2a a+p
Hence, I(x) = exp ( -a x ).
2a
(b) L[l{x) ;/?] = J e"** sin** dx j dt

C t cos t , r cos t dt 71
=0J?+ 0
)7 +*=^’'
p 2

71
According to the table of transforms I( x ) = a o ( x — 1), x ^ 1. If x = 1,
TO THE READER

Mir Publishers would be grateful for your comments


on the content, translation and design of this book.
We would also be pleased to receive any other sugges­
tions you may wish to make.
Our address is:
Mir Publishers,
2 Pervy Rizhsky Pereulok,
1-110, GSP, Moscow, 129820, USSR

Printed in the Union o f Soviet Socialist Republics


Printed in the Union o f Soviet Socialist Republics
ABOUT THE AUTHORS

Professor Ya. S. BUGROV, D. Sc.


(Math.), the author of many scientific
papers and monographs on mathemati­
cal analysis.
Academician S. M. NIKOLSKY, a
State Prize Winner, the author of more
than 130 scientific papers and several
monographs including Quadrature For­
mulas, Approximation o f Functions o f
Several Variables and Embedding Theo­
rems and Integral Representation o f
Functions and Embedding Theorems (co­
author). He contributed many funda­
mental results to the theory of approxi­
mation of functions, variational me­
thods for solving boundary-value prob­
lems and functional analysis. His two-
volume textbook A Course o f Mathe­
matical Analysis (for university stud­
ents) was issued in English in 1977 and
reprinted in 1981 by Mir Publishers,
Moscow.
For his monograph Approximation o f
Functions o f Several Variables and Em­
bedding Theorems he was awarded the
Chebyshev Prize of the USSR Aca­
demy of Sciences.
Higher mathematic/

YaS.BUG RO l/ S.M. NIKOLSKY

Other Books in the Series

Fundamentals of Linear Algebra

and Analytical Geometry

Differential and Integral Calculus

Differential Equations. Multiple Integrals.

Series. Theory of Functions of a Complex

Variable

MIR PUBLISHERS /HOSCCW

You might also like